pharm questions

Réussis tes devoirs et examens dès maintenant avec Quizwiz!

4. The nurse is teaching a young adult patient who will begin receiving interferon. Which statement by the patient indicates understanding of the teaching? a. "I may have a low-grade fever while taking this medication." b. "I may have serious cardiovascular side effects because of this drug." c. "I should take antiemetics prior to each dose of this medication. d. "I may need to avoid people who are sick while I'm taking this drug."

ANS: C Antiemetics should be given prior to treatment to prevent nausea from occurring. Fevers are common and are usually high. Cardiovascular side effects tend to occur in older patients. Neutropenia is rare with interferon and does not predispose patients to infection.

4. A patient who is taking aspirin for arthritis pain asks the nurse why it also causes gastrointestinal upset. The nurse understands that this is because aspirin a. increases gastrointestinal secretions. b. increases hypersensitivity reactions. c. inhibits both COX-1 and COX-2. d. is an acidic compound.

ANS: C Aspirin is a COX-1 and COX-2 inhibitor. COX-1 protects the stomach lining, so when it is inhibited, gastric upset occurs. Aspirin does not increase gastrointestinal secretions or hypersensitivity reactions. It is a weak acid.

5. A patient is taking aspirin to help prevent myocardial infarction and is experiencing moderate gastrointestinal upset. The nurse will contact the patient's provider to discuss changing from aspirin to which drug? a. A COX-2 inhibitor b. Celecoxib (Celebrex) c. Enteric-coated aspirin d. Nabumetone (Relafen)

ANS: C Aspirin is used to inhibit platelet aggregation to prevent cardiovascular accident and myocardial infarction. Patients taking aspirin for this purpose would not benefit from COX-2 inhibitors, since the COX-1 enzyme is responsible for inhibiting platelet aggregation. The patient should take enteric-coated aspirin to lessen the gastrointestinal distress. Celecoxib and nabumetone are both COX-2 inhibitors.

14. The nurse is caring for a patient in the post-anesthesia recovery unit. The nurse notes that the patient received atropine sulfate 2 mg 30 minutes prior to anesthesia induction. The patient has received 1,000 mL of intravenous fluids and has 700 mL of urine in the urinary catheter bag. The patient reports having a dry mouth. The nurse notes a heart rate of 82 beats per minute. What action will the nurse perform? a. Administer a fluid bolus. b. Give the patient ice chips. c. Palpate the patient's bladder. d. Reassess the patient in 15 minutes.

ANS: C Atropine can cause urinary retention. The patient's urine output is less than the fluid intake, so the nurse should palpate the bladder to assess for distension. Dry mouth is an expected side effect and does not indicate dehydration.

13. A patient will be discharged on beta blockers. Which skill is essential for the nurse to teach the patient's family? a. How to prepare a low-sodium diet b. Assessments to detect fluid retention c. How to monitor heart rate and blood pressure d. Early signs of changing level of consciousness

ANS: C Because of the action and side effects of beta blockers, heart rate and blood pressure should be monitored frequently.

8. The nurse is caring for a patient who has recently begun taking atenolol (Tenormin) to treat hypertension. The patient reports dizziness, nausea, vomiting, and decreased libido since beginning the medication. What will the nurse do? a. Hold the next dose until the provider can be notified of these side effects. b. Reassure the patient that these symptoms are common and not worrisome. c. Recommend that the patient discuss these effects with the provider. d. Suggest that the patient request a different beta-adrenergic blocker.

ANS: C Beta-adrenergic blockers can cause these side effects, which are often dose-related. Patients experiencing these side effects should be encouraged to discuss them with their providers. Beta blockers should not be discontinued abruptly, or rebound symptoms may occur. Since symptoms may be dose-related, reassuring the patient is not correct. All beta blockers have similar side effects.

3. The nurse is performing a history on a patient who will begin taking bevacizumab (Avastin). Which aspect of the patient's history should be reported to the oncologist treating this patient? a. History of hepatitis b. Hypertension c. Recent surgical history d. Weight loss

ANS: C Bevacizumab carries a boxed warning for gastrointestinal perforations, wound dehiscence, impaired wound healing, hemorrhage, and fistula formation after surgery. The drug should not be used within 28 days after major surgery.

9. A patient who has parkinsonism has been taking carbidopa-levodopa and has shown improvement in symptoms but develops dystonic movements, nausea, and vomiting. Which medication will the nurse expect the provider to order for this patient to replace carbidopa-levodopa? a. Amantadine HCl (Symmetrel) b. Benztropine (Cogentin) c. Bromocriptine mesylate (Parlodel) d. Tacrine (Cognex)

ANS: C Bromocriptine is often used for patients who do not tolerate carbidopa-levodopa. Amantadine is useful for treating Parkinson's disease but does not have sustained effects. Benztropine is given to reduce muscle rigidity and some tremors. Tacrine is used to treat Alzheimer's disease.

6. The nurse is teaching a patient who will begin taking oral theophylline (Theo-Dur) when discharged home from the hospital. What information will the nurse include when teaching the patient about this drug? a. An extra dose should be taken when symptoms worsen. b. Anorexia and gastrointestinal upset are unexpected side effects. c. Avoid caffeine while taking this medication. d. Food will decrease the amount of drug absorbed.

ANS: C Caffeine and theophylline are both xanthine derivatives and should not be taken together because of the increased risk of toxicity and severe adverse effects. Theophylline has a narrow therapeutic range and must be dosed carefully; patients should never increase or decrease the dose without consulting their provider. Gastrointestinal symptoms are common side effects. Food slows absorption but does not prevent the full dose from being absorbed.

3. A patient has a blood pressure of 155/95 mm Hg. The nurse understands that this patient's risk of cardiovascular disease is _____ greater than normal. a. two times b. three times c. four times d. six times

ANS: C Cardiovascular disease (CVD) risk doubles with each increase of 20/10 mm Hg above normal, starting at 115/75 mm Hg. This patient's blood pressure is 40/20 above normal, which increases the risk four times. A blood pressure of 135/85 would be two times greater. The patient's risk would still be four times greater with a blood pressure of 155/70 or 130/95, since systolic and diastolic blood hypertension are each powerful predictors of CVD.

9. A patient has begun taking spironolactone (Aldactone) in addition to a thiazide diuretic. With the addition of the spironolactone, the nurse will counsel this patient to a. continue taking a potassium supplement daily. b. recognize that abdominal cramping is a transient side effect. c. report decreased urine output to the provider. d. take these medications at bedtime.

ANS: C Caution must be used when giving potassium-sparing diuretics to patients with poor renal function, so patients should be taught to report a decrease in urine output. Patients taking potassium-sparing diuretics are at risk for hyperkalemia, so they should not take potassium supplements. Abdominal cramping should be reported to the provider. The medications should be taken in the morning for patients who sleep during the night.

8. The nurse is preparing to give a dose of oral clindamycin (Cleocin) to a patient who is being treated for a skin infection caused by Staphylococcus aureus. The patient has had several doses of the medication and reports having nausea. Which action will the nurse take next? a. Administer the next dose when the patient has an empty stomach. b. Hold the next dose and contact the patient's provider. c. Instruct the patient to take the next dose with a full glass of water. d. Request an order for an antacid to give along with the next dose.

ANS: C Clindamycin should be taken with a full glass of water to minimize gastronintestinal (GI) irritation such as nausea, vomiting, and stomatitis. Giving the medication on an empty stomach will increase the likelihood of GI upset. It is not necessary to hold the next dose or to give an antacid.

12. A patient will begin using an albuterol metered-dose inhaler to treat asthma symptoms. The patient asks the nurse about the difference between using an oral form of albuterol and the inhaled form. The nurse will explain that the inhaled form of albuterol a. has a more immediate onset than the oral form. b. may cause more side effects than the oral preparation. c. requires an increased dose in order to have therapeutic effects. d. will not lead to tolerance with increased doses.

ANS: A Inhaled medications have more immediate effects than oral preparations. As long as they are used correctly, systemic side effects are less common. Less drug is needed for therapeutic effects, since the drug is delivered directly to target tissues. Increased doses will lead to drug tolerance.

19. The nurse is instructing a patient who will take psyllium (Metamucil) to treat constipation. What information will the nurse include when teaching this patient? a. The importance of consuming adequate amounts of water b. The need to monitor for systemic side effects c. The onset of action of 30 to 60 minutes after administration d. The need to use the dry form of Metamucil to prevent cramping

ANS: A Insufficient fluid intake can cause the drug to solidify in the gastrointestinal tract. Psyllium is not digestible, so it does not have systemic side effects. Onset of action for psyllium is between 10 and 24 hours. The dry form can cause cramping.

15. A patient who takes the oral antidiabetic agent metformin (Glucophage) will begin taking levothyroxine (Synthroid). The nurse will teach this patient to monitor for a. hyperglycemia. b. hypoglycemia. c. hyperkalemia. d. hypokalemia.

ANS: A Insulin and oral antidiabetic drugs may need to be increased in patients taking levothyroxine. Patients should be taught to monitor for hyperglycemia, because of the reduced effects of these drugs.

6. The nurse is preparing to administer phenytoin to an 80-year-old patient and notes the following order: IVP phenytoin 50 mg. The nurse will perform which action? a. Administer the undiluted drug through a Y-tube over two minutes. b. Contact the provider to question the route and the dose. c. Dilute the drug in dextrose solution and infuse over 15 to 20 minutes. d. Request an order to administer the drug intramuscularly.

ANS: A Intravenous phenytoin should be administered undiluted through a 3-way stopcock or Y-tubing. In older patients it should be infused at a rate of 25 mcg/min. The dose and the route are appropriate. Phenytoin will precipitate in dextrose solution. Intramuscular injection is very irritating to tissues and is not used.

10. The child who is a candidate for treatment with syrup of ipecac after ingestion of a toxic substance or overdose is the child who has ingested which substance? a. Acetaminophen elixir b. Chlorine bleach c. Kerosene d. Toilet cleanser

ANS: A Ipecac should not be given to patients who have ingested caustic substances or petroleum distillates since regurgitation carries a risk of aspiration. Acetaminophen is not a caustic substance or a petroleum distillate. Chlorine bleach and toilet cleanser are caustic substances. Kerosene is a petroleum distillate.

11. The nurse is teaching a group of parents about the use of syrup of ipecac. Which instruction will the nurse provide? a. "Do not administer ipecac without consulting a poison control center." b. "Expect the onset of emesis to be immediate." c. "Give ipecac with a glass of milk to increase its emetic effect." d. "Use ipecac fluid extract and not ipecac syrup."

ANS: A Ipecac should not be used for caustic substances or petroleum distillates. Ipecac should be given only after determining whether it is safe. The onset of emesis is in 15 to 30 minutes. Ipecac should not be given with milk or carbonated beverages. Ipecac syrup should be used.

10. A patient has symptoms that are characteristic of multiple sclerosis (MS). Which diagnostic tests are likely to be ordered to aid in the diagnosis of this patient? a. Cerebrospinal fluid (CSF) immunoglobulin G and magnetic resonance imaging (MRI) b. CSF proteins and an angiography c. Serum albumin and a computed tomography (CT) scan d. Serum anti-acetylcholine antibodies and x-rays

ANS: A Laboratory tests that may suggest MS include CSF IgG and MRI.

9. A patient is ordered to receive insulin lispro at mealtimes. The nurse will instruct this patient to administer the medication at which time? a. 5 minutes before eating b. 15 minutes after eating c. 30 minutes before eating d. 10 minutes after eating

ANS: A Lispro acts faster than other insulins, and patients should be taught to give this medication not more than 5 minutes before eating.

2. The nurse is caring for a patient who is receiving an intravenous antibiotic. The patient has a serum drug trough of 1.5 mcg/mL. The normal trough for this drug is 1.7 mcg/mL to 2.2 mcg/mL. What will the nurse expect the patient to experience? a. Inadequate drug effects b. Increased risk for superinfection c. Minimal adverse effects d. Slowed onset of action

ANS: A Low peak levels may indicate that the medication is below the therapeutic level. They do not indicate altered risk for superinfection, a decrease in adverse effects, or a slowed onset of action.

9. A patient who takes high-dose aspirin to treat rheumatoid arthritis has a serum salicylate level of 35 mg/dL. The nurse will perform which action? a. Assess the patient for tinnitus. b. Monitor the patient for signs of Reye's syndrome. c. Notify the provider of severe aspirin toxicity. d. Request an order for an increased aspirin dose.

ANS: A Mild toxicity occurs at levels above 30 mg/dL, so the nurse should assess for signs of toxicity, such as tinnitus. This level will not increase the risk for Reye's syndrome. Severe toxicity occurs at levels greater than 50 mg/dL. The dose should not be increased.

9. The nurse is admitting a patient who has been taking minoxidil (Loniten) to treat hypertension. Prior to beginning therapy with this medication, the patient had a blood pressure of 170/95 mm Hg and a heart rate of 72 beats per minute. The nurse assesses the patient and notes a blood pressure of 130/72 mm Hg and a heart rate of 78 beats per minute, and also notes a 2.2-kg weight gain since the previous hospitalization and edema of the hands and feet. The nurse will contact the provider to discuss which intervention? a. Adding hydrochlorothiazide to help increase urine output b. Adding metoprolol (Lopressor) to help decrease the heart rate c. Increasing the dose of minoxidil to lower the blood pressure d. Restricting fluids to help with weight reduction

ANS: A Minoxidil is a direct-acting vasodilator which can cause sodium and water retention. Combining this drug with a diuretic can help reduce edema by increasing urine output. If the patient were tachycardic, a beta blocker might be added. It is not necessary to increase the minoxidil dose or to restrict fluids.

6. A patient who is 7 months pregnant and who has arthritis asks the nurse if she can take aspirin for pain. The nurse will tell her not to take aspirin for which reason? a. It can result in adverse effects on her fetus. b. It causes an increased risk of Reye's syndrome. c. It increases hemorrhage risk. d. It will cause increased gastrointestinal distress.

ANS: A Patients should not take aspirin during the third trimester of pregnancy because it can cause premature closure of the ductus arteriosus in the fetus. It does not increase her risk of Reye's syndrome. Aspirin taken within a week of delivery will increase the risk of bleeding. It can cause gastrointestinal distress, but this is not the reason for caution.

12. The nurse is providing teaching to a patient who will begin taking a cephalosporin to treat an infection. Which statement by the patient indicates a need for further teaching? a. "I may stop taking the medication if my symptoms clear up." b. "I should eat yogurt while taking this medication." c. "I should stop taking the drug and call my provider if I develop a rash." d. "I will not consume alcohol while taking this medication."

ANS: A Patients should take all of an antibiotic regimen even after symptoms clear to ensure complete treatment of the infection. Patients are often advised to eat yogurt or drink buttermilk to prevent superinfection. A rash is a sign of hypersensitivity, and patients should be counseled to stop taking the drug and notify the provider if this occurs. Alcohol consumption may cause adverse effects and should be avoided by patients while they are taking cephalosporins.

11. A patient will begin taking rosuvastatin calcium (Crestor) to treat hyperlipidemia. The patient asks the nurse how to take the medication for best effect. Which statement by the nurse is correct? a. "Increase your fluid intake while taking this medication." b. "Stop taking the medication if you develop muscle aches." c. "Take the medication with food to improve absorption." d. "You may increase dietary fat while taking this medication."

ANS: A Patients taking antihyperlipidemics should be advised to increase fluid intake. It is not necessary to take with food. Patients should never stop taking a statin without consulting the provider. Patients should continue a low-fat diet while taking statins.

5. The nurse is teaching a patient who will begin taking bethanechol (Urecholine). Which statement by the patient indicates a need for further teaching? a. "Excessive sweating is a normal reaction to this medication." b. "Excess salivation is a serious side effect." c. "I should get out of bed slowly while taking this drug." d. "I will not take the drug if my heart rate is less than 60 beats per minute."

ANS: A Patients taking bethanechol should be instructed to report increased salivation and diaphoresis since they can be early signs of overdosing. They should also be taught to rise slowly to avoid orthostatic hypotension and to hold the drug if their heart rate is low.

8. The nurse is teaching a patient about rifampin. Which statement by the patient indicates understanding of the teaching? a. "I should not wear soft contact lenses while taking rifampin." b. "I will need regular eye examinations while taking this drug." c. "I will report orange urine to my provider immediately." d. "I understand that renal toxicity is a common adverse effect."

ANS: A Patients taking rifampin should be warned that urine, feces, saliva, sputum, sweat, and tears may turn a harmless red-orange color. Patients should not wear soft contact lenses to avoid permanent staining. Regular eye exams are necessary for patients who receive isoniazid and ethambutol. Orange urine is a harmless side effect and does not need to be reported. Renal toxicity is not common with rifampin.

10. A patient is taking clopidogrel bisulfate (Plavix). When teaching this patient about dietary restrictions while taking this medication, the nurse will instruct the patient to avoid excessive consumption of which food? a. Garlic b. Grapefruit c. Green, leafy vegetables d. Red meats

ANS: A Patients taking this drug may experience increased bleeding when taken with garlic. There is no restriction for grapefruit as there is with many other medications. Green, leafy vegetables should be restricted in patients taking warfarin. Red meats are not contraindicated.

5. The nurse is teaching a patient who will begin taking butabarbital (Butisol). What information will the nurse include when teaching this patient? a. "Avoid alcohol while taking this drug." b. "This drug may be used long-term." c. "This medication will take effect immediately." d. "You will not experience a hangover effect."

ANS: A Patients who are taking barbiturates should avoid alcohol. Barbiturates are for short-term use. Butabarbital has a sleep onset time of 1 hour, so it will not help patients fall asleep. Patients who take barbiturates frequently experience a hangover effect.

14. A young adult female who is taking metronidazole (Flagyl) to treat trichomoniasis calls the nurse to report severe headache, flushing, palpitations, cramping, and nausea. What will the nurse do next? a. Ask about alcohol consumption. b. Reassure her that these are harmless side effects. c. Tell her that this signals a worsening of her infection. d. Tell her to go to the emergency department immediately.

ANS: A Patients who are taking metronidazole can experience a disulfiram-like reaction when they drink alcohol. These are not harmless adverse effects or a sign of worsening of her infection.

7. The nurse is preparing to assist with blood collection on a newly admitted patient who has been taking phenytoin for several years. The provider has ordered a complete blood count and liver function tests. Which other blood test will the nurse discuss with the provider? a. Blood glucose b. Coagulation studies c. Renal function tests d. Serum electrolytes

ANS: A Patients who have taken hydantoins for long periods might have an elevated blood sugar. The nurse should discuss this test with the provider.

4. A patient will begin using ipratropium bromide (Atrovent), albuterol (Proventil), and an inhaled glucocorticoid medication (steroid) to treat chronic bronchitis. When teaching this patient about disease and medication management, the nurse will instruct the patient to administer these medications in which order? a. Albuterol, ipratropium bromide, steroid b. Albuterol, steroid, ipratropium bromide c. Ipratropium bromide, albuterol, steroid d. Steroid, ipratropium bromide, albuterol

ANS: A Patients who use a beta agonist should be taught to use it 5 minutes before administering ipratropium bromide, and ipratropium bromide should be given 5 minutes prior to an inhaled glucocorticoid. This helps the bronchioles to dilate so the subsequent medication can be deposited in the bronchioles for improved effect.

16. The nurse is preparing to administer a first dose of clopidogrel (Plavix) to a patient. As part of the history, the nurse learns that the patient has a previous history of peptic ulcers, diabetes, and hypertension. The nurse understands it will be necessary to notify the provider and obtain an order for a. a proton pump inhibitor (PPI) medication. b. frequent serum glucose monitoring. c. increased antihypertensive medications. d. nonsteroidal antiinflammatory medications.

ANS: A Patients with a previous history of peptic ulcer are at increased risk for gastric bleeding and should take a PPI or histamine2 blocker to prevent this. There is no indication for increased glucose monitoring or an increase in antihypertensive drugs. Nonsteroidal antiinflammatory drugs are contraindicated.

9. The nurse is caring for a patient who is receiving vincristine (Oncovin), a plant alkaloid chemotherapeutic agent, to treat non-Hodgkin's lymphoma. The nurse observes that the patient has difficulty walking. What action will the nurse take? a. Ask about numbness or tingling in the fingers and toes. b. Assess heart rate and blood pressure to evaluate for orthostatic hypotension. c. Assess the temperature to evaluate for infection. d. Request an order for a complete blood count and electrolytes.

ANS: A Peripheral neuropathy can occur with this drug and is manifested by difficulty walking and numbness and tingling in the fingers and toes. Orthostatic hypotension is not a side effect. Infection is always a concern, and regular evaluation of complete blood count and electrolytes is performed but not related to signs of peripheral neuropathy.

20. A parent of a child who has been taking valproic acid (Depakote) for several years calls the clinic to report a recent recurrence of seizures and states that the child is having 3 or 4 seizures per week. The nurse will perform which action? a. Ask the parent about to describe the child's drug regimen. b. Request an order for a serum valproic acid level. c. Suggest that the parent take the child to the emergency department. d. Tell the parent that the provider will increase the child's dose of Depakote.

ANS: A Questions pertaining to medication adherence are a no-cost, non-invasive way of troubleshooting cause of decreased drug effect. The serum drug level will be assessed next. Children may need changes in doses as they grow. The child is not in status epilepticus so does not need to go to the emergency department. The dose will not be increased until the serum drug level is known.

5. The nurse is caring for an infant who has respiratory syncytial virus (RSV) and who will receive ribavirin. The nurse expects to administer this drug by which route? a. Inhalation b. Intramuscular c. Intravenous d. Oral

ANS: A Ribavirin is given by inhalation to treat RSV. Oral ribavirin is used to treat hepatitis C, and intravenous ribavirin is used to treat hepatitis C and Lassa fever.

10. A patient who has parkinsonism will begin taking selegiline HCl (Eldepryl) to treat symptoms. What information will the nurse include when teaching this patient about this drug? a. "Avoid consuming foods that are high in tyramine." b. "This drug will prevent the need to take levodopa." c. "You may have red wine with dinner on occasion." d. "You will not have serious drug interactions with this drug."

ANS: A Selegiline (Eldepryl) inhibits monoamine oxidase-B, and it has similar adverse reactions to other monoamine oxidase inhibitors. Patients should be cautioned against consuming foods containing tyramine because of the risk of hypertensive crisis. Red wine is high in tyramine. Use of this drug may delay, but will not prevent, the need for levodopa. Severe adverse drug interactions may occur between this drug and tricyclic antidepressants.

10. The nurse is teaching a parent about methylphenidate (Ritalin) to treat attention deficit/hyperactivity disorder (ADHD). Which statement by the parent indicates understanding of the teaching? a. "I should consult a pharmacist when giving my child OTC medications." b. "I will only give my child diet soft drinks while administering this medication." c. "Medication therapy means that behavioral therapy will not be necessary." d. "Weight gain is a common side effect of this medication."

ANS: A Since many OTC medications contain stimulants, parents should consult a pharmacist or the provider before giving them with methylphenidate. Diet soft drinks often contain caffeine, a stimulant, and should be avoided with methylphenidate use. Behavioral therapy should still be an essential part of the treatment for ADHD. Weight loss is common.

12. A patient has been taking spironolactone (Aldactone) to treat heart failure. The nurse will monitor for a. hyperkalemia. b. hypermagnesemia. c. hypocalcemia. d. hypoglycemia.

ANS: A Spironolactone is a potassium-sparing diuretic and can cause hyperkalemia.

14. The nurse is caring for a patient whose provider has just ordered a switch from atenolol (Tenormin) to reserpine. When preparing the patient to take this medication, what will the nurse do? a. Ask about herbal supplements. b. Counsel that NSAIDs are safe to take with reserpine. c. Teach about potential side effects of mood elevation and euphoria. d. Tell the patient to expect immediate therapeutic effects.

ANS: A St. John's wort may antagonize hypotensive effects of reserpine. Reserpine should not be taken with NSAIDs. Side effects include depression, not mood elevation. Therapeutic effects may take 2 to 3 weeks.

5. The nurse is caring for a patient who is receiving warfarin (Coumadin) and notes bruising and petechiae on the patient's extremities. The nurse will request an order for which laboratory test? a. International normalized ratio (INR) b. Platelet level c. Partial thromboplastin time (PTT) and activated partial thromboplastin time (aPTT) d. Vitamin K level

ANS: A The INR is the test used most frequently to report prothrombin time results in patients taking warfarin. Warfarin is not an antiplatelet drug, so platelet levels are not indicated. PTT and aPTT are used to monitor heparin therapy. Vitamin K is an antidote for warfarin; levels are not routinely checked.

14. A patient is receiving a thrombolytic medication. The patient calls the nurse to report having bloody diarrhea. The nurse will anticipate administering which medication? a. Aminocaproic acid (Amicar) b. Enoxaparin sodium (Lovenox) c. Protamine sulfate d. Vitamin K

ANS: A The antithrombolytic drug aminocaproic acid is used to treat hemorrhage. Nurses giving thrombolytic drugs should monitor patients for bleeding from the mouth and rectum. Enoxaparin is given for disseminated intravascular coagulation. Protamine sulfate is an antidote for heparin. Vitamin K is an antidote for warfarin.

8. A patient who has travelled to an area with prevalent malaria has chills, fever, and diaphoresis. The nurse recognizes this as which phase of malarial infection? a. Erythrocytic phase b. Incubation phase c. Prodromal phase d. Tissue phase

ANS: A The erythrocytic phase of malarial infection occurs when the parasite invades the red blood cells and is characterized by chills, fever, and sweating.

13. The nurse is caring for a patient who takes low-dose erythromycin as a prophylactic medication. The patient will begin taking cefaclor for treatment of an acute infection. The nurse should discuss this with the provider because taking both of these medications simultaneously can cause which effect? a. Decreased effectiveness of cefaclor. b. Increased effectiveness of cefaclor. c. Decreased effectiveness of erythromycin. d. Increased effectiveness of erythromycin.

ANS: A The interaction of cefaclor and erythromycin will produce a decrease in the action of the cefaclor.

1. A patient is receiving interferon alpha (Roferon-A) subcutaneously. The patient experiences chills, fatigue, and malaise, and the nurse assesses a temperature of 102° F. The nurse will notify the provider of the temperature and will anticipate which order? a. Administer acetaminophen (Tylenol). b. Change to intravenous interferon alpha. c. Give diphenhydramine (Benadryl). d. Obtain a serum BUN and creatinine level.

ANS: A The major side effects of interferon are flulike symptoms with chills, fever, fatigue, malaise, and myalgia. Acetaminophen is given to treat this initially. Changing to an IV form does not alter the side effects. Diphenhydramine is given for nausea caused by interferon alpha. It is not necessary to obtain laboratory work when these symptoms initially occur.

15. The nurse is caring for a 70-year-old patient who has recently begun taking amlodipine (Norvasc) 5 mg/day to control hypertension. The nurse notes mild edema of the patient's ankles, a blood pressure of 130/70 mm Hg, and a heart rate of 80 beats per minute. The patient reports flushing and dizziness. The nurse will notify the provider and a. ask to decrease the dose to 2.5 mg/day. b. discuss twice daily dosing. c. request an order for a diuretic. d. suggest adding propranolol to the regimen.

ANS: A This patient is experiencing side effects of the medication. Elderly patients often require lower doses, so the nurse should ask about a dose reduction. Older adults generally require 2.5 to 5.0 mg/day. Twice daily dosing is not recommended. Unless edema persists, a diuretic is not indicated.

18. A patient reports having three to four stools, which are sometimes hard, per week. The nurse will perform which action? a. Recommend increased fluids and dietary fiber. b. Request an order for a laxative as needed. c. Request an order for a stool softener. d. Suggest discussing chronic constipation with the provider.

ANS: A This patient is having stools that are within the normal range for frequency. Nonpharmacologic measures should be used first to help soften stools.

14. A patient has recently begun taking carbamazepine (Tegretol) as an adjunct medication to treat refractory seizures. The patient has a serum carbamazepine level of 18 mcg/mL. What action will the nurse take? a. Ask the patient about usual dietary preferences. b. Reassure the patient that this is a therapeutic drug level. c. Report a subtherapeutic drug dose to the provider. d. Suspect a drug-drug interaction.

ANS: A This patient's carbamazepine level is high. When taken with grapefruit juice, an interaction may occur that causes toxicity. The nurse should question the patient about food and fluid preferences. The therapeutic level is 5 to 12 mcg/mL. This is a toxic level, not subtherapeutic.

11. A patient experiences a blood clot in one leg, and the provider has ordered a thrombolytic medication. The patient learns that the medication is expensive and asks the nurse if it is necessary. Which response by the nurse is correct? a. "The drug will decrease the likelihood of permanent tissue damage." b. "This medication also acts to prevent future blood clots from forming." c. "You could take aspirin instead of this drug to achieve the same effect." d. "Your body will break down the clot, so the drug is not necessary."

ANS: A Thrombolytic medications are given primarily to prevent permanent tissue damage caused by compromised blood flow to the affected area. Thrombolytics do not prevent clots from forming. Aspirin prevents, but does not dissolve, clots. Although the body will break down the clot, the drug is needed to prevent tissue damage due to active ischemia.

2. A patient who has breast cancer tells the nurse that a cousin who had breast cancer received trastuzamab (Herceptin) and wonders why this drug is not given to her. The nurse will explain that her cancer cells do not have a. significant HER2 receptors. b. epidermal growth factor receptor-tyrosine kinase. c. the BRCA1 suppressor gene. d. VGEF proteins.

ANS: A Trastuzamab acts by binding to the HER2 protein on the surface of cancer cells that overexpress this receptor. If patients do not have this overexpression, this targeted therapy will not work. Gefitinib is used when EGFR-TK are present. Bevacizumab is used when VGEF proteins are present.

3. The nurse is caring for a patient who has migraine headaches. The patient reports having these headaches more frequently. Which is an appropriate recommendation for this patient? a. "Avoid chocolate and caffeine." b. "Engage in strenuous exercise." c. "Have a glass of red wine with dinner." d. "Take ibuprofen prophylactically."

ANS: A Triggering factors for migraine headache include foods such as chocolate, caffeine, and red wine. Intense physical exertion can trigger migraines. Prophylactic ibuprofen is not indicated.

19. The nurse is preparing to administer methocarbamol (Robaxin) to a patient who is experiencing acute muscle spasms. The nurse notes that the patient's urine has turned black. What will the nurse do? a. Administer the next dose of methocarbamol since this is a harmless side effect. b. Contact the provider to discuss changing to cyclobenzaprine (Flexeril). c. Obtain an order for a complete blood count to evaluate blood loss. d. Request an order for liver function tests since this indicates hepatotoxicity.

ANS: A Urine may turn green, brown, or black in patients taking methocarbamol, and this is a harmless side effect. There is no need to change medications or order lab tests.

13. The nurse is caring for a patient who was admitted with a fractured leg and for observation of a closed head injury after a motor vehicle accident. The patient reports having pain at a level of 3 on a 1 to 10 pain scale. The nurse will expect the provider to order which analgesic medication for this patient? a. Acetaminophen (Tylenol) PO b. Hydromorphone HCl (Dilaudid) IM c. Morphine sulfate PCA d. Transdermal fentanyl (Duragesic)

ANS: A Use of opioid analgesics is contraindicated for patients with head injuries because of the risk of increased intracranial pressure. If opioids are necessary because of severe pain, they must be given in reduced doses. This patient is experiencing mild pain, so acetaminophen is an appropriate analgesic.

6. A patient who takes digoxin to treat heart failure will begin taking a vasodilator. The patient asks the nurse why this new drug has been ordered. The nurse will explain that the vasodilator is used to a. decrease ventricular stretching. b. improve renal perfusion. c. increase cardiac output. d. promote peripheral fluid loss.

ANS: A Vasodilators are given to decrease venous blood return to the heart, resulting in decreased cardiac filling and decreased ventricular stretching, in turn reducing preload, contractility, and oxygen demand on the heart.

1. A patient describes having vivid dreams to the nurse. The nurse understands that these occur during which stage of sleep? a. Rapid eye movement (REM) sleep b. Stage 2 nonrapid eye movement sleep c. Stage 3 nonrapid eye movement sleep d. Stage 4 nonrapid eye movement sleep

ANS: A Vivid dreams occur during REM sleep.

5. The nurse is preparing to administer intravenous monoclonal antibodies to a patient who has cancer. What is an important nursing action for this patient? a. Having resuscitation equipment readily available b. Monitoring the patient's renal function during the infusion c. Observing the patient closely for development of a rash d. Performing careful intake and output

ANS: A When administering monoclonal antibodies intravenously, resuscitation equipment should be nearby, and nurses should stay with the patient for the first 15 minutes of the infusion.

3. A patient is taking esomeprazole (Nexium) 15 mg per day to treat a duodenal ulcer. After 10 days of treatment, the patient reports that the pain has subsided. The nurse will counsel the patient to a. continue the medication for 4 more weeks. b. reduce the medication dose by half. c. stop taking the medication. d. take the medication every other day.

ANS: A With treatment, ulcer pain may subside in 10 days, but the healing process may take 1 to 2 months. Patients should be counseled to take the drug for the length of time prescribed. Reducing the dose or taking less frequently is not indicated.

7. The nurse is caring for a patient who will begin taking atenolol (Tenormin). What information will the nurse include when teaching the patient about taking this medication? a. The drug must be taken twice daily. b. The patient must rise slowly from a chair or bed. c. The medication is safe to take during pregnancy. d. Use NSAIDs as needed for mild to moderate pain.

ANS: B The side effects commonly associated with beta blockers include bradycardia, hypotension, and dizziness. Patients should be instructed to use caution when rising from a sitting or lying position to avoid orthostatic hypotension. Atenolol may be taken once daily. Atenolol is contraindicated in the pregnant patient. NSAIDs decrease the effects of beta blockers and should be avoided.

18. The nurse is preparing to administer digoxin to a patient who has a serum digoxin level of 2.5 ng/mL. The patient takes 0.25 mg of digoxin per day. What action will the nurse take? a. Administer the next dose as ordered. b. Notify the provider of digoxin toxicity. c. Request an order to decrease the digoxin dose. d. Suggest that the patient may need an increased digoxin dose.

ANS: B The therapeutic range of digoxin is between 0.5 and 2 mg/mL. This patient's level is high, indicating toxicity. The nurse should not give the next dose or request a change in dose.

4. A patient with a peptic ulcer has been diagnosed with H. pylori. The provider has ordered lansoprazole (Prevacid), clarithromycin (Biaxin), and metronidazole (Flagyl). The patient asks the nurse why two antibiotics are needed. The nurse will explain that two antibiotics a. allow for less toxic dosing. b. combat bacterial resistance. c. have synergistic effects. d. improve acid suppression.

ANS: B The use of two antibiotics when treating H. pylori peptic ulcer disease helps to combat bacterial resistance because H. pylori develops resistance rapidly. Giving two antibiotics, in this case, is not to reduce the dose or to cause synergistic effects. Antibiotics do not affect acid production.

7. The nurse is caring for a patient who is receiving intravenous theophylline. The patient complains of headache and nausea. The nurse will contact the provider to a. change the medication to an oral theophylline. b. obtain an order for a serum theophylline level. c. request an order for an analgesic medication. d. suggest an alternative methylxanthine medication.

ANS: B Theophylline has a narrow therapeutic index and a risk for severe symptoms with toxic levels. When patients report symptoms of theophylline adverse effects, a serum drug level should be obtained. Giving an oral theophylline would only compound the problem if the patient has a toxic drug level. Analgesics may be used, but only after toxicity is ruled out. Adding a different methylxanthine will compound the symptoms and will likely result in drug interaction or unwanted synergism.

13. The nurse is caring for a patient who has been taking an NSAID for 4 weeks for osteoarthritis. The patient reports decreased pain, but the nurse notes continued swelling of the affected joints. The nurse will perform which action? a. Assess the patient for drug-seeking behaviors. b. Notify the provider that the drug is not effective. c. Reassure the patient that swelling will decrease eventually. d. Remind the patient that this drug is given for pain only.

ANS: B This medication is effective for both pain and swelling. After 4 weeks, there should be some decrease in swelling, so the nurse should report that this medication is ineffective. There is no indication that this patient is seeking an opioid analgesic. The drug should be effective within several weeks. NSAIDs are given for pain and swelling.

16. A patient who is unconscious and has a pulse is brought to the emergency department. The patient is wearing a Medic-Alert bracelet indicating type 1 diabetes mellitus. The nurse will anticipate an order to administer a. cardiopulmonary resuscitation (CPR). b. glucagon. c. insulin. d. orange juice.

ANS: B This patient is most likely hypoglycemic and will need a carbohydrate. Glucagon is given parenterally if patients are unable to ingest a carbohydrate, such as orange juice. CPR is not indicated. Insulin will compound the hypoglycemia.

9. The nurse is performing an admission assessment on a stable patient admitted after a motor vehicle accident. The patient reports having "bad pain." What will the nurse do first? a. Administer acetaminophen (Tylenol). b. Ask the patient to rate the pain on a 1 to 10 scale. c. Attempt to determine what type of pain the patient has. d. Request an order for an intravenous opioid analgesic.

ANS: B To ascertain severity of pain, the nurse should ask the patient to rate the pain on a scale of 1 to 10. Further assessments include location and type of pain. Pain medication should be given after the severity of pain is assessed so that an appropriate analgesic may be given.

3. A patient reports difficulty falling asleep most nights and is constantly fatigued. The patient does not want to take medications to help with sleep. What nonpharmacologic measure will the nurse recommend? a. "Exercise in the evening to promote bedtime fatigue." b. "Get out of bed at the same time each morning." c. "Have a glass of wine at bedtime to help you relax." d. "Take daytime naps to minimize daytime fatigue."

ANS: B To promote sleep, patients should be advised to arise at the same time each morning to establish a routine. Patients should avoid strenuous exercise before bedtime. Patients should not consume alcohol 6 hours before bedtime. Patients should not take daytime naps.

8. The nurse is caring for a patient who will begin taking theophylline at home. During the assessment, the nurse learns that the patient smokes. The nurse reports this to the provider and will expect the provider to a. decrease the dose of theophylline. b. increase the dose of theophylline. c. keep the theophylline dose as ordered. d. discontinue the theophylline.

ANS: B Tobacco smoking increases the metabolism of theophylline, so the dose should be increased. Decreasing the dose will lead to subtherapeutic effects.

19. Which topical antifungal medication is used to treat vaginal candidiasis? a. Haloprogin (Halotex) b. Miconazole (Monistat) c. Oxiconazole (Oxistat) d. Terbenafine HCl (Lamisil)

ANS: B Topical miconazole is used to treat vaginal candidiasis.

8. The nurse is teaching a patient about using high-dose aspirin to treat arthritis. What information will the nurse include when teaching this patient? a. "A normal serum aspirin level is between 30 and 40 mg/dL." b. "You may need to stop taking this drug a week prior to surgery." c. "You will need to monitor aspirin levels if you are also taking warfarin." d. "Your stools may become dark, but this is a harmless side effect."

ANS: B Aspirin should be discontinued prior to surgery to avoid prolonged bleeding time. A normal serum level is 15 to 30 mg/dL. Patients taking warfarin and aspirin will have increased amounts of warfarin, so the INR will need to be monitored. Tarry stools are a symptom of gastrointestinal bleeding and should be reported.

10. A patient with cancer is receiving pegfilgrastim (Neulasta). The patient reports bone pain, which the nurse recognizes as a. a sign of cancer metastasis. b. an indication of expansion of bone marrow. c. caused by osteomyelitis. d. worsening neutropenia.

ANS: B Bone pain is common with these drugs and is caused by expansion of the bone marrow. It does not indicate metastasis. The bone pain is not due to osteomyelitis or neutropenia.

12. The nurse is working in a neonatal intensive care unit and is caring for an infant who is experiencing multiple periods of apnea and bradycardia. Which drug will the nurse expect to administer? a. Albuterol (Proventil) b. Caffeine (Cafcit) c. Doxapram (Dopram) d. Methylphenidate (Ritalin)

ANS: B Caffeine is given to newborns that are experiencing apnea spells. The other drugs are not used for this purpose.

19. A patient has hypocalcemia caused by parathyroid hormone deficiency. Which medication will the nurse anticipate giving to this patient? a. Calcitonin b. Calcitriol c. Calcium d. Vitamin D

ANS: B Calcitriol is given for management of hypocalcemia caused by parathyroid hormone deficiency. Calcitonin is used to treat hyperparathyroidism. Calcium and vitamin D are not useful in parathyroid deficiency.

6. The nurse is preparing to care for a Native-American patient who has hypertension. The nurse understands that which antihypertensive medication would be most effective in this patient? a. Acebutolol (Sectral) b. Captopril (Capoten) c. Carteolol HCl (Cartrol) d. Metoprolol (Lopressor)

ANS: B Captopril is an angiotensin II inhibitor. Native-American patients do not respond well to beta blockers. Acebutolol, carteolol, and metoprolol are all beta blockers.

6. The nurse is teaching a patient who will take oral cyclophosphamide (Cytoxan). Which statement by the patient indicates understanding of the teaching? a. "I should follow a diet high in organ meats and beans while taking this drug." b. "I should brush my teeth and gums vigorously twice daily." c. "I should report any low-grade temperature elevation immediately." d. "I should take the drug at bedtime to minimize side effects."

ANS: C Even a low-grade temperature should be reported because it can indicate significant infection in immunocompromised patients. Patients should eat a low-purine diet while taking this medication. Patients should brush teeth and gums with a soft bristle toothbrush. Patients should take the medication early in the day to avoid accumulation in the bladder.

17. The nurse is helping to develop a plan of care for a patient who has advanced Alzheimer's disease. The patient will be taking a new medication. Which is a realistic goal for this patient? a. Demonstrate improved cognitive function. b. Exhibit improved ability to provide self-care. c. Receive appropriate assistance for care needs. d. Show improved memory for recent events.

ANS: C For the most part, drugs to treat AD do not result in improvement of symptoms but help slow the progress. The most realistic care plan for a patient with advanced AD is that they will receive appropriate and safe care.

4. An adult patient is brought to the emergency department for treatment of an asthma exacerbation. The patient uses inhaled albuterol as needed to control wheezing. The nurse notes expiratory wheezing, tremors, restlessness, and a heart rate of 120 beats per minute. The nurse suspects that the patient has a. over-used the albuterol. b. not been using albuterol. c. taken a beta-adrenergic blocker. d. taken a monoamine oxidase (MAO) inhibitor.

ANS: A High doses of albuterol may affect beta1 receptors, causing an increase in heart rate. Patients having an asthma exacerbation may over-use their albuterol inhalers when seeking relief. Patients may have wheezing and increased heart rate during an untreated asthma exacerbation, but they will not have tremors and restlessness.

6. The nurse is teaching a patient how to use phenylephrine HCl (Neo-Synephrine) nasal spray to treat congestion from a viral upper respiratory infection. What instruction will the nurse give the patient? a. Stop using the medication after 3 days. b. Spray the medication into the nose while lying supine. c. Use frequently since systemic side effects do not occur. d. Use the medication with any other over-the-counter medications.

ANS: A Nurses should explain to patients that continuous use of nasal sprays containing adrenergic agonists may result in rebound nasal congestion; these sprays should not be used more than 3 days. To avoid systemic absorption, spray should be administered while the patient is in an upright position. The medication may cause systemic side effects and should not be routinely used with other OTC cold medications.

9. The nurse prepares to change a patient's medication from an intravenous to an oral form and notes that the oral form is ordered in a higher dose. The nurse understands that this is due to differences in a. bioavailability. b. pinocytosis. c. protein binding. d. tachyphylaxis.

ANS: A Oral drugs may have less bioavailability because a lower percentage of the drug reaches the systemic circulation. Pinocytosis refers to the process by which cells carry a solute across a membrane. Protein binding can occur with both routes. Tachyphylaxis describes a rapid decrease in response to drugs that occurs when tolerance develops quickly.

9.The client is being treated with clobetasol propionate (Clobex). In scheduling the client's next appointment with the primary care provider, the nurse recognizes that the client's lesions need to be reassessed in _____ weeks. a. 2 b. 4 c. 6 d. 8

ANS: A The lesions should be reassessed after 2 weeks of treatment with the medication.

2.The client has been placed on tetracycline (Sumycin) for acne control. In planning a dosage schedule, the nurse anticipates that the client will be taking: a.low doses over a period of months. b.low doses for 3 to 4 weeks. c.high doses for 10 days to 2 weeks. d.high doses for at least 1 year.

ANS: A The most effective course of tetracycline dosage is low doses of the drug over a period of months.

3.Photosensitivity is a major complication of tetracycline (Sumycin). In providing client teaching, the nurse should suggest that the client ________ while on the medication. a.avoid direct sunlight b.wear a hat and long sleeves in the sun c.use a tanning bed only with supervision d.use a suntan lotion with a high SPF

ANS: A To avoid photosensitivity, the client should avoid direct sunlight while she is taking the medication.

17. The nurse is preparing to administer tolterodine tartrate (Detrol LA) to a patient who has incontinence. Which symptom would be a contraindication for this drug? a. Decreased bowel sounds b. Drooling c. Gastric upset d. Pain

ANS: A A decrease in bowel sounds could signal the beginning of paralytic ileus. Detrol is contraindicated in patients with paralytic ileus.

11. The nurse is preparing to give trimethoprim-sulfamethoxazole (TMP-SMX) to a patient and notes a petechial rash on the patient's extremities. The nurse will perform which action? a. Hold the dose and notify the provider. b. Request an order for a blood glucose level. c. Request an order for a BUN and creatinine level. d. Request an order for diphenhydramine (Benadryl).

ANS: A A petechial rash can indicate a severe adverse reaction and should be reported.

10. The nurse is preparing to administer the first dose of intravenous ceftriaxone (Rocephin) to a patient. When reviewing the patient's chart, the nurse notes that the patient previously experienced a rash when taking amoxicillin. What is the nurse's next action? a. Administer the drug and observe closely for hypersensitivity reactions. b. Ask the provider whether a cephalosporin from a different generation may be used. c. Contact the provider to report drug hypersensitivity. d. Notify the provider and suggest an oral cephalosporin.

ANS: A A small percentage of patients who are allergic to penicillin could also be allergic to a cephalosporin product. Patients should be monitored closely after receiving a cephalosporin if they are allergic to penicillin. There is no difference in hypersensitivity potential between different generations or method of delivery of cephalosporins.

9. A patient taking trimethoprim-sulfamethoxazole (TMP-SMX) to treat a urinary tract infection complains of a sore throat. The nurse will contact the provider to request an order for which laboratory test(s)? a. Complete blood count with differential b. Throat culture c. Urinalysis d. Coagulation studies

ANS: A A sore throat can indicate a life-threatening anemia, so a complete blood count with differential should be ordered.

2. A patient has a serum cholesterol level of 270 mg/dL. The patient asks the nurse what this level means. Which response by the nurse is correct? a. "You have a high risk for coronary artery disease." b. "You have a moderate risk for coronary artery disease." c. "You have a low risk for coronary artery disease." d. "You have no risk for coronary artery disease."

ANS: A A value of 270 mg/dL for serum cholesterol puts the patient at high risk.

15. The nurse is caring for a patient who has multiple sclerosis. The patient is experiencing an acute attack. Which drug does the nurse anticipate the provider will order? a. Adrenocorticotropic hormone (ACTH) b. Cyclophosphamide (Cytoxan) c. Glatiramer acetate (Copaxone) d. Interferon-B (IFN-B)

ANS: A ACTH is given to treat an acute attack of MS. Glatiramer acetate and interferon are used for remission-exacerbation states. Cyclophosphamide is given for chronic, progressive symptoms.

15. The nurse is providing teaching for the family of a patient who has been newly diagnosed with Alzheimer's disease (AD). Which statement by the family member indicates understanding of the teaching? a. "Alzheimer's disease is a chronic, progressive condition." b. "Alzheimer's disease affects memory but not personality." c. "The onset of Alzheimer's disease is usually between 65 and 75 years." d. "With proper treatment, symptoms of this disease can be arrested."

ANS: A AD is chronic and progressive, and there is no cure. It affects memory and personality. The onset is usually between 45 and 65 years. Symptoms cannot be arrested but may be slowed with treatment.

3. The parent of a 5-year-old child asks the nurse to recommend an over-the-counter pain medication for the child. Which analgesic will the nurse recommend? a. Acetaminophen (Tylenol) b. Aspirin (Ecotrin) c. Diflunisal (Dolobid) d. Ibuprofen (Motrin)

ANS: A Acetaminophen is safe to give children and does not cause gastrointestinal upset or interfere with platelet aggregation. Aspirin carries an increased risk of Reye's syndrome in children. Diflunisal (Dolobid) is not available over the counter.

14. A patient is brought to the emergency department and reports having taken "a lot" of acetaminophen extra-strength tablets 16 hours prior. The nurse will expect the provider to order a. acetylcysteine (Mucomyst). b. dornase alfa (Pulmozyme). c. gastric lavage. d. renal enzyme tests.

ANS: A Acetylcysteine is used as an antidote for acetaminophen overdose if given within 12 to 24 hours of ingestion. Dornase alfa is used to treat cystic fibrosis. Gastric lavage is no longer used as treatment. Liver enzyme tests are indicated since acetaminophen is hepatotoxic.

4. The nurse is caring for a patient who has recurrent urinary tract infections. The patient's current infection is not responding to an antibiotic that has been used successfully several times in the past. The nurse understands that this is most likely due to a. acquired bacterial resistance. b. cross-resistance. c. inherent bacterial resistance. d. transferred resistance.

ANS: A Acquired resistance occurs when an organism has been exposed to the antibacterial drug. Cross-resistance occurs when an organism that is resistant to one drug is also resistant to another. Inherent resistance occurs without previous exposure to the drug. Transferred resistance occurs when the resistant genes of one organism are passed to another organism.

13. A child is brought to the emergency department after ingestion of a toxic substance. The child is alert and conscious and is reported to have ingested kerosene 20 minutes prior. The nurse will anticipate administering a. activated charcoal. b. an anticholinergic antiemetic. c. gastric lavage. d. syrup of ipecac.

ANS: A Activated charcoal is used when patients have ingested a caustic substance or a petroleum distillate in a patient who is alert and awake. Gastric lavage is no longer used as therapy. Syrup of ipecac is not recommended.

6. The nurse receives the following order for a patient who is diagnosed with herpes zoster virus: PO acyclovir (Zovirax) 400 mg TID for 7 to 10 days. The nurse will contact the provider to clarify which part of the order? a. Dose and frequency b. Frequency and duration c. Drug and dose d. Drug and duration

ANS: A Acyclovir is used for herpes zoster, but the dose should be 800 mg 5 times daily for 7 to 10 days. The nurse should clarify the dose and frequency. For herpes simplex, 400 mg 3 times daily is correct.

7. The parent of a child who is taking amphetamine (Adderall) to treat attention deficit/hyperactivity disorder (ADHD) asks the provider to recommend an over-the-counter medication to treat a cold. What will the nurse tell the parent? a. "Avoid any products containing pseudoephedrine or caffeine." b. "Never give over-the-counter medications with Adderall." c. "Sudafed is a safe and effective decongestant." d. "Use any over-the-counter medication from the local pharmacy."

ANS: A Adderall is a stimulant, so other stimulants, such as caffeine and pseudoephedrine, should be avoided because a high plasma caffeine level can be fatal.

4. The nurse is caring for an African-American patient who has been taking a beta blocker to treat hypertension for several weeks with only slight improvement in blood pressure. The nurse will contact the provider to discuss a. adding a diuretic medication. b. changing to an ACE inhibitor. c. decreasing the beta blocker dose. d. doubling the beta blocker dose.

ANS: A African Americans do not respond well to beta blockers and ACE inhibitors, but do tend to respond to diuretics and calcium channel blockers. Changing to an ACE inhibitor or altering the beta blocker dose are not indicated. Hypertension in African-American patients can be controlled by combining beta blockers with diuretics.

1. The nurse is caring for a patient recently diagnosed with mild emphysema and provides teaching about the disease and medications for treatment. Which statement by the patient indicates understanding of the medication regimen? a. "I should use albuterol when my symptoms worsen." b. "I will need to take oral prednisone on a daily basis." c. "My provider will prescribe prophylactic antibiotics." d. "My symptoms are reversible with proper medications."

ANS: A Albuterol is used to treat bronchospasm during symptom flares. Oral prednisone is given for acute flares but not generally on a daily basis until symptoms are chronic and severe because of the risk of adrenal suppression. Prophylactic antibiotics are not given regularly because of the risk of antibiotic resistance. Symptoms of emphysema are not reversible.

17. Which antigout medication is used to treat chronic tophaceous gout? a. Allopurinol (Zyloprim) b. Colchicine c. Probenecid (Benemid) d. Sulfinpyrazone (Anturane)

ANS: A Allopurinol inhibits the biosynthesis of uric acid and is used long-term to manage chronic gout. Colchicine does not inhibit uric acid synthesis or promote uric acid secretion and is not used for chronic gout. Probenecid can be used for chronic gout but is not the first choice. Sulfinpyrazone has many serious side effects.

2. The nurse is performing a medication history on a patient who has glaucoma. The patient cannot remember the name of the drug prescribed but tells the nurse that the drug causes light sensitivity. The nurse knows that the drug is among which class of medications? a. Alpha-adrenergic agonists b. Beta-adrenergic blockers c. Cholinergic agonists d. Cholinesterase inhibitors

ANS: A Alpha-adrenergic agonists cause mydriasis, which increases sensitivity to light. Beta-adrenergic blockers cause miosis, which impairs vision in the dark. Cholinergic agonists and cholinesterase inhibitors may cause myopia and will impair vision in the dark.

10. A patient is taking doxazosin mesylate (Cardura) 1 mg per day to treat hypertension. The nurse notes a blood pressure of 110/72 mm Hg and a heart rate of 92 beats per minute. The nurse will contact the provider to discuss which change to the drug regimen? a. Changing to a beta-adrenergic blocker b. Decreasing the drug dose c. Increasing the drug dose d. Adding a diuretic

ANS: A Alpha-adrenergic blockers can cause orthostatic hypotension and reflex tachycardia. Beta blockers do not cause reflex tachycardia. Decreasing or increasing the drug dose is not recommended. Diuretics are added if blood pressure is not well-controlled.

2. A nursing student asks why a direct-acting cholinergic agonist drug that is selective to muscarinic receptors is described as being non-specific. The nurse will explain that this is because a. muscarinic receptors are present in many different tissues. b. the action of cholinesterase alters the bioavailability at different sites. c. these drugs can also affect nicotinic receptors. d. they vary in their reversible and irreversible effects.

ANS: A Although drugs classified as direct-acting cholinergic agonists are primarily selective for muscarinic receptors, they are non-specific because muscarinic receptors are located in different sites, causing actions in various organs. They are not affected differently by cholinesterase activity and have negligible actions on nicotinic receptors.

5. A patient has been using an amphetamine drug as an anorexiant for several weeks and asks the nurse about long-term adverse effects of this type of medication. The nurse will explain to the patient that these drugs a. can cause cardiac dysrhythmias. b. contribute to the development of narcolepsy. c. do not have severe effects when used properly. d. will cause orthostatic hypotension.

ANS: A Amphetamines can cause adverse effects in the central nervous, endocrine, gastrointestinal, and cardiovascular systems even when used as directed. Cardiac dysrhythmias can occur with continued use. Amphetamines do not cause narcolepsy or hypotension.

10. The nurse is teaching a nursing student about the antifungal drug amphotericin B. Which statement by the student indicates a need for further teaching? a. "Amphotericin B may be given intravenously or by mouth." b. "Patients who take this drug should have potassium and magnesium levels assessed." c. "Patients with renal disease should not take amphotericin B." d. "This drug is used for severe systemic infections."

ANS: A Amphotericin B is not absorbed from the gastrointestinal tract, so is not given by mouth. It can cause nephrotoxicity and electrolyte imbalance. It is highly toxic and is reserved for severe, systemic infections.

17. An appropriate goal when teaching a patient who has diarrhea is that the patient a. will have less frequent, more formed stools. b. will not have a stool for 1 to 2 days. c. will receive adequate intravenous fluids. d. will receive appropriate antibiotic therapy.

ANS: A An appropriate goal is that patients will have formed, less frequent stools not an absence of stools. Receiving adequate intravenous fluids or antibiotic therapy are interventions not goals.

8. The nurse is assessing a patient who takes warfarin (Coumadin). The nurse notes a heart rate of 92 beats per minute and a blood pressure of 88/78 mm Hg. To evaluate the reason for these vital signs, the nurse will assess the patient's a. gums, nose, and skin. b. lung sounds and respiratory effort. c. skin turgor and oral mucous membranes. d. urine output and level of consciousness.

ANS: A An increased heart rate followed by a decreased systolic pressure can indicate a fluid volume deficit caused by internal or external bleeding. The nurse should examine the patient's mouth, nose, and skin for bleeding. These vital signs do not indicate a pulmonary problem. Skin turgor and mucous membranes as well as urine output and level of consciousness may be assessed to determine the level of fluid deficit, but finding the source of blood loss is more important. Signs of gastrointestinal bleeding should also be assessed.

5. A patient who takes propantheline bromine (Pro-Banthine) and omeprazole (Prilosec) for an ulcer will begin taking an antacid. The nurse will give which instruction to the patient regarding how to take the antacid? a. Take the antacid 2 hours after taking the propantheline. b. Take the antacid along with a meal. c. Take the antacid with milk. d. Take the antacid with the propantheline bromine.

ANS: A Antacids can slow the absorption of anticholinergics and should be taken 2 hours after anticholinergic administration. Antacids should be given 1 to 3 hours after a meal and should not be given with dairy products.

9. A patient who has irritable bowel syndrome would most likely receive which type of drug to treat this condition? a. An anticholinergic b. A cholinergic esterase inhibitor c. A muscarinic agent d. A nicotinic agent

ANS: A Anticholinergic drugs are used to treat peptic ulcers and intestinal spasticity because of their actions to decrease gastric secretions and gastrointestinal spasms.

2. A patient who has epilepsy will begin an anticonvulsant medication. The patient asks the nurse how long the medication will be necessary. How will the nurse respond? a. "The medication is usually taken for a lifetime." b. "The medication will be given until you are seizure-free." c. "You will need to take the medication for 3 to 5 years." d. "You will take the medication as needed for seizure activity."

ANS: A Anticonvulsants are given to prevent seizures and are usually taken throughout the patient's lifetime. Stopping the medication will lead to recurrence of seizures in most patients. Some patients may attempt to stop taking the medications after 3 to 5 years of no seizure activity. Anticonvulsants are not given as needed.

18. The nurse is assessing a patient who has gout who will begin taking allopurinol (Zyloprim). The nurse reviews the patient's medical record and will be concerned about which laboratory result? a. Elevated BUN and creatinine b. Increased serum uric acid c. Slight increase in the white blood count d. Increased serum glucose

ANS: A Antigout drugs are excreted via the kidneys, so patients should have adequate renal function.

1. A patient who has narrow-angle glaucoma asks the nurse to recommend a medication to alleviate cold symptoms such as nasal congestion and runny nose. The nurse will suggest the patient talk to the provider about which medication? a. Azelastine (Astelin) b. Cetirizine (Zyrtec) c. Chlorpheniramine maleate (Chlor-Trimeton) d. Diphenhydramine (Benadryl)

ANS: A Antihistamines have anticholinergic effects, which are contraindicated in patients with narrow-angle glaucoma. Cetirizine and azelastin are second-generation antihistamines, with fewer anticholinergic side effects, but azelastine is a nasal spray and is less likely to have systemic side effects.

6. The nurse is teaching a group of nursing students about the use of antipsychotic drugs for antiemetic purposes. The nurse will explain that, when given as antiemetics, these drugs are given a. in smaller doses. b. less frequently. c. with anticholinergics. d. with antihistamines.

ANS: A Antipsychotic medications have antiemetic properties in smaller doses.

7. The nurse is performing a health history on a patient who has arthritis. The patient reports tinnitus. Suspecting a drug adverse effect, the nurse will ask the patient about which medication? a. Aspirin (Bayer) b. Acetaminophen (Tylenol) c. Anakinra (Kineret) d. Prednisone (Deltasone)

ANS: A Aspirin causes tinnitus at low toxicity levels. The nurse should question the patient about this medication. The other medications do not have this side effect.

6. The nurse is providing teaching to a patient who will begin taking aspirin to treat arthritis pain. Which statement by the patient indicates a need for further teaching? a. "I should increase fiber and fluids while taking aspirin." b. "I will call my provider if I have abdominal pain." c. "I will drink a full glass of water with each dose." d. "I will notify my provider of ringing in my ears."

ANS: A Aspirin is not constipating, so patients do not need to be counseled to consume extra fluids and fiber. Abdominal pain can occur with gastrointestinal bleeding, and tinnitus (ringing in the ears) can be an early sign of toxicity, so patients should be taught to contact their provider if these occur. Taking a full glass of water with each dose helps minimize gastrointestinal side effects.

9. A patient who has recently had a myocardial infarction (MI) will begin taking clopidogrel (Plavix) to prevent a second MI. Which medication will the nurse expect the provider to order as adjunctive therapy for this patient? a. Aspirin b. Enoxaparin sodium (Lovenox) c. Ticagrelor (Brilinta) d. Warfarin (Coumadin)

ANS: A Aspirin is often used with clopidogrel to inhibit platelet aggregation to increase the effectiveness of this drug. Enoxaparin is used to prevent venous thrombosis. Ticagrelor is similar to clopidogrel and is not used along with clopidogrel. Warfarin is used to prevent thrombosis.

14. A patient who is taking diphenoxylate with atropine (Lomotil) to treat diarrhea asks the nurse why it contains atropine. The nurse will explain that atropine is added to a. decrease abdominal cramping. b. increase intestinal motility. c. minimize nausea and vomiting. d. provide analgesia.

ANS: A Atropine is added to decrease abdominal cramping and intestinal motility. It does not affect nausea and vomiting or pain.

16. A patient who is intubated develops bradycardia because of vagal stimulation. Which medication will the nurse anticipate administering to treat this symptom? a. Atropine sulfate (Atropine) b. Benztropine (Cogentin) c. Bethanechol chloride (Urecholine) d. Metoclopramide (Reglan)

ANS: A Atropine is used to treat bradycardia caused by vagal stimulation.

10. A patient who has stable angina pectoris is given nitroglycerin to use as needed. In addition to pharmacotherapy, the nurse will give the patient which instruction? a. Avoid extremes in weather. b. Begin a rigorous exercise program. c. Drink glass of red wine daily. d. Seek medical care at first sign of pain.

ANS: A Avoiding extreme weather conditions is important to help prevent anginal attacks. Patients should be instructed to avoid strenuous exercise; avoid alcohol, which can enhance hypotensive effects of nitrates; and use nitroglycerin at the first sign of pain.

5. The nurse provides home care instructions for a patient who will take a high dose of azithromycin after discharge from the hospital. Which statement by the patient indicates understanding of the teaching? a. "I may take antacids 2 hours before taking this drug." b. "I should take acetaminophen for fever or mild pain." c. "I should expect diarrhea to be a common, mild side effect." d. "I should avoid dairy products while taking this drug."

ANS: A Azithromycin peak levels may be reduced by antacids when taken at the same time so patients should be cautioned to take antacids 2 hours before or 2 hours after taking the drug. High-dose azithromycin carries a risk for hepatotoxicity when taken with other potentially hepatotoxic drugs such as acetaminophen. Diarrhea may indicate pseudomembranous colitis and should be reported. There is no restriction for dairy products when taking azithromycin.

13. The nurse is preparing a patient for surgery. The patient received a hypnotic medication the night prior and the nurse is administering midazolam (Versed) and atropine. The patient asks why all of these medications are necessary. The nurse will tell the patient that they are given for which reason? a. To decrease the amount of general anesthesia needed b. To minimize post-operative drowsiness c. To prolong the anesthetized state d. To speed up anesthesia induction

ANS: A Balanced anesthesia includes giving a hypnotic the night prior to surgery, premedication with an opioid analgesic or benzodiazepine plus an anticholinergic, and then a short-acting barbiturate, an inhaled gas, and a muscle relaxant. One effect of this is to decrease the amount of general anesthetic needed. It may reduce post-operative nausea and vomiting, but does not decrease drowsiness. It does not affect the duration of anesthesia, which is dependent on the length of time the inhaled gas is given, or the rate of induction.

11. A patient with chronic obstructive pulmonary disease (COPD) who has a persistent nonproductive cough asks about a medication that will not cause sedation. The nurse will encourage the patient to discuss which medication with the provider? a. Benzonatate HCl (Tessalon Perles) b. Dextromethorphan hydrobromide (Benylin DM) c. Guaifenesin and codeine d. Promethazine with dextromethorphan

ANS: A Benzonatate will not cause sedation and is safe for patients with COPD. Dextromethorphan is contraindicated in patients with COPD. Codeine and promethazine cause sedation.

4. The nurse is administering timolol (Timoptic) eye drops to a patient who has glaucoma. To prevent bradycardia, the nurse will perform which action? a. Apply pressure to the lacrimal ducts. b. Have the patient sit up after instilling the drops. c. Prepare to administer an alpha-adrenergic agonist. d. Wait 5 minutes between drops.

ANS: A Bradycardia is a systemic side effect of timolol. Applying pressure to the lacrimal ducts prevents the medication from being systemically absorbed and causing systemic side effects such as bradycardia.

17. A parent expresses concern that a 5-year-old child may develop epilepsy because the child experienced a febrile seizure at age 18 months. What will the nurse tell this parent? a. "A child who has had a febrile seizure is considered to have epilepsy." b. "A small percentage of children who have febrile seizures develop epilepsy." c. "I recommend discussing prophylactic anticonvulsant drugs with the provider." d. "Treat fevers aggressively with aspirin and NSAIDs to prevent seizures."

ANS: B Epilepsy develops in 2.5% of children who have one or more febrile seizures. One febrile seizure does not cause a diagnosis of epilepsy. Prophylactic anticonvulsants are given to high-risk patients. Children should not receive aspirin for fever because of the risk of Reye's syndrome.

3. The nurse administers subcutaneous epinephrine to a patient who is experiencing an anaphylactic reaction. The nurse should expect to monitor the patient for which symptom? a. Bradycardia b. Decreased urine output c. Hypotension d. Nausea and vomiting

ANS: B Epinephrine can cause renal vasoconstriction and thereby reduce renal perfusion and decrease urinary output. Epinephrine causes tachycardia and elevates blood pressure. Nausea and vomiting are not expected to occur.

1. The nurse caring for a patient who will receive penicillin to treat an infection asks the patient about previous drug reactions. The patient reports having had a rash when taking amoxicillin (Amoxil). The nurse will contact the provider to a. discuss giving a smaller dose of penicillin. b. discuss using erythromycin (E-mycin) instead of penicillin. c. request an order for diphenhydramine (Benadryl). d. suggest that the patient receive cefuroxime (Ceftin).

ANS: B Erythromycin is the drug of choice when penicillin is not an option. Giving smaller doses of penicillin does not prevent hypersensitivity reactions. Benadryl is useful when a hypersensitivity reaction has occurred. A small percentage of patients allergic to penicillins may be hypersensitive to cephalosporins.

11. The nurse is caring for a patient who has been diagnosed with petit mal seizures. The nurse will anticipate teaching this patient about which antiepileptic medication? a. Carbamazepine (Tegretol) b. Ethosuximide (Zarontin) c. Phenobarbital (Luminal) d. Phenytoin (Dilantin)

ANS: B Ethosuximide is used to treat petit mal seizures. The other drugs are not used to treat petit mal seizures.

18. A 25 year-old female patient will begin taking phenytoin for epilepsy. The patient tells the nurse she is taking oral contraceptives (OCPs). Which response will the nurse give? a. "Continue taking OCPs because phenytoin is not safe during pregnancy." b. "You should use a backup method of contraception along with OCPs." c. "You should stop taking OCPs because of drug-drug interactions with phenytoin." d. "You should take low-dose aspirin while taking these medications to reduce your risk of stroke."

ANS: B Female patients who take oral contraceptives and anticonvulsants should be advised to use a backup method of contraception because of reduced effectiveness of OCPs. Patients should be cautioned to consult with a provider if considering pregnancy because of the teratogenic effects of anticonvulsants. Patients should not stop taking OCPs and do not need to take precautions against stroke.

18. The nurse is caring for a patient who is receiving clopidogrel (Plavix). The patient calls the nurse to report flulike symptoms. The nurse notes a heart rate of 76 beats per minute, a blood pressure of 110/76 mm Hg, and a respiratory rate of 20 breaths per minute. The nurse suspects that the patient is experiencing which condition? a. Anaphylaxis b. An expected drug side effect c. Hemorrhage d. Possible myocardial infarction

ANS: B Flulike symptoms are a side effect of clopidogrel. The patient has normal vital signs. Anaphylaxis is characterized by respiratory distress and hypotension. Hemorrhage is characterized by tachycardia and hypotension.

7. A patient who is receiving cancer chemotherapy has been ordered to receive epoetin alfa (Procrit) 150 units/kg 3 times weekly. The nurse reviews the patient's chart and notes a hemoglobin level of 10.1 g/dL. The nurse will perform which action? a. Administer the medication as ordered. b. Hold the dose and notify the provider. c. Reduce the dose by 25%. d. Request an order for an increased dose.

ANS: B For patients receiving cancer chemotherapy, erythropoietin-stimulating agents should not be initiated at a hemoglobin level greater than or equal to 10 g/dL.

5. The nurse is caring for a patient who develops marked edema and a low urine output as a result of heart failure. Which medication will the nurse expect the provider to order for this patient? a. Digoxin (Lanoxin) b. Furosemide (Lasix) c. Hydrochlorothiazide (HydroDIURIL) d. Spironolactone (Aldactone)

ANS: B Furosemide is a loop diuretic and is given when the patient's condition warrants immediate removal of body fluid, as in heart failure. Digoxin improves cardiac function but does not remove fluid quickly. The other diuretics may be used when immediate fluid removal is not necessary.

13. A patient who is taking acyclovir (Zovirax) to treat an oral HSV-1 infection asks the nurse why oral care is so important. The nurse will tell the patient that meticulous oral care helps to a. minimize transmission of disease. b. prevent gingival hyperplasia. c. reduce viral resistance to the drug. d. shorten the duration of drug therapy.

ANS: B Good oral care can prevent gingival hyperplasia in patients with HSV-1.

12. A patient who has a nonproductive cough will begin taking guaifenesin to help with secretions. When teaching this patient about the medication, the nurse will provide which instruction? a. "Avoid driving or using heavy machinery." b. "Drink extra water while taking the medication." c. "Monitor urine output closely." d. "Take with an oral antihistamine for better effects."

ANS: B Guaifenesin is an expectorant, and patients taking this medication should be advised to increase fluid intake to at least 8 glasses of water per day. (Remember to assess for contraindications to increasing fluid intake [e.g., heart failure, kidney failure with dialysis, etc.].) Guaifenesin does not cause drowsiness or urinary retention. Antihistamines will dry secretions, making them harder to expectorate.

4. A patient is being treated with isoniazid (INH), rifampin, and pyrazinamide in phase I of treatment for tuberculosis. The organism develops resistance to isoniazid. Which drug will the nurse anticipate the provider will order to replace the isoniazid? a. Ciprofloxacin (Cipro) b. Ethambutol (Myambutol) c. Kanamycin d. Streptomycin sulfate

ANS: B If there is bacterial resistance to isoniazid, the first phase may be changed to ethambutol, rifampin, and pyrazinamide. Ciprofloxacin, kanamycin, and streptomycin are not generally first-line antitubercular drugs.

10. The nurse provides teaching for a patient who will begin taking indomethacin (Inderal) to treat rheumatoid arthritis. Which statement by the patient indicates a need for further teaching? a. "I should limit sodium intake while taking this drug." b. "I should take indomethacin on an empty stomach." c. "I will need to check my blood pressure frequently." d. "I will take the medication twice daily."

ANS: B Indomethacin is very irritating to the stomach and should be taken with food. It can cause sodium retention and elevated blood pressure, so patients should limit sodium intake. The medication is taken twice daily.

15. The nurse is caring for a patient who has rheumatoid arthritis and who is receiving infliximab (Remicade) IV every 8 weeks. Which laboratory test will the nurse anticipate that this patient will need? a. Calcium level b. Complete blood count c. Electrolytes d. Potassium

ANS: B Infliximab is an immunomodulator and can cause agranulocytosis, so patients should have regular CBC evaluation.

10. A patient who has been taking butabarbital (Butisol) for several weeks reports being drowsy and having difficulty performing tasks at work most mornings. The nurse suspects that which drug effects have occurred? a. Dependence b. Hangover c. Tolerance d. Withdrawal

ANS: B Intermediate-acting hypnotics, such as butabarbital, are useful for sustaining sleep, but patients often experience residual drowsiness in the morning, or hangover. Drug dependence occurs when patients develop a need for the drug. Tolerance refers to a reduced drug effect requiring larger amounts of drug to get the desired effect. Withdrawal occurs when stopping the drug causes symptoms that can only be alleviated by taking the drug.

16. A patient is diagnosed with histoplasmosis and will begin taking ketoconazole. What information will the nurse include when teaching this patient about this medication? a. "Take the medicine twice daily." b. "Take the medication with food." c. "You may consume small amounts of alcohol." d. "You will not need lab tests while taking this drug."

ANS: B Ketoconazole should be taken with food. It is administered once daily. Patients taking antifungals should not consume alcohol. Antifungals can cause liver and renal toxicity, so patients will need lab monitoring.

13. A patient is admitted to the hospital to treat hypothyroidism. For rapid improvement in symptoms, the nurse will expect to administer which medication? a. Levothyroxine sodium (Synthroid) b. Liothyronine (Cytomel) c. Liotrix (Thyrolar) d. Thyroid desiccated (Armour Thyroid)

ANS: B Liothyronine has a short half-life and rapid onset of action and is not recommended for maintenance therapy but is used as initial therapy for severe myxedema. Levothyroxine is the drug of choice for replacement therapy. Liotrix is a second-line drug. Thyroid desiccated is used for hypothyroidism to reduce goiter size.

14. The nurse is teaching a group of nursing students about multiple sclerosis (MS). Which statement by the nurse is correct? a. "MS is characterized by degeneration of neurons and nerves in the brain and spinal cord." b. "MS is characterized by lesions or plaques on myelin sheaths of nerves." c. "MS is characterized by neuritic plaques and neurofibrillary tangles in the CNS." d. "MS is characterized by weak muscles and decreased nerve impulses caused by decreased ACh."

ANS: B MS is characterized by lesions on myelin sheaths of nerves.

6. The nurse is teaching a child and a parent about taking methylphenidate (Ritalin) to treat attention deficit/hyperactivity disorder (ADHD). Which statement by the parent indicates understanding of the teaching? a. "I should give this drug to my child at bedtime." b. "My child should avoid products containing caffeine." c. "The drug should be stopped immediately if my child develops aggression." d. "We should monitor my child's weight since weight gain is common."

ANS: B Methylphenidate is a stimulant, so other stimulants such as caffeine should be avoided because a high plasma caffeine level can be fatal. The medication should be taken in the morning. Patients should be taught not to stop the drug abruptly to avoid withdrawal symptoms. Weight loss is common.

8. The nurse is checking an 8-year-old child who has attention deficit/hyperactivity disorder (ADHD) into a clinic for an annual well-child visit. The child takes methylphenidate HCl (Ritalin). Which assessments are especially important for this child? a. Heart rate, respiratory rate, and oxygen saturation b. Height, weight, and blood pressure c. Measures of fine- and gross-motor development d. Nausea, vomiting, and gastrointestinal upset

ANS: B Methylphenidate may cause growth suppression, so the child's height and weight should be assessed. Methylphenidate may also increase blood pressure, so the nurse should pay careful attention to blood pressure.

9. The parent of a child who is receiving chemotherapy asks the nurse why metoclopramide (Reglan) is not being used to suppress vomiting. The nurse will explain that, in children, this drug is more likely to cause which effect? a. Excess sedation b. Extrapyramidal symptoms c. Paralytic ileus d. Vertigo

ANS: B Metoclopramide can cause extrapyramidal symptoms, and these effects are more likely in children. Children are not more prone to sedative effects, paralytic ileus, or vertigo while taking this drug.

2. A nursing student asks how nonsteroidal antiinflammatory drugs (NSAIDs) work to suppress inflammation and reduce pain. The nurse will explain that NSAIDs a. exert direct actions to cause relaxation of smooth muscle. b. inhibit cyclooxygenase that is necessary for prostaglandin synthesis. c. interfere with neuronal pathways associated with prostaglandin action. d. suppress prostaglandin activity by blocking tissue receptor sites.

ANS: B NSAIDs act by inhibiting COX-1 and COX-2 to help block prostaglandin synthesis. They do not have direct action on tissues, nor do they interfere with chemical receptor sites or neuronal pathways.

4. The nurse is preparing to administer digoxin to a patient who has heart failure. The patient reports nausea, vomiting, and a headache. The nurse notes a respiratory rate of 18 breaths per minute, a heart rate of 58 beats per minute, and a blood pressure of 120/78 mm Hg. What will the nurse do next? a. Administer the next dose as ordered since these are mild side effects. b. Hold the dose and notify the provider of possible digoxin toxicity. c. Reassure the patient that these are common, self-limiting side effects. d. Request an order for an antiemetic and an analgesic medication.

ANS: B Nausea, vomiting, and headache are common signs of digoxin toxicity as is a heart rate less than 60 beats per minute. The nurse should hold the dose and notify the provider.

2. Children who experience nightmares have these during which stage of sleep? a. Early morning sleep b. Nonrapid eye movement sleep c. Rapid eye movement sleep d. Sleep induction

ANS: B Nightmares that occur in children take place during NREM sleep.

15. A patient asks the nurse why nitroglycerin is given sublingually. The nurse will explain that nitroglycerin is administered by this route for which reason? a. To avoid hypotension b. To increase absorption c. To minimize gastrointestinal upset d. To prevent hepatotoxicity

ANS: B Nitroglycerin is given sublingually to avoid first-pass metabolism by the liver, which would occur if the drug is swallowed. It does not prevent hypotension. Gastrointestinal upset and hepatotoxicity usually do not occur.

1. A patient is diagnosed with epilepsy and asks the nurse what may have caused this condition. The nurse explains that epilepsy is most often a. caused by head trauma. b. idiopathic in origin. c. linked to a stroke. d. related to brain anoxia.

ANS: B Of all seizure cases, 75% are primary, or idiopathic, with no known cause. The remaining are secondary and may be related to head trauma, stroke, or anoxic events.

17. The nurse checks on a patient who has received sumatriptan (Imitrex) for treatment of a migraine headache. The patient reports moderate improvement in headache pain and reports feeling dizzy. The nurse notes a blood pressure of 160/85 mm Hg. Which action by the nurse is correct? a. Notify the provider of the dizziness. b. Notify the provider of the increased blood pressure. c. Plan to administer a second dose in 1 hour. d. Request an order for intranasal sumatriptan.

ANS: B Triptans can cause increased blood pressure, which is an adverse drug reaction and should be reported to the provider. Dizziness is a common side effect but not potentially life-threatening. The second dose should not be given if the patient is experiencing elevated blood pressure. Intranasal sumatriptan has the same adverse effects.

13. The nurse is caring for a patient who will begin taking captopril (Capoten) for hypertension. The nurse reviews the patient's laboratory test results and notes increased BUN and creatinine. Which action will the nurse take? a. Administer the captopril and monitor vital signs. b. Contact the provider to discuss changing to fosinopril (Monopril). c. Obtain an order for intravenous fluids to improve urine output. d. Request an order to add hydrochlorothiazide (HydroDIURIL).

ANS: B Patients who have renal insufficiency will not require a decrease in dose with fosinopril, as they would with other angiotensin-converting enzyme (ACE) inhibitors. If captopril is given, it should be given in a reduced dose. Increased IV fluids are not indicated.

5. The nurse is preparing to administer a first dose of benztropine (Cogentin) to a patient diagnosed with parkinsonism. The nurse would notify the patient's provider if the patient had a history of which condition? a. Asthma b. Glaucoma c. Hypertension d. Irritable bowel disease

ANS: B Patients with a history of glaucoma should not take anticholinergic medications. Anticholinergics are not contraindicated in patients who have asthma, hypertension, or irritable bowel disease.

8. The nurse is performing an assessment on a patient who will begin taking propranolol (Inderal) to treat hypertension. The nurse learns that the patient has a history of asthma and diabetes. The nurse will take which action? a. Administer the medication and monitor the patient's serum glucose. b. Contact the provider to discuss another type antihypertensive medication. c. Request an order for renal function tests prior to administering this drug. d. Teach the patient about the risks of combining herbal medications with this drug.

ANS: B Patients with chronic lung disease are at risk for bronchospasm with beta blockers, especially those like propranolol which are non-selective. Beta blockers, with the exception of carvedilol, also decrease the efficacy of many oral antidiabetic medications. The nurse should discuss a change in medications to one that does not carry this risk.

10. A female patient who takes phenytoin for epilepsy becomes pregnant. The nurse will notify the patient's provider and will anticipate that the provider will take which action? a. Add valproic acid (Depakote) for improved seizure control. b. Change the medication to phenobarbital (Luminal). c. Closely monitor this patient's serum phenytoin levels. d. Discontinue all anticonvulsant medications.

ANS: B Phenytoin has serious teratogenic effects, so women who are pregnant should not take it. Phenobarbital is typically used because possible teratogenic effects are less pronounced. Teratogenicity increases with multiple anticonvulsants.

13. The nurse is preparing to administer an intravenous polymyxin antibiotic. The patient reports dizziness along with numbness and tingling of the hands and feet. The nurse will perform which action? a. Administer the drug since these are harmless side effects. b. Hold the drug and notify the provider of these adverse reactions. c. Obtain an order for an oral form of this medication. d. Request an order for serum electrolytes.

ANS: B Polymyxins can cause nephrotoxicity and neurotoxicity. This patient has signs of neurotoxicity, so the nurse should notify the provider. These effects are generally reversible when the drug is discontinued. It is not correct to administer the drug when these symptoms are present. Polymyxins are not absorbed orally. Serum electrolytes are not indicated.

4. The nurse receives an order to administer a purine nucleoside antiviral medication. The nurse understands that this medication treats which type of virus? a. Hepatitis virus b. Herpes virus c. HIV d. Influenza virus

ANS: B Purine nucleosides, such as acyclovir, are used to treat herpes simplex viruses 1 and 2, herpes zoster virus, varicella-zoster virus, and cytomegalovirus.

15. A patient who is taking metronidazole (Flagyl) reports reddish-brown urine. Which action will the nurse take? a. Obtain an order for BUN and creatinine levels. b. Reassure the patient that this is a harmless effect. c. Request an order for a urinalysis. d. Test her urine for occult blood.

ANS: B Reddish-brown urine is a harmless side effect of metronidazole and is not cause for concern.

9. A patient has recently begun taking phenytoin (Dilantin) for a seizure disorder. The nurse notes a reddish-brown color to the patient's urine. Which action will the nurse take? a. Ask the provider to order a serum drug level. b. Reassure the patient that this is a harmless side effect. c. Report possible thrombocytopenia to the provider. d. Request an order for a urinalysis and creatinine clearance.

ANS: B Reddish-brown urine is a harmless side effect of phenytoin. The nurse should reassure the patient. It is not necessary to order a serum drug level or renal function studies. It is not a symptom of thrombocytopenia.

1. A patient who has been instructed to use a liquid antacid medication to treat gastrointestinal upset asks the nurse about how to take this medication. What information will the nurse include when teaching this patient? a. Take a laxative if constipation occurs. b. Take 60 minutes after meals and at bedtime. c. Take with at least 8 ounces of water to improve absorption. d. Take with milk to improve effectiveness.

ANS: B Since maximum acid secretion occurs after eating and at bedtime, antacids should be taken 1 to 3 hours after eating and at bedtime. Taking antacids before meals slows gastric emptying time and causes increased gastrointestinal (GI) secretions. Patients should not self-treat constipation or diarrhea. Patients should use 2 to 4 ounces of water when taking to ensure that the drug enters the stomach; more than that will increase GI secretions. Antacids should not be taken with milk or foods high in vitamin D.

16. The nurse is performing an admission assessment on a patient who has recently begun taking reserpine. The patient reports using St. John's wort. The nurse anticipates that the patient will have a. hypotension. b. hypertension. c. bradycardia. d. tachycardia.

ANS: B St. John's wort antagonizes the hypotensive effects of reserpine, causing hypertension.

1. The nurse is teaching a group of nursing students about diabetes. The nurse explains that which type of diabetes is the most common? a. Type 1 diabetes mellitus b. Type 2 diabetes mellitus c. Diabetes insipidus d. Secondary diabetes

ANS: B Type 2 diabetes mellitus is the most common type of diabetes.

1. A patient who has cancer is about to begin chemotherapy. The patient asks the nurse why two chemotherapeutic agents are being used instead of just one. Which response by the nurse is correct? a. "The drugs may be given in less toxic doses if two drugs are used." b. "Two agents used together can have synergistic effects." c. "Use of two drugs will increase tumorcidal activity in the G0 phase of the cell." d. "Using two agents will shorten the length of time chemotherapy is needed."

ANS: B Using two or more chemotherapeutic agents can have a synergistic effect. Combination therapy typically uses two drugs with different dose-limiting toxicities, but the use of more than one drug does not allow for using less toxic doses. Combination therapy allows cell kill in all phases of the cell cycle. Combination therapy does not shorten the length of time chemotherapy is needed.

7. An adolescent female has dysmenorrhea associated with heavy menstrual periods. The patient's provider has recommended ibuprofen (Motrin). When teaching this patient about this drug, the nurse will tell her that ibuprofen a. may decrease the effectiveness of oral contraceptive pills. b. may increase bleeding during her period. c. should be taken on an empty stomach to increase absorption. d. will decrease the duration of her periods.

ANS: B When nonsteroidal antiinflammatory drugs (NSAIDs) are used to treat dysmenorrhea, excess bleeding may occur during the first 2 days of a period. NSAIDs do not decrease the effect of OCPs. NSAIDs are irritating to the stomach, so patients should take with food or a full glass of water. NSAIDs will not decrease the duration of periods.

6. The nurse performs a medication history on a patient who will begin targeted therapy for cancer with gefitinib (Iressa). The nurse learns that the patient is taking carbamazepine, a histamine2 blocker, and warfarin. The nurse will anticipate that the provider will make which change to the medication regimen? a. decrease the gefitinib dose. b. decrease the warfarin dose. c. increase the histamine2 blocker dose. d. increase the carbamazepine dose.

ANS: B When patients taking warfarin take gefitinib, the effectiveness of the warfarin is greatly increased, and bleeding risks increase. Carbamazepine and histamine2 blockers decrease the effectiveness of gefitinib, so decreasing the gefitinib dose or decreasing the carbamazapine or histamine2 blocker is not recommend

9. The nurse assumes care for a patient who is currently receiving a dose of intravenous vancomycin (Vancocin) infusing at 20 mg/min. The nurse notes red blotches on the patient's face, neck, and chest and assesses a blood pressure of 80/55 mm Hg. Which action will the nurse take? a. Request an order for IV epinephrine to treat anaphylactic shock. b. Slow the infusion to 10 mg/min and observe the patient closely. c. Stop the infusion and obtain an order for a BUN and serum creatinine. d. Suspect Stevens-Johnson syndrome and notify the provider immediately.

ANS: B When vancomycin is infused too rapidly, "red man" syndrome may occur; the rate should be 10 mg/min to prevent this. This is a toxic reaction, not an allergic one, so epinephrine is not indicated. Stevens-Johnson syndrome is characterized by a rash and fever. Red man syndrome is not related to renal function.

6. Which antacid is likely to cause acid rebound? a. Aluminum hydroxide b. Calcium carbonate c. Magnesium hydroxide d. Magnesium trisilicate

ANS: B While calcium carbonate is most effective in neutralizing acid, a significant amount can be systemically absorbed and can cause acid rebound. The other antacids do not have significant systemic absorption.

11. The nurse is providing teaching for a patient who will begin taking zolpidem tartrate (Ambien) 10 mg at bedtime as a sleep aid. Which statement by the patient indicates understanding of the teaching? a. "I should take this medication with food to avoid stomach upset." b. "I will take this medication within 30 minutes of bedtime." c. "If this medication is not effective, I may increase the dose to 15 mg." d. "Tolerance and drug dependence do not occur with this medication."

ANS: B Zolpidem is a nonbenzodiazepine sleep aid. It should be taken 30 minutes before desired sleep. Food decreases the absorption, so it should be taken on an empty stomach. The maximum dose is 10 mg. Tolerance and dependence may occur.

1. Cholinergic drugs have specific effects on the body. What are the actions of cholinergic medications? (Select all that apply.) a. Dilate pupils b. Decrease heart rate c. Stimulate gastric muscle d. Dilate blood vessels e. Dilate bronchioles f. Increase salivation g. Constrict pupils

ANS: B, C, D, F, G Decreasing heart rate, stimulating gastric muscles, dilating blood vessels, increasing salivation, and constricting pupils are actions of the cholinergic drugs.

1. A patient has congestive heart failure and has been taking digoxin (Lanoxin) for 9 years. The patient is admitted with signs and symptoms of digoxin toxicity. Which signs and symptoms are associated with digoxin toxicity? (Select all that apply.) a. Dysuria b. Vomiting c. Tachycardia d. Yellow haloes in the visual field e. Diarrhea f. Insomnia

ANS: B, D, E Vomiting, yellow haloes in the visual field, and diarrhea are classic signs of digoxin toxicity. Bradycardia, not tachycardia, will likely be noted.

12. A patient will begin taking streptomycin as part of the medication regimen to treat tuberculosis. Before administering this medication, the nurse will review which laboratory values in the patient's medical record? a. Complete blood count (CBC) with differential white cell count b. Blood urea nitrogen (BUN) and creatinine c. Potassium and magnesium levels d. Serum fasting glucose

ANS: B Streptomycin can cause significant renal toxicity.

1. Which is a characteristic that distinguishes sulfonamides from other drugs used to treat bacterial infection? a. Sulfonamides are bactericidal. b. Sulfonamides are derived from biologic substances. c. Sulfonamides have antifungal and antiviral properties. d. Sulfonamides increase bacterial synthesis of folic acid.

ANS: B Sulfonamides are bacteriostatic, not bactericidal. They are not derived from biologic substances. They are not antifungals or antivirals. They act by decreasing bacterial synthesis of folic acid.

8. A patient is admitted to the hospital for treatment of pneumonia after complaining of high fever and shortness of breath. The patient was not able to produce sputum for a culture. The nurse will expect the patient's provider to order a. a broad-spectrum antibiotic. b. a narrow-spectrum antibiotic. c. multiple antibiotics. d. the pneumococcal vaccine.

ANS: A Broad-spectrum antibiotics are frequently used to treat infections when the offending organism has not been identified by culture and sensitivity (C&S). Narrow-spectrum antibiotics are usually effective against one type of organism and are used when the C&S indicates sensitivity to that antibiotic. The use of multiple antibiotics, unless indicated by C&S, can increase resistance. The pneumococcal vaccine is used to prevent, not treat, an infection.

5. A child exhibits acromegaly caused by a tumor that cannot be destroyed with radiation. Which medication will most likely be used to treat this child? a. Bromocriptine mesylate (Parlodel) b. Octreotide acetate (Sandostatin) c. Somatrem (Protropin) d. Somatropin (Genotropin)

ANS: A Bromocriptine is a prolactin-release inhibitor and is used to inhibit release of growth hormone from the pituitary gland if the tumor cannot be destroyed by radiation. Octreotide may be used as well, but it is expensive and is typically used as adjunct therapy to radiation. Somatrem and somatropin are used to treat growth hormone deficiency and would make acromegaly worse.

16. A postoperative patient has a history of opioid abuse. Which analgesic medication will the nurse expect the provider to order for this patient? a. Buprenorphine (Buprenex) b. Butorphanol tartrate (Stadol) c. Naloxone (Narcan) d. Pentazocine (Talwin)

ANS: A Buprenorphine is an opioid agonist-antagonist analgesic and was developed to help decrease opioid abuse. Butophanol and pentazocine are also in this class, but reports say that they cause dependence. Naloxone is an opioid antagonist and is given to reverse the effects of opioids if toxicity occurs.

7. The nurse is preparing to administer olopatadine (Patanol) eyedrops to a patient who has allergic conjunctivitis. The patient tells the nurse that the drops have caused burning and stinging. What action will the nurse take? a. Administer the drops and reassure the patient that this is a normal side effect. b. Offer an over-the-counter eye lubricant to minimize this adverse effect. c. Request an order for antibiotic eyedrops. d. Withhold the medication and notify the provider.

ANS: A Burning and stinging are the most common side effects of this class of drugs but do not warrant withholding the medication. An over-the-counter lubricant is not indicated. These symptoms are not an indication of infection.

4. The nurse is caring for a patient who reports being able to fall asleep but has difficulty staying asleep. The nurse will contact the provider to obtain an order for which medication? a. Butabarbital (Butisol) b. Flurazepam (Dalmane) c. Secobarbital (Seconal) d. Temazepam (Restoril)

ANS: A Butabarbital is an intermediate-acting barbiturate and is useful as a sleep sustainer to maintain long periods of sleep. They have an onset of 1 hour, so are not useful for those who have trouble falling asleep. Flurazepam and temazepam are benzodiazepines and are used to induce sleep. Secobarbital is used for preoperative sedation.

1. A nursing student asks the nurse to explain the role of cyclooxygenase-2 (COX-2) and its role in inflammation. The nurse will explain that COX-2 a. converts arachidonic acid into a chemical mediator for inflammation. b. directly causes vasodilation and increased capillary permeability. c. irritates the gastric mucosa to cause gastrointestinal upset. d. releases prostaglandins, which cause inflammation and pain in tissues.

ANS: A COX-2 is an enzyme that converts arachidonic acid into prostaglandins and their products, and this synthesis causes pain and inflammation. They do not act directly to cause inflammation. COX-1 irritates the gastric mucosa. COX-2 synthesizes but does not release prostaglandins.

13. A college-age student is brought to the emergency department by friends after consuming NoDoz tablets along with several cups of coffee and a few energy drinks. The patient is complaining of nausea and diarrhea and appears restless. The nurse understands that a. arrhythmias and convulsions may occur. b. caffeine dependence does not occur. c. effects of the substances will wear off shortly. d. severe adverse effects do not occur.

ANS: A Caffeine and other stimulants can cause cardiac arrhythmias and seizures. Caffeine dependence may occur.

7. A patient who is receiving chemotherapy will be given dronabinol (Marinol) to prevent nausea and vomiting. The nurse will tell the patient that this drug will be given at which time? a. Before and after the chemotherapy b. During chemotherapy c. Immediately prior to chemotherapy d. 24 hours prior to chemotherapy

ANS: A Cannabinoids are given prior to chemotherapy and for 24 hours after chemotherapy.

14. A patient is receiving high doses of a cephalosporin. Which laboratory values will this patient's nurse monitor closely? a. Blood urea nitrogen (BUN), serum creatinine, and liver function tests b. Complete blood count and electrolytes c. Serum calcium and magnesium d. Serum glucose and lipids

ANS: A Cefazolin will produce an increase in the patient's BUN, creatinine, AST, ALT, ALP, LDH, and bilirubin.

3. A patient begins taking cholestyramine (Questran) to treat hyperlipidemia. The patient reports abdominal discomfort and constipation. The nurse will provide which instruction to the patient? a. Increase fluid and slowly increase fiber intake. b. Stop taking the medication immediately. c. Take an over-the-counter laxative. d. Take the medication on an empty stomach.

ANS: A Cholestyramine can cause gastrointestinal upset and constipation, and these symptoms can be reduced with increased fluids and foods high in fiber. Stopping the medication is not indicated. Over-the-counter laxatives are not recommended until other methods have been tried. Giving the medication on an empty stomach will not relieve the discomfort.

1. The nurse is caring for a patient who has increased intraocular pressure. The provider has ordered a cholinergic agonist. The nurse will perform a thorough health history to make sure the patient does not have a history of which condition? a. Asthma b. Diabetes c. Hypertension d. Renal disease

ANS: A Cholinergic agonists can worsen conditions such as asthma and chronic bronchitis because they can cause airway constriction.

8. A patient who takes phenytoin reports regular alcohol consumption. The nurse might expect a serum phenytoin level in this patient to be in which range? a. 5 to 10 mcg/mL b. 10 to 20 mcg/mL c. 20 to 30 mcg/mL d. 30 to 50 mcg/mL

ANS: A Chronic ingestion of alcohol increases hydantoin metabolism, which would decrease serum drug levels. The therapeutic range is 10 to 20 mcg/mL, so a level lower than this may be expected in patients who consume alcohol regularly.

8. The nurse is caring for a patient who has Zollinger-Ellison syndrome. Which medication order would the nurse question for this patient? a. Cimetadine (Tagamet) b. Pantoprazole (Protonix) c. Rabeprazole (Aciphex) d. Ranitidine (Zantac)

ANS: A Cimetidine is not effective for treating Zollinger-Ellison syndrome. The other medications are used to treat Zollinger-Ellison syndrome.

2. A patient reports having recurring headaches described as 1 to 2 headaches per day for several weeks. The nurse understands that these headaches are most likely descriptive of which type of headache? a. Cluster headache b. Migraine headache c. Simple headache d. Tension headache

ANS: A Cluster headaches reoccur 1 to 3 times daily in a period lasting from approximately 2 weeks to 3 months. Migraine headaches are severe and characterized by an aura prior to the headache. Tension headaches are related to stress.

4. A patient has been taking cholestyramine (Questran) to treat hyperlipidemia type II. The patient reports abdominal cramping and constipation. The patient's serum low- density lipoprotein (LDL) has decreased from 170 mg/dL to 110 mg/dL, and triglycerides have not changed from 150 mg/dL since beginning the medication. The provider changes the medication to colesevelam HCl (Welchol).The patient asks the nurse why the medication was changed, and the nurse will explain that colesevelam HCl is ordered for which reason? a. It has fewer side effects. b. It has more convenient dosing. c. It provides greater LDL reduction. d. It provides greater triglyceride reduction.

ANS: A Colesevelam is similar to cholestyramine but has fewer gastrointestinal side effects. This patient has demonstrated good results with the bile acid sequestrant, so the provider needs to offer a preparation with fewer adverse effects. Both drugs are given twice daily.

5. A patient who has cancer will begin treatment with a colony-stimulating factor. The patient verbalizes understanding of this drug's action with which statement? a. "This drug allows higher doses of chemotherapy." b. "This drug has antitumor activity." c. "This drug has cytotoxic effects." d. "This drug has antiviral effects."

ANS: A Colony-stimulating factors permit the delivery of higher doses of drugs because they counter myelosuppression. They do not have antitumor activity, cytotoxic effects, or antiviral effects.

12. A patient has been taking a drug that has a protein-binding effect of 75%. The provider adds a new medication that has a protein-binding effect of 90%. The nurse will expect a. decreased drug effects of the first drug. b. decreased therapeutic range of the first drug. c. increased drug effects of the first drug. d. increased therapeutic range of the first drug.

ANS: C Adding another highly protein-bound drug will displace the first drug from protein-binding sites and release more free drug increasing the drug's effects. This does not alter the therapeutic range, which is the serum level between drug effectiveness and toxicity.

1. The nurse is preparing to administer a drug and learns that it is an indirect-acting cholinergic agonist. The nurse understands that this drug a. acts on muscarinic receptors. b. acts on nicotinic receptors. c. inhibits cholinesterase. d. inhibits cholinergic receptors.

ANS: C Agents that inhibit cholinesterase, which is the enzyme that destroys acetylcholine, indirectly enhance the actions of acetylcholine.

17. The nurse is caring for a patient who has taken an overdose of aspirin several hours prior. The provider orders sodium bicarbonate to be given. The nurse understands that this drug is given for which purpose? a. To counter the toxic effects of the aspirin b. To decrease the half-life of the aspirin c. To increase the excretion of the aspirin d. To neutralize the acid of the aspirin

ANS: C Aspirin is a weak acid and is more readily excreted in alkaline urine. Sodium bicarbonate alkalizes the urine. It does not act as an antidote to aspirin, decrease the half-life, or neutralize its pH.

14. The nurse gives 800 mg of a drug that has a half-life of 8 hours. How much drug will be left in the body in 24 hours if no additional drug is given? a. None b. 50 mg c. 100 mg d. 200 mg

ANS: C Eight hours after the drug is given, there will be 400 mg left. Eight hours after that (16 hours), there will be 200 mg left. At 24 hours, there will be 100 mg left.

3. The nurse is teaching a patient who will be discharged home with a prescription for an enteric-coated tablet. Which statement by the patient indicates understanding of the teaching? a. "I may crush the tablet and put it in applesauce to improve absorption." b. "I should consume acidic foods to enhance absorption of this medication." c. "I should expect a delay in onset of the drug's effects after taking the tablet." d. "I should take this medication with high-fat foods to improve its action."

ANS: C Enteric-coated tablets resist disintegration in the acidic environment of the stomach and disintegrate when they reach the small intestine. There is usually some delay in onset of actions after taking these medications. Enteric-coated tablets should not be crushed or chewed, which would alter the time and location of absorption. Acidic foods will not enhance the absorption of the medication. The patient should not to eat high-fat food before ingesting an enteric-coated tablet, because high-fat foods decrease the absorption rate.

4.The client is being treated extensively with silver nitrate cream. The nurse plans to closely monitor the client's level of: a.calcium. b.sodium. c.potassium. d. magnesium.

ANS: C Extensive use of silver nitrate can lead to hypokalemia.

6. A nurse is preparing to administer an oral drug that is best absorbed in an acidic environment. How will the nurse give the drug? a. On an empty stomach b. With a full glass of water c. With food d. With high-fat food

ANS: C Food can stimulate the production of gastric acid so medications requiring an acidic environment should be given with a meal. High-fat foods are useful for drugs that are lipid soluble.

1.A client is diagnosed with acne. The client expresses concern to the nurse that he has not yet been placed on medication. The nurse explains to the client that the initial nonpharmacologic approach for treating acne vulgaris includes: a. application of large doses of vitamin A. b.application of large doses of vitamin C. c.cleansing of the skin gently several times a day. d. vigorously scrubbing skin in the morning and at bedtime.

ANS: C Gentle cleansing is one of the chief nonpharmacologic treatments of acne.

5. A provider has prescribed ipratropium bromide/albuterol sulfate (Combivent) for a patient who has chronic obstructive pulmonary disease (COPD). The nurse explains that this combination product is prescribed primarily for which reason? a. To be more convenient for patients who require both medications b. To improve compliance in patients who may forget to take both drugs c. To increase forced expiratory volume, an indicator of symptom improvement d. To minimize the side effects that would occur if the drugs are given separately

ANS: C Combivent is more effective and has a longer duration of action than if either agent is used alone, and the two agents combined increase the FEV1. While it is more convenient and may improve compliance, this is not the primary reason for using it. The combination does not alter drug side effects.

11. The nurse is caring for a patient who has recurrent muscle spasms. The provider has ordered metaxalone (Skelaxin) to treat the spasms. The nurse learns that the patient has a history of drug and alcohol abuse. The nurse will contact the provider to discuss switching this patient to which medication? a. Carisoprodol (Soma) b. Chlorzoxazone (Parafon forte DSC) c. Cyclobenzaprine (Flexeril) d. Methocarbamol (Robaxin)

ANS: C Cyclobenzaprine is a muscle relaxant that does not cause drug dependence. The other muscle relaxants can cause drug dependence.

11. Which cholinesterase inhibitor would be prescribed for a patient who has Alzheimer's disease? a. Ambenonium chloride (Myletase) b. Benztropine (Cogentin) c. Donepezil HCl (Aricept) d. Neostigmine methylsulfate (Prostigmin)

ANS: C Donepezil is used to treat Alzheimer's disease. Ambenonium and neostigmine are used to treat myasthenia gravis. Benztropine is used to treat Parkinson's disease.

14. A patient is brought to the emergency department with a drug overdose causing respiratory depression. Which drug will the nurse expect to administer? a. Albuterol (Proventil) b. Caffeine (Cafcit) c. Doxapram (Dopram) d. Methylphenidate (Ritalin)

ANS: C Doxapram is given to treat respiratory depression caused by drug overdose.

10. The nurse is caring for a patient who will begin taking doxycycline to treat an infection. The nurse should plan to give this medication a. 1 hour before or 2 hours after a meal. b. with an antacid to minimize GI irritation. c. with food to improve absorption. d. with small sips of water.

ANS: C Doxycycline is a lipid-soluble tetracycline and is better absorbed when taken with milk products and food. It should not be taken on an empty stomach. Antacids impair absorption of tetracyclines. Small sips of water are not necessarily indicated.

3. The nurse is caring for several patients who are receiving antibiotics. Which order will the nurse question? a. Azithromycin (Zithromax) 500 mg IV in 500 mL fluid b. Azithromycin (Zithromax) 500 mg PO once daily c. Erythromycin 300 mg IM QID d. Erythromycin 300 mg PO QID

ANS: C Erythromycin and other macrolides should not be given intramuscularly because they cause painful tissue irritation.

2. A patient is diagnosed with mycoplasma pneumonia. Which antibiotic will the nurse expect the provider to order to treat this infection? a. Azithromycin (Zithromax) b. Clarithromycin (Biaxin) c. Erythromycin (E-Mycin) d. Fidaxomicin (Dificid)

ANS: C Erythromycin is the drug of choice for treating mycoplasma pneumonia.

12. A patient who has type 1 diabetes mellitus asks the nurse about using a combination insulin product such as Humalog 75/25. The nurse will tell the patient that use of this product a. depends on individual insulin needs. b. is useful for patient with insulin resistance. c. means less rotation of injection sites. d. requires refrigeration at all times.

ANS: A Combination products are convenient because the patient does not have to mix insulin, but the products depend on individual needs, since the doses are fixed. They are not used for patients with insulin resistance. Patients must continue to rotate injection sites. They do not require refrigeration after first use.

7. The nurse administers intravenous corticotropin (Acthar) to a patient. A serum cortisol level drawn 60 minutes later shows no change in serum cortisol levels from prior to the dose. What is the nurse's first action? a. Notify the provider to discuss a possible non-functioning adrenal gland. b. Recognize the need for an increased dose to treat pituitary insufficiency. c. Request an order for a second dose of corticotropin to treat cortisone deficiency. d. Request an order to repeat the serum cortisol level in 1 to 2 hours.

ANS: A Corticotropin is given to diagnose adrenal gland disorders as well as to treat adrenal gland insufficiency. When given intravenously, the serum cortisol level should increase within 30 to 60 minutes if the adrenal gland is functioning. The nurse should report adrenal gland dysfunction. The provider will determine how to treat. Since the levels should increase in 30 to 60 minutes, there is no need to repeat the test in 1 to 2 hours.

15. A patient who is using inhaled cromolyn sodium (Intal) daily calls the clinic to report experiencing cough and a bad taste. The nurse will instruct the patient to perform which action? a. Drink water before and after using the inhaler. b. Schedule an appointment to discuss these effects with the provider. c. Stop taking the medication immediately. d. Use the inhaler only as needed for acute bronchospasms.

ANS: A Cough and a bad taste are the most common side effects associated with cromolyn sodium, and these effects can be decreased by drinking water before and after using the drug. The effects are not serious and do not warrant discussion with the provider. Stopping the medication abruptly can cause a rebound bronchospasm. This medication is not useful in acute bronchospasm.

12. An intubated child is brought to the emergency department while having a seizure that has been progressing for 20 minutes. Which drug will the nurse anticipate administering to this patient? a. Diazepam (Valium) b. Phenobarbital (Luminal) c. Phenytoin (Dilantin) d. Valproic acid (Depakote)

ANS: A Diazepam is given to patients in status epilepticus and is administered IV. The other anticonvulsant medications do not have a rapid onset and are not used for emergencies.

6. The nurse is preparing to administer dipivefrin (Propine) drops as a mydriatic agent. Which assessment would cause the nurse to withhold the drug and notify the provider? a. Blood pressure of 140/90 mm Hg b. Heart rate of 60 beats per minute c. Respiratory rate of 12 breaths per minute d. Temperature of 37.9° C

ANS: A Dipivefrin is a prodrug of epinephrine and can cause systemic cardiovascular side effects such as tachycardia and hypertension. An elevated blood pressure warrants holding the drug and notifying the provider.

4. The nurse is teaching a patient who is about to take a long car trip about using dimenhydrinate (Dramamine) to prevent motion sickness. What information is important to include when teaching this patient? a. "Do not drive while taking this medication." b. "Dry mouth is a sign of toxicity with this mediation." c. "Take the medication 1 to 2 hours prior to beginning the trip." d. "Take 100 mg up to 6 times daily for best effect."

ANS: A Drowsiness is a common side effect of dimenhydrinate, so patients should be cautioned against driving while taking this drug. Dry mouth is a common side effect and not a sign of toxicity. The drug should be taken 30 minutes prior to travel. The maximum recommended dose is 400 mg per day.

5. The nurse is preparing to administer phenytoin (Dilantin) to a patient who has a seizure disorder. The patient appears drowsy, and the nurse notes that the last random serum drug level was 18 mcg/mL. What action will the nurse take? a. Administer the dose since the patient is not toxic. b. Contact the provider to discuss decreasing the phenytoin dose. c. Give the drug and monitor closely for adverse effects. d. Report drug toxicity to the providers.

ANS: A Drowsiness is a common side effect of phenytoin and is not cause for alarm. The patient's drug level is normal, since 10-20 mcg/mL is the therapeutic range. The nurse should administer the dose. It is not necessary to decrease the dose or monitor the patient more closely than usual.

8. The nurse is reviewing medication information with a nursing student prior to administering an oral drug and notes that the drug has extensive first-pass effects. Which statement by the student indicates a need for further teaching about this medication? a. "The first-pass effect means the drug may be absorbed into systemic circulation from the intestinal lumen." b. "The first-pass effect means the drug may be changed to an inactive form and excreted." c. "The first-pass effect means the drug may be changed to a metabolite, which may be more active than the original." d. "The first-pass effect means the drug may be unchanged as it passes through the liver."

ANS: A Drugs that undergo first-pass metabolism are absorbed into the portal vein from the intestinal lumen and go through the liver where they are either unchanged or are metabolized to an inactive or a more active form.

8. The nurse provides teaching for a patient receiving corticotropin. The nurse will instruct the patient to contact the provider if which condition occurs? a. Bruising b. Constipation c. Myalgia d. Nausea

ANS: A Ecchymosis is an adverse reaction to corticotropin and should be reported. Constipation and nausea are known side effects but are not serious. Myalgia is not common.

4. A patient who has received heparin after previous surgeries will be given enoxaparin sodium (Lovenox) after knee-replacement surgery. The patient asks how this drug is different from heparin. The nurse will explain that enoxaparin a. decreases the need for laboratory tests. b. has a shorter half-life than heparin. c. increases the risk of hemorrhage. d. may be taken orally instead of subcutaneously.

ANS: A Enoxaparin is a low-molecular-weight heparin, which produces more stable responses at lower doses, thus reducing the need for frequent lab monitoring. It has a longer half-life than heparin. It decreases the risk of hemorrhage because it is more stable at lower doses. It is given subcutaneously.

9. A patient who has been taking theophylline at home reports having palpitations and jitteriness. What action will the nurse take? a. Ask the patient if herbal medications are used. b. Notify the provider to report theophylline toxicity. c. Recommend that the patient increase fluid intake. d. Request an order for renal function studies.

ANS: A Ephedra is a stimulant that potentiates theophylline and may increase side effects. Patients should be questioned about use of herbal medications. To determine toxicity, serum drug levels must be drawn; at this point, the patient reports symptoms of theophylline side effects. Increasing fluid intake will not alleviate symptoms. Renal function studies are not indicated.

5. The nurse is caring for a postoperative older patient who received PO hydrocodone with acetaminophen (Lortab) 45 minutes prior after reporting a pain level of 8 on a scale of 1 to 10. The patient reports a pain level of 4, and the nurse notes a respiratory rate of 20 breaths per minute, a heart rate of 92 beats per minute, and a blood pressure of 170/95 mm Hg. Which action will the nurse take? a. Contact the provider and request an order for a more potent opioid analgesic. b. Reassess the patient in 30 minutes. c. Request an order for ibuprofen to augment the opioid analgesic. d. Suggest that the patient use nonpharmacologic measures to relieve pain.

ANS: A Even though the patient reports decreased pain, the patient's vital signs indicate continued discomfort. The nurse should contact the provider to request a stronger analgesic. The pain medication should have been effective within 30 minutes. Ibuprofen is used for musculoskeletal pain. Nonpharmacologic measures may be useful, but the patient still needs a stronger analgesic.

6. A patient begins taking nicotinic acid (Niacin) and reports dizziness and flushing of the skin. The nurse will perform which action? a. Contact the provider to discuss decreasing the dose. b. Counsel the patient to increase fluid intake. c. Request an order for renal function tests. d. Schedule the medication to be taken with meals.

ANS: A Flushing of the skin and dizziness are common side effects of nicotinic acid, but with careful drug titration and concomitant use of aspirin, these effects can be minimized. Increasing fluid intake or taking with food does not alter these adverse effects. Nicotinic acid can affect liver enzymes not renal function.

2. A patient who is about to begin chemotherapy asks the nurse when the risk of infection is highest. The nurse will tell the patient that infection risk is greatest at which point? a. A week to 10 days after each chemotherapy dose b. During the week immediately after chemotherapy c. Immediately prior to each dose of chemotherapy d. When the patient's temperature is elevated by 1° F

ANS: A Following chemotherapy administration, the time at which the blood count, including white blood cells, is lowest is typically 7 to 10 days after treatment.

9. A patient is preparing to travel to a country with prevalent malaria. To prevent contracting the disease, the provider has ordered chloroquine HCl (Aralen). The nurse will instruct the patient to take this drug according to which schedule? a. 500 mg weekly beginning 2 weeks prior to travel and continuing for 6 to 8 weeks after travel b. 1000 mg weekly beginning 2 weeks prior to travel and continuing for 6 to 8 weeks after travel c. 500 mg once followed by 500 mg per dose in 6 hours, 24 hours, and 48 hours d. 1000 mg once followed by 500 mg per dose in 6 hours, 24 hours, and 48 hours

ANS: A For malaria prophylaxis, chloroquine is given 500 mg/dose weekly for 2 weeks prior to travel and then weekly until 6 to 8 weeks after exposure. The dosing schedule of 1000 mg once, followed by 500 mg in 6, 24, and 48 hours is used to treat acute malaria.

17. A patient who has type 2 diabetes mellitus asks the nurse why the provider has changed the oral antidiabetic agent from tolbutamide (Orinase) to glipizide (Glucotrol). The nurse will explain that glipizide a. has a longer duration of action. b. has fewer gastrointestinal side effects. c. may be taken on an as-needed basis. d. results in less hypoglycemic potential.

ANS: A Glipizide is a second-generation oral antidiabetic agent. It has a longer duration of action than the first-generation antidiabetic agents such as tolbutamide. It has many gastrointestinal side effects. It is taken once daily, not as needed. It has greater hypoglycemic activity than first-generation antidiabetics.

9. A patient will begin taking albuterol (Proventil) to treat asthma. When teaching the patient about this drug, the nurse will make which recommendation? a. Report rapid or irregular heart rate. b. Drink 8 to 16 extra ounces of fluid each day. c. Monitor serum glucose daily. d. Take a calcium supplement.

ANS: A High dosages of albuterol may affect beta1 receptors, causing an increase in heart rate that could be dangerous. It is not necessary to consume extra fluids or take a calcium supplement while using this drug. Serum glucose may be elevated slightly, but this is not a concern in non-diabetic patients.

12. The nurse is caring for a patient who is receiving a high dose of tetracycline (Sumycin). Which laboratory values will the nurse expect to monitor while caring for this patient? a. Blood urea nitrogen (BUN) and creatinine levels b. Complete blood counts c. Electrolytes d. Liver enzyme levels

ANS: A High doses of tetracyclines can lead to nephrotoxicity, especially when given along with other nephrotoxic drugs. Renal function tests should be performed to monitor for nephrotoxicity.

4. The nurse is caring for a patient who has a seizure disorder. The nurse notes that the patient has reddened gums that bleed when oral care is given. The nurse recognizes this finding as a. an adverse effect of the phenytoin. b. a drug interaction with aspirin. c. a symptom of hepatotoxicity. d. a sign of poor self-care.

ANS: A Hydantoins commonly cause gingival hyperplasia, which causes overgrowth of reddened gum tissue that bleeds easily. It is not a sign of a drug interaction or a symptom of hepatotoxicity. It does not indicate a lack of self-care.

5. The nurse is caring for a patient who is diagnosed with tuberculosis. The patient tells the nurse that the provider plans to order a prophylactic antitubercular drug for family members and asks which drug will be ordered. The nurse will expect the provider to order which drug? a. Isoniazid (INH) b. Pyrazinamide c. Rifampin (Rifadin) d. Streptomycin

ANS: A INH is the drug of choice for prophylactic treatment of patients who have had close contact with a patient who has tuberculosis.

8. A patient has been receiving an erythropoietin-stimulating agent (ESA) for 8 weeks. The nurse reviews the patient's chart and notes no increase in hemoglobin levels from 8 g/dL on week 3 of therapy. The nurse will request an order for a. a complete blood count and serum iron levels. b. an increased dose of the erythropoietin-stimulating agent. c. more frequent dosing of the ESA. d. packed red blood cell infusions.

ANS: A If there is no response, ESAs should be discontinued after 8 weeks of therapy. If a patient does not respond, iron deficiency or underlying hematologic disease should be considered and evaluated.

10. The nurse notes a blood pressure of 160/90 mm Hg in a patient taking a thiazide diuretic. The patient reports taking an herbal medication that a friend recommended. Which herbal product is likely, given this patient's blood pressure? a. Ginkgo b. Hawthorne c. Licorice d. St. John's wort

ANS: A Increased blood pressure can result when ginkgo is used in combination with a thiazide diuretic. Hawthorne can potentiate hypotension. Licorice can increase potassium loss, leading to hypokalemia. St. John's wort is not listed as an herbal alert substance with thiazide diuretics.

17. The nurse is reviewing a patient's chart prior to administering gentamycin (Garamycin) and notes that the last serum peak drug level was 9 mcg/mL and the last trough level was 2 mcg/mL. What action will the nurse take? a. Administer the next dose as ordered. b. Obtain repeat peak and trough levels before giving the next dose. c. Report possible drug toxicity to the patient's provider. d. Report a decreased drug therapeutic level to the patient's provider.

ANS: C Gentamycin peak values should be 5 to 8 mcg/mL, and trough levels should be 0.5 to 2 mcg/mL. Peak levels give information about whether or not a drug is at toxic levels, while trough levels indicate whether a therapeutic level is maintained. This drug is at a toxic level, and the next dose should not be given.

18. A patient is diagnosed with tinea capitis. The provider will order which systemic antifungal medication for this patient? a. Anidulafungin (Eraxis) b. Fluconazole (Diflucan) c. Griseofulvin (Fulvicin) d. Ketoconazole (Nizoral)

ANS: C Griseofulvin is used to treat tinea capitis. Anidulafungin is used to treat esophageal candidiasis, candidemia, and other Candida infections. Fluconazole is used to treat Candida infections and cryptococcal meningitis. Ketoconazole is used to treat Candida infections, histoplasmosis, blastomycosis, and other infections.

1. The parents of an 11-year-old boy ask about growth hormone therapy for their child who is shorter than his 10-year-old sister. The nurse will tell the parents that growth hormone a. does not affect other hormones when given. b. is available as an oral tablet to be taken once daily. c. is given after tests prove that it is necessary. d. may be given until the child's desired height is reached.

ANS: C Growth hormone is given only when growth hormone deficiency is determined. It cannot be given orally. It antagonizes insulin secretion and thus can lead to the development of diabetes mellitus. It cannot be given after the epiphyses are fused.

3. The nurse is caring for a patient who is receiving a third dose of high-dose cyclophosphamide (Cytoxan). The nurse notes hematuria. The nurse will notify the provider and will perform which action? a. Ask whether the patient takes allopurinol (Lopurin). b. Assess the patient's skin and fingernails for darkening. c. Question the patient about fluid intake. d. Reassure the patient that this is an expected side effect.

ANS: C Hemorrhagic cystitis is a common adverse effect of high-dose cyclophosphamide and can be mitigated by increasing fluid intake. Allopurinol is given to treat gout, which is characterized by uric acid crystalluria. Darkening of the fingernails and skin is a common adverse effect of cyclophosphamide but is unrelated to hemorrhagic cystitis.

4. The nurse is caring for a patient who is receiving a high dose of intravenous azithromycin to treat an infection. The patient is also taking acetaminophen for pain. The nurse should expect to review which lab values when monitoring for this drug's side effects? a. Complete blood counts b. Electrolytes c. Liver enzymes d. Urinalysis

ANS: C High doses of macrolides, when taken with other, potentially hepatotoxic drugs such as acetaminophen may cause hepatotoxicity, so liver enzymes should be carefully monitored.

5. The nurse is teaching a patient how to administer insulin. The patient is thin with very little body fat. The nurse will suggest injecting insulin a. by pinching up the skin and injecting straight down. b. in the abdomen only with the needle at a 90-degree angle. c. subcutaneously with the needle at a 45- to 60-degree angle. d. using the thigh and buttocks areas exclusively.

ANS: C In a thin person, with little fatty tissue, the needle is inserted at a 45- to 60-degree angle. In other patients, a 45- to 90-degree angle is acceptable. There is no recommendation for preferring one site over another.

11. The nurse is caring for a postpartum woman who is refusing opioid analgesics but is rating her pain as a 7 or 8 on a 10-point pain scale. The nurse will contact the provider to request an order for which analgesic medication? a. Diclofenac sodium (Voltaren) b. Ketoprofen (Orudis) c. Ketorolac (Toradol) d. Naproxyn (Naprosyn)

ANS: C Ketorolac is the first injectable NSAID and has shown analgesic efficacy equal or superior to that of opioid analgesics. The other NSAIDs listed are not used for postoperative pain.

4. The nurse is performing an admission assessment on an adolescent who reports taking extra-strength acetaminophen (Tylenol) regularly to treat daily headaches. The nurse will notify the patient's provider and discuss an order for a. a selective serotonin receptor agonist (SSRA). b. hydrocodone with acetaminophen for headache pain. c. liver enzyme tests. d. serum glucose testing.

ANS: C Large doses or overdoses of acetaminophen can be toxic to hepatic cells, so when large doses are administered over a long period, liver function should be assessed. Daily headaches are not typical of migraine headaches, so SSRA medication is not indicated. Hydrocodone with acetaminophen is not indicated without further evaluation of headaches. Serum glucose is not indicated.

18. The nurse is providing discharge teaching for a patient who will receive oral levofloxacin (Levaquin) to treat pneumonia. The patient takes an oral hypoglycemic medication and uses over-the-counter (OTC) antacids to treat occasional heartburn. The patient reports frequent arthritis pain and takes acetaminophen when needed. Which statement by the nurse is correct when teaching this patient? a. "You may take antacids with levofloxacin to decrease gastrointestinal upset." b. "You may take nonsteroidal anti-inflammatory medications (NSAIDs) for arthritis pain." c. "You should monitor your serum glucose more closely while taking levofloxacin." d. "You should take levofloxacin on an empty stomach to improve absorption."

ANS: C Levofloxacin may increase the effects of oral hypoglycemic medications, so patients taking these should be advised to monitor their serum glucose levels closely. Antacids decrease the absorption of levofloxacin. NSAIDs taken with levofloxacin can cause central nervous system reactions, including seizures.

4. The nurse is teaching a patient who is newly diagnosed with type 1 diabetes mellitus about insulin administration. Which statement by the patient indicates a need for further teaching? a. "I may use a chosen site daily for up to a week." b. "I should give each injection a knuckle length away from a previous injection." c. "I will not be concerned about a raised knot under my skin from injecting insulin." d. "Insulin is absorbed better from subcutaneous sites on my abdomen.

ANS: C Lipohypertrophy is a raised lump or knot on the skin surface caused by repeated injections into the same site, and this can interfere with insulin absorption. Patients are encouraged to use the same site for a week, giving each injection a knuckle length away from the previous injection. Insulin absorption is greater when given in abdominal areas.

6. The nurse is preparing to administer clarithromycin to a patient. When performing a medication history, the nurse learns that the patient takes warfarin to treat atrial fibrillation. The nurse will perform which action? a. Ask the provider if azithromycin may be used instead of clarithromycin. b. Obtain an order for continuous cardiovascular monitoring. c. Request an order for periodic serum warfarin levels. d. Withhold the clarithromycin and notify the provider.

ANS: C Macrolides can increase serum levels of other drugs such as warfarin. If these drugs are used with macrolides, serum drug levels should be monitored. All macrolides have this drug interaction. Cardiovascular monitoring is not indicated. The drug may be given as long as serum drug levels are monitored.

21. A pregnant woman who is in labor has a blood pressure of 189/110 mm Hg and exhibits muscle contractions followed by jerking of her arms and legs. The nurse will prepare to administer which medication to this patient? a. Carbamazepine (Tegretol) b. Diazepam (Valium) c. Magnesium sulfate d. Phenobarbital (Luminal)

ANS: C Magnesium sulfate is used to control seizures during eclampsia.

14. A child is diagnosed with pinworms. Which anthelmintic drug will the provider order for this child? a. Bithionol (Bitin) b. Diethylcarbamazine (Hetrazan) c. Mebendazole (Vermox) d. Praziquantel (Biltricide)

ANS: C Mebendazole is used to treat pinworms. The other drugs treat other types of parasites.

18. A patient who has been taking a sulfonylurea antidiabetic medication will begin taking metformin (Glucophage). The nurse understands that this patient is at increased risk for which condition? a. Hypoglycemia b. Hyperglycemia c. Renal failure d. Respiratory distress

ANS: C Metformin can lead to renal failure. It does not produce hypoglycemia or hyperglycemia. It does not increase the risk of respiratory distress.

15. A patient reports difficulty staying awake during the daytime in spite of getting adequate sleep every night. Which medication will the nurse expect the provider to order for this patient? a. Caffeine (NoDoz) b. Methylphenidate (Ritalin) c. Modafinil (Provigil) d. Theophylline

ANS: C Modafinil is given to treat narcolepsy.

1. A patient who experiences motion sickness when flying asks the nurse the best time to take the medication prescribed to prevent motion sickness for a 0900 flight. The nurse will instruct the patient to take the medication at which time? a. As needed, at the first sign of nausea b. At 0700, before leaving for the airport c. At 0830, just prior to boarding the plane d. When seated, just prior to takeoff

ANS: C Motion sickness medication has its onset in 30 minutes. The patient should be instructed to take the medication a half hour prior to takeoff. It is not used as needed.

14. A patient reports experiencing flatulence and abdominal distension to the nurse. Which over-the-counter medication will the nurse recommend? a. Alka-Seltzer b. Maalox c. Mylicon d. Tums

ANS: C Mylicon is a brand-name simethicone, which is an anti-gas agent. Maalox Gas contains simethicone, while regular Maalox does not. The other products do not contain simethicone.

8. The patient who has nasal congestion asks the nurse to recommend a decongestant medication. The nurse performs a medication history and learns that the patient takes a beta blocker to treat hypertension. Which over-the-counter product will the nurse recommend? a. Diphenhydramine (Benadryl) b. Ephedrine HCl (Pretz-D) c. Phenylephrine nasal (NeoSynephrine Nasal) d. Loratadine (Claritin)

ANS: C NeoSynephrine Nasal is a topical decongestant and causes less systemic side effects than ephedrine, which should not be given with beta blockers. Diphenhydramine and loratadine are antihistamines, not decongestants.

3. The nurse assumes care of a patient who has myasthenia gravis and notes that a dose of neostigmine (Prostigmin) due 1 hour prior was not given. The nurse will anticipate the patient to exhibit which symptoms? a. Excessive salivation b. Muscle spasms c. Muscle weakness d. Respiratory paralysis

ANS: C Neostigmine must be given on time to prevent myasthenic crisis, which is characterized by generalized, severe muscle weakness. The other symptoms are characteristic of cholinergic crisis, caused by too much medication.

3. A patient is diagnosed with influenza and will begin taking a neuraminidase inhibitor. The nurse knows that this drug is effective when taken within how many hours of onset of flu symptoms? a. 12 hours b. 24 hours c. 48 hours d. 72 hours

ANS: C Neuraminidase inhibitors, such as zanamivir and oseltamivir, should be taken within 48 hours of onset of symptoms for best effect.

3. A patient receiving interferon experiences confusion, somnolence, and aphasia. The nurse will perform which action? a. Discontinue the medication immediately. b. Inform the family that these symptoms may persist for years. c. Reassure the patient that these side effects are reversible. d. Request an order for lorazepam.

ANS: C Neurologic side effects, such as confusion, somnolence, and aphasia, are reversible after the drug is stopped. It is not necessary to stop the medication unless the symptoms progress and become severe. Lorazepam is not indicated.

7. The nurse is caring for an 80-year-old patient who has just begun taking a thiazide diuretic to treat hypertension. What is an important aspect of care for this patient? a. Encouraging increased fluid intake b. Increasing activity and exercise c. Initiating a fall risk protocol d. Providing a low potassium diet

ANS: C Older patients experience a higher risk of orthostatic hypotension when taking antihypertensive medications. Fall risk also increases with a need for increased trips to the bathroom. A fall risk protocol should be implemented. Increasing fluids and activity and limiting potassium are not indicated.

18. Which muscle relaxant is used in surgery as a skeletal muscle relaxant? a. Baclofen (Lioresal) b. Chlorzoxazone (Parafon forte) c. Pancuronium bromide (Pavulon) d. Methocarbamol (Robaxin)

ANS: C Pancuronium bromide is used as a depolarizing muscle relaxant during anesthesia.

2. A nursing student asks the nurse to differentiate the pathology of Alzheimer's disease from that of Parkinson's disease. Which description is correct? a. Alzheimer's disease involves a possible excess of acetylcholine and neuritic plaques. b. Alzheimer's disease is caused by decreased amounts of dopamine and degeneration of cholinergic neurons. c. Parkinson's disease is characterized by an imbalance of dopamine and acetylcholine. d. Parkinson's disease involves increased dopamine production and decreased acetylcholine.

ANS: C Parkinson's disease (PD) is characterized by an imbalance of dopamine (DA) and acetylcholine (ACh) caused by an unexplained degeneration of the dopaminergic neurons allowing the excitatory response of acetylcholine to exceed the inhibitory response of dopamine. Alzheimer's disease (AD) may result from decreased ACh, degeneration of cholinergic neurons, and neuritic plaques. Dopamine does not appear to play a role in Alzheimer's disease.

21. A patient is taking prednisolone and fludrocortisone (Florinef). When teaching this patient about dietary intake, the nurse will instruct the patient to consume a diet a. high in carbohydrates. b. high in fat. c. high in protein. d. low in potassium.

ANS: C Patients receiving fludrocortisone are at risk for negative nitrogen balance and should consume a high-protein diet.

8. The nurse is teaching a patient about home administration of insulin. The patient will receive regular (Humulin R) and NPH (Humulin NPH) insulin at 0700 every day. What is important to teach this patient? a. "Draw up the medications in separate syringes." b. "Draw up the NPH insulin first." c. "Draw up the regular insulin first." d. "Draw up the medications after mixing them in a vial."

ANS: C Patients should be instructed to draw up regular insulin first so that NPH is not mixed into the vial of regular insulin. It is not necessary to use separate syringes. Patients do not mix the medications in a vial.

7. The nurse is teaching a patient who will receive acyclovir for a herpes virus infection. What information will the nurse include when teaching this patient? a. Blood cell counts should be monitored closely. b. Dizziness and confusion are harmless side effects. c. Increase fluid intake while taking this medication. d. Side effects are rare with this medication.

ANS: C Patients taking acyclovir should increase fluid intake to maintain hydration. A complete blood count is not required. Dizziness and confusion should be reported to the provider. Antiviral medications have many side effects.

10. A patient is taking chloroquine (Aralen) to treat acute malaria. Which statement by the patient indicates understanding of this medication? a. "I should abstain from alcohol while taking this medication." b. "I should report urine output less than 1000 mL/day." c. "I should report visual changes immediately." d. "I should take this drug on an empty stomach."

ANS: C Patients taking chloroquine (Aralen) have a risk of visual injury related to side effects of blurred vision and should report visual changes to the provider. There is no restriction on alcohol. Patient should report urine output of less than 600 mL/day, and patients should take the drug with food.

9. A patient who is diagnosed with peptic ulcer disease has been started on a regimen that includes ranitidine (Zantac) 300 mg daily at bedtime. The patient calls the clinic 1 week later to report no relief from discomfort. What action will the nurse take? a. Contact the provider to discuss changing to cimetidine (Tagamet). b. Notify the provider to discuss increasing the dose. c. Reassure the patient that the drug may take 1 to 2 weeks to be effective. d. Suggest that the patient split the medication into twice daily dosing.

ANS: C Patients taking histamine2 blockers can expect abdominal pain to decrease after 1 to 2 weeks of drug therapy. Cimetidine is not as potent as ranitidine and interacts with many medications through the cytochrome P450 system. Three hundred milligrams is the maximum dose.

9. The nurse is teaching a patient who will be discharged home from the hospital to take amoxicillin (Amoxil) twice daily for 10 days. Which statement by the nurse is correct? a. "Discontinue the antibiotic when your temperature returns to normal and your symptoms have improved." b. "If diarrhea occurs, stop taking the drug immediately and contact your provider." c. "Stop taking the drug and notify your provider if you develop a rash while taking this drug." d. "You may save any unused antibiotic to use if your symptoms recur."

ANS: C Patients who develop signs of allergy, such as rash, should notify their provider before continuing medication therapy. Patients should be counseled to continue taking their antibiotics until completion of the prescribed regimen even when they feel well. Diarrhea is an adverse effect but does not warrant cessation of the drug. Before deciding to stop taking a medication due to a side effect, encourage the patient to contact the provider first. Patients should discard any unused antibiotic.

17. The nurse is caring for a patient in the post-anesthesia care unit who has received a spinal anesthetic. Which action will the nurse perform? a. Ambulate the patient as soon as consciousness returns. b. Elevate the head of the bed to a semi-Fowler's position. c. Have the patient lay flat for 6 to 8 hours after the surgery. d. Turn the patient from side to side every 15 minutes.

ANS: C Patients who have had spinal anesthesia should remain flat for 6 to 8 hours to decrease the likelihood of losing spinal fluid, causing a headache.

5. The nurse is preparing to administer amoxicillin (Amoxil) to a patient and learns that the patient previously experienced a rash when taking penicillin. Which action will the nurse take? a. Administer the amoxicillin and have epinephrine available. b. Ask the provider to order an antihistamine. c. Contact the provider to discuss using a different antibiotic. d. Request an order for a beta-lactamase resistant drug.

ANS: C Patients who have previously experienced manifestations of allergy to a penicillin should not use penicillins again unless necessary. The nurse should contact the provider to discuss using another antibiotic from a different class. Epinephrine and antihistamines are useful when patients are experiencing allergic reactions, depending on severity.

19. A 45-year-old patient who is overweight has had a diagnosis of type 2 diabetes for 2 years. The patient uses 20 units of insulin per day. The patient's fasting blood glucose (FBG) is 190 mg/dL. The patient asks the nurse about using an oral antidiabetic agent. The nurse understands that oral antidiabetic agents a. cannot be used if the patient is overweight. b. cannot be used once a patient requires insulin. c. may be used since this patient meets criteria. d. may not be used since this patient's fasting blood glucose is too high.

ANS: C Patients who require less than 40 units of insulin per day and who have a fasting blood glucose less than or equal to 200 mg/dL are candidates for oral antidiabetic agents. Being overweight is an indication, not a contraindication.

15. The nurse is assessing a patient prior to administering thrombolytic therapy. Which is an important assessment for this patient? a. Determining whether the patient has a history of diabetes b. Finding out about a history of renal disease c. Assessing which medications are taken for discomfort d. Assessing whether the patient eats green, leafy vegetables

ANS: C Patients who take aspirin or nonsteroidal antiinflammatory drugs (NSAIDs) should be monitored closely for excessive bleeding when given thrombolytics. There are no contraindications or precautions for patients with diabetes or renal disease. Foods rich in vitamin K are of concern for patients taking warfarin.

17. A patient who has AIDS is at risk to contract aspergillosis. The nurse will anticipate that which antifungal medication will be ordered prophylactically for this patient? a. Metronidazole (Flagyl) b. Micafungin (Mycamine) c. Posaconazole (Noxafil) d. Voriconazole (Vfend)

ANS: C Posaconazole is given for prophylactic treatment of Aspergillus and Candida infections.

8. A woman who is 2 months pregnant reports having morning sickness every day and asks if she can take any medications to treat this problem. The nurse will recommend that the patient take which action first? a. Contact the provider to discuss a possible need for intravenous fluids. b. Contact the provider to discuss a prescription antiemetic. c. Use nonpharmacologic measures such as saltines. d. Take over-the-counter antiemetics such as diphenhydramine.

ANS: C Pregnant women should avoid antiemetics during the first trimester of pregnancy because of possible teratogenic effects. The nurse should recommend nonpharmacologic measures such as saltines. If this is not effective, intravenous fluids may become necessary. Pregnant women should consult with their provider before taking prescription or over-the-counter antiemetics.

18. A patient with Graves disease exhibits tachycardia, heat intolerance, and exophthalmos. Prior to surgery, which drug is used to alter thyroid hormone levels? a. Liotrix (Thyrolar) b. Propranolol (Inderal) c. Propylthiouracil (PTU) d. Thyroid (Thyrotab)

ANS: C Propylthiouracil is a potent antithyroid drug used in preparation for a subtotal thyroidectomy. Liotrix and thyroid are used as thyroid replacement. Propranolol is used to treat hypertension associated with hyperthyroidism.

1. A patient calls the clinic in November to report a temperature of 103° F, headache, a nonproductive cough, and muscle aches. The patient reports feeling well earlier that day. The nurse will schedule the patient to see the provider and will expect the provider to order which medication? a. Amantadine HCl (Symmetrel) b. Influenza vaccine c. Rimantadine HCl (Flumadine) d. An over-the-counter drug for symptomatic treatment

ANS: C Rimantadine is used for treatment of influenza. Amantadine is used primarily for prophylaxis, and this patient already has symptoms. The influenza vaccine may be given later to protect against other strains. Over-the-counter medications may be used as adjunct treatment.

3. The nurse is teaching a nursing student about the minimal effective concentration (MEC) of antibiotics. Which statement by the nursing student indicates understanding of this concept? a. "A serum drug level greater than the MEC ensures that the drug is bacteriostatic." b. "A serum drug level greater than the MEC broadens the spectrum of the drug." c. "A serum drug level greater than the MEC helps eradicate bacterial infections." d. "A serum drug level greater than the MEC increases the therapeutic index."

ANS: C The MEC is the minimum amount of drug needed to halt the growth of a microorganism. A level greater than the MEC helps eradicate infections. Drugs at or above the MEC are usually bactericidal, not bacteriostatic. Raising the drug level does not usually broaden the spectrum or increase the therapeutic index of a drug.

5. The nurse is caring for a patient who is taking digoxin to treat heart failure. The patient's electrocardiogram shows a ventricular dysrhythmia. The nurse will notify the provider and will anticipate an order for which medication? a. Digoxin immune Fab (Digibind) b. Furosemide (Lasix) c. Phenytoin (Dilantin) d. Potassium

ANS: C The antidysrhythmics phenytoin and lidocaine are effective in treating digoxin-induced ventricular dysrhythmias. Digoxin immune Fab is used to treat severe digitalis toxicity, characterized by bradycardia, nausea, and vomiting. Unless a potassium deficit is present, giving potassium could worsen the dysrhythmia.

13. The nurse is performing a medication history on a patient who reports long-term use of montelukast (Singulair) and an albuterol metered-dose inhaler (Proventil). The nurse will contact the provider to discuss an order for which laboratory tests? a. Cardiac enzymes and serum calcium b. Electrolytes and a complete blood count c. Liver function tests and serum glucose d. Urinalysis and serum magnesium

ANS: C The beta2 agonists can increase serum glucose levels and montelukast can elevate liver enzymes, so these should be monitored in patients taking these medications.

16. The nurse is performing a health history on a patient who has multiple sclerosis. The patient reports episodes of muscle spasticity and recurrence of muscle weakness and diplopia. The nurse will expect this patient to be taking which medication? a. Adrenocorticotropic hormone (ACTH) b. Cyclophosphamide (Cytoxan) c. Cyclobenzaprine (Flexeril) d. Interferon-B (IFN-B)

ANS: D This patient is showing signs of remission and exacerbation of MS symptoms. Interferon is used to treat this phase. ACTH is used for acute attacks. Cyclophosphamide is used for chronic, progressive symptoms. Cyclobenzaprine is a centrally acting muscle relaxant that is used for muscle spasms to decrease pain and increase range of motion.

6. The nurse is caring for a patient who has hypothyroidism. To assist in differentiating between primary and secondary hypothyroidism, the nurse will expect the provider to order which drug? a. Liothyronine sodium (Cytomel) b. Liotrix (Thyrolar) c. Methimazole (Tapazole) d. Thyrotropin (Thytropar)

ANS: D Thyrotropin is a purified extract of thyroid-stimulating hormone and is used as a diagnostic agent to differentiate between primary and secondary hypothyroidism. Liothyronine and liotrix are thyroid replacement drugs. Methimazole is used to decrease thyroid hormone secretion.

7. The nurse is preparing to administer the first dose of an antibiotic to a patient admitted for a urinary tract infection. Which action is most important prior to administering the antibiotic? a. Administering a small test dose to determine whether hypersensitivity exists b. Having epinephrine available in the event of a severe hypersensitivity reaction c. Monitoring baseline vital signs, including temperature and blood pressure d. Obtaining a specimen for culture and sensitivity

ANS: D To obtain the most accurate culture, the specimen should be obtained before antibiotic therapy begins. It is important to obtain cultures when possible in order to correctly identify the organism and help determine which antibiotic will be most effective. Administering test doses to determine hypersensitivity is sometimes done when there is a strong suspicion of allergy when a particular antibiotic is needed. Epinephrine is kept close at hand when there is a strong suspicion of allergy.

12. A child is being treated for pinworms, and the parent asks the nurse how to prevent spreading this to other family members. What will the nurse tell the parent? a. "Give your child baths every day." b. "Obtain a daily stool specimen from your child." c. "Wash your child's clothing in hot water." d. "Your child should wash hands well after using the toilet."

ANS: D To prevent the spread of pinworms, good hand washing after toileting is recommended. Patients should take showers, not baths. It is not necessary to get regular stool specimens or to wash clothing in hot water.

16. A patient who has been taking nitroglycerin for angina has developed variant angina, and the provider has added verapamil (Calan) to the patient's regimen. The nurse will explain that verapamil is given for which purpose? a. To facilitate oxygen use by the heart b. To improve renal perfusion c. To increase cardiac contractility d. To relax coronary arteries

ANS: D Verapamil is a calcium channel blocker and is used to relax coronary artery spasm in patients with variant angina. It does not facilitate coronary muscle oxygen use, improve renal function, or increase cardiac contractility.

15. The patient has been started on a treatment regimen that includes atenolol (Tenormin) and complains to the nurse of feeling weak. Which is the best response from the nurse? a. "I will hold your next dose of the medication." b. "You may need an increase in your next dose of the medication." c. "This is an adverse reaction to the medication. I will stop the drug." d. "This is a side effect of the medication. I will notify your physician."

ANS: D Weakness can be a side effect of atenolol. Beta blockers should not be stopped abruptly.

1. A patient is diagnosed with borderline hypertension and states a desire to make lifestyle changes to avoid needing to take medication. The nurse will recommend which changes? a. Changing from weight bearing exercise to yoga b. Decreased fluid intake and increased potassium intake c. Stress reduction and increased protein intake d. Weight reduction and decreased sodium intake

ANS: D Weight loss decreases the stress on the heart and the afterload. Decreasing salt intake decreases the amount of retained fluid. Changing to yoga from weight-bearing exercise, limiting fluids, and increasing potassium are not indicated. Stress reduction is recommended, but increasing protein is not.

2. A patient who has symptoms of peptic ulcer disease will undergo a test that requires drinking a liquid containing 13C urea and breathing into a container. The nurse will explain to the patient that this test is performed to a. assess the level of hydrochloric acid. b. detect H. pylori antibodies. c. measure the pH of gastric secretions. d. test for the presence of 13CO2.

ANS: D When H. pylori is suspected, a noninvasive test is performed by administering 13C urea which, in the presence of H. pylori, will release 13CO2. The test does not measure the amount of HCl acid or the pH and does not detect H. pylori antibodies.

8. The charge nurse observes a nurse administer undiluted intravenous pyridostigmine bromide (Mestinon) at a rate of 0.8 mg/min. The charge nurse will stop the infusion and perform which action? a. Administer atropine sulfate to prevent cholinergic crisis. b. Monitor the patient closely for respiratory distress. c. Suggest that the nurse dilute the medication with colloidal fluids. d. Tell the nurse to slow the rate of infusion of the pyridostigmine.

ANS: D When given, IV pyridostigmine should be administered undiluted at a rate of 0.5 mg/min and should not be added to IV fluids. It is not necessary to administer atropine, since the patient is not symptomatic of cholinergic crisis.

11. A patient who uses an inhaled glucocorticoid medication reports having a sore tongue. The nurse notes white spots on the patient's tongue and oral mucous membranes. After notifying the provider, the nurse will remind the patient to perform which action? a. Avoid using a spacer with the inhaled glucocorticoid medication. b. Clean the inhaler with hot, soapy water after each use. c. Consume yogurt daily while using this medication. d. Rinse the mouth thoroughly with water after each use.

ANS: D When using inhaled glucocorticoid medications, Candida albicans oropharyngeal infections may be prevented by rinsing the mouth and throat with water after each dose. Patients should also use a spacer to reduce deposits of the drug in the oral cavity. The inhaler should be washed with warm water daily, but not after each use. There is no indication that yogurt is effective.

7. The nurse is teaching a patient who will begin taking furosemide. The nurse learns that the patient has just begun a 2-week course of a steroid medication. What will the nurse recommend? a. Consume licorice to prevent excess potassium loss. b. Report a urine output greater than 600 mL/24 hours. c. Obtain an order for a potassium supplement. d. Take the furosemide at bedtime.

ANS: C The interaction of furosemide and a steroid drug can result in an increased loss of potassium. Patients should take a potassium supplement. Patients should avoid licorice while taking furosemide, partially due to the hypokalemic effects of both substances. Urine output greater than 600 mL/24 hours is normal. Patients should take furosemide in the morning to avoid nocturia.

9. The nurse is counseling an adolescent patient who has recurrent otitis externa and who works as a lifeguard in the summer about preventing this condition. The nurse will teach this patient to a. avoid using ear plugs while swimming. b. request a prescription for prophylactic antibiotic eardrops. c. use a hair dryer to dry the ears after swimming. d. wear a medical alert bracelet.

ANS: C To help prevent otitis externa, patients should be counseled to use a portable hair dryer to dry the ears after swimming. Ear plugs are recommended. Prophylactic antibiotic eardrops are not indicated. A medical alert bracelet is not necessary.

8. The nurse is caring for a patient who is receiving sulfadiazine. The nurse knows that this patient's daily fluid intake should be at least which amount? a. 1000 mL/day b. 1200 mL/day c. 2000 mL/day d. 2400 mL/day

ANS: C To prevent crystalluria, patients should consume at least 2000 mL/day.

10. The nurse is caring for an African-American patient who has been diagnosed with glaucoma. The nurse anticipates that which medication will be most effective for this client? a. Bimatoprost (Lumigan) b. Latanoprost (Xalatan) c. Travoprost (Travatan Z) d. Unoprostone (Rescula)

ANS: C Travoprost has been found to be more effective in African Americans than in non-African Americans.

2. A patient develops type 2 diabetes mellitus. The nurse will explain that this type of diabetes a. is generally triggered by medications. b. is not as common as type 1 diabetes. c. is often related to heredity and obesity. d. will not require insulin therapy.

ANS: C Type 2 diabetes is often caused by obesity and hereditary factors. Secondary diabetes is triggered by medications. Type 2 diabetes is the most common type of diabetes. Patients with type 2 diabetes may become insulin-dependent.

16. The nurse is caring for a patient in the post-anesthesia care unit and notes that the patient received isoflurane (Forane) to induce anesthesia. When will the nurse expect the patient to recover consciousness? a. Immediately b. In 15 to 30 minutes c. In 1 hour d. In hours

ANS: C Upon discontinuation of isoflurane, recovery of consciousness usually occurs in 1 hour.

6. The nurse is caring for a patient who is receiving trihexyphenidyl (Artane) to treat parkinsonism. The patient reports having a dry mouth, and the nurse notes a urine output of 300 mL in the past 8 hours. Which action will the nurse perform? a. Encourage increased oral fluids. b. Obtain an order for intravenous fluids. c. Report the urine output to the provider. d. Request an order for renal function tests.

ANS: C Urinary retention can occur with anticholinergic medications. Dry mouth is a harmless side effect. The nurse should report the lower than expected urine output to the provider. Increasing fluid intake will not increase urine output in the patient with urinary retention. Renal function tests are not indicated since this is a neuromuscular problem of the bladder caused by the medication.

7. A patient is admitted to the hospital after developing pneumonia. During the admission assessment, the patient reports having used a nasal decongestant spray for the past few weeks but thinks the nasal congestion is getting worse. The nurse will a. request an order for a systemic decongestant medication. b. request an order so the patient can continue to use the decongestant spray. c. tell the patient the congestion will clear up after stopping the spray. d. tell the patient to increase oral fluid intake.

ANS: C Use of nasal decongestants longer than 3 days can cause rebound congestion. This will clear up when the decongestant spray is discontinued for several days or weeks. A systemic decongestant is not indicated. Continuing the spray will increase the congestion. Increasing fluid intake is not recommended.

15. The nurse is performing a health history on a patient who is ordered to begin therapy with valproic acid (Depakote) to treat epilepsy. Which aspect of the patient's medical history will cause the nurse to be concerned? a. Chronic obstructive pulmonary disease b. Gastrointestinal disease c. Liver disease d. Renal disease

ANS: C Valproic acid can elevate liver enzymes. Patients with a history of liver disease should be monitored closely while taking this drug.

6. A patient who is taking warfarin has an international normalized ratio (INR) of 5.5. The nurse will anticipate giving a. fresh frozen plasma. b. intravenous iron. c. oral vitamin K. d. protamine sulfate.

ANS: C Vitamin K is an antagonist against warfarin, an oral anticoagulant. Patients with an INR of 5.5 should be given a low dose of oral vitamin K. Too much vitamin K may reduce the effectiveness of warfarin for up to 2 weeks. Fresh frozen plasma and intravenous iron are given for anemia caused by blood loss. Protamine sulfate is given for heparin overdose.

7. A female patient who is allergic to penicillin will begin taking an antibiotic to treat a lower respiratory tract infection. The patient tells the nurse that she almost always develops a vaginal yeast infection when she takes antibiotics and that she will take fluconazole (Diflucan) with the antibiotic being prescribed. Which macrolide order would the nurse question for this patient? a. Azithromycin (Zithromax) b. Clarithromycin (Biaxin) c. Erythromycin (E-Mycin) d. Fidaxomicin (Dificid)

ANS: C When erythromycin is given concurrently with fluconazole, erythromycin blood concentration and the risk of sudden cardiac death increase.

2. A patient is diagnosed with heart failure, and the prescriber has ordered digoxin. The patient asks what lifestyle changes will help in the management of this condition. The nurse will recommend which changes? a. Aerobic exercise and weight lifting 2 or 3 times weekly b. Changing from cigarette smoking to pipe smoking c. Consuming 2 teaspoons or less of salt every day d. Having no more than one alcoholic beverage per day

ANS: D Alcohol should either be completely avoided or restricted to no more than one per day. Mild exercise, such as walking, is recommended. All nicotine deprives the heart of oxygen. Salt should be limited to no more than one teaspoon per day.

11. A patient who has Raynaud's disease will begin taking an alpha-adrenergic blocker. The patient asks the nurse how the drug works to treat symptoms. The nurse explains that alpha-adrenergic blockers treat Raynaud's disease by causing a. decreased peripheral vascular resistance. b. orthostatic hypotension. c. reflex tachycardia. d. vasodilation.

ANS: D Alpha-adrenergic blockers can be used to treat peripheral vascular disease because they cause vasodilation.

14. During balanced anesthesia, which type of medication is given while the surgery is performed? a. Anticholinergics b. Benzodiazepines c. Hypnotics d. Inhaled anesthetic

ANS: D An inhaled anesthetic is given to induce anesthesia and is maintained throughout the surgical procedure. The other medications are given prior to anesthesia induction.

10. A woman who has advanced breast cancer will begin receiving androgen therapy. The nurse will explain to the patient that androgen therapy is used to a. enhance her own estrogen production. b. give her a sense of well-being. c. minimize hot flashes. d. promote regression of her tumor.

ANS: D Androgen is used to treat breast cancer to promote regression of tumors. Other hormonal therapies are used in other circumstances to promote well-being.

10. The nurse is teaching a patient about the use of an anticholinergic medication. What information will the nurse include when teaching this patient about this medication? a. "Check your heart rate frequently to monitor for bradycardia." b. "Drink extra fluids while you are taking this medication." c. "Rise from a chair slowly to avoid dizziness when taking this drug." d. "Use gum or lozenges to decrease dry mouth caused by this drug."

ANS: D Anticholinergic medications cause dry mouth, so patients should be advised to use gum or lozenges to counter this side effect. Anticholinergics cause increased heart rate and increased blood pressure. Anticholinergics can cause urinary retention so patients should not increase fluid intake.

16. A woman who is pregnant is taking an anticonvulsant medication to treat a seizure disorder. The nurse will ensure that the patient takes which dietary supplement toward the end of her pregnancy? a. Folate (folic acid) b. Iron c. Vitamin C d. Vitamin K

ANS: D Anticonvulsants act as inhibitors of vitamin K and can contribute to hemorrhage in infants shortly after birth. Women taking these drugs should receive vitamin K within the last week to 10 days of their pregnancies.

7. The nurse is preparing to administer the anticholinergic medication benztropine (Cogentin) to a patient who has Parkinson's disease. The nurse understands that this drug is used primarily for which purpose? a. To decrease drooling and excessive salivation b. To improve mobility and muscle strength c. To prevent urinary retention d. To suppress tremors and muscle rigidity

ANS: D Antiparkinson-anticholinergic drugs are used mainly to reduce tremors and muscle rigidity.

4. The patient asks the nurse what apoptosis means. The nurse will explain that apoptosis refers to a. alteration of cellular functions. b. inhibition of cell division. c. prevention of cell phase progression. d. slow, planned cellular death.

ANS: D Apoptosis is programmed cell death, designed to ensure that tissues contain only healthy and optimally functional cells.

3. The parents of a 16-year-old boy who plays football want their child to receive growth hormone to improve muscle strength. What will the nurse tell the parents? a. "Growth hormone may be used to improve strength in young athletes." b. "If the epiphyses are not fused, growth hormone may be an option." c. "Small doses of growth hormone may be used indefinitely for this purpose." d. "Using growth hormone to build muscle mass is not recommended."

ANS: D Athletes should be advised not to take growth hormone to build muscle because of its effects on blood sugar and other side effects.

17. A patient who is taking clopidogrel (Plavix) and aspirin is preparing for orthopedic surgery. The nurse will consult with the surgeon and provide which instruction to the patient? a. Continue taking aspirin and stop taking clopidogrel 2 weeks prior to surgery. b. Continue taking clopidogrel and stop taking aspirin 5 days prior to surgery. c. Continue both medications to prevent thromboembolic events during surgery. d. Stop taking both medications 7 days prior to surgery.

ANS: D Because both drugs can prolong bleeding time, patients should discontinue the drugs 7 days prior to surgery.

12. The nurse is teaching a patient about sublingual nitroglycerin administration. What information will the nurse include when teaching this patient? a. Call 911 if pain does not improve after three doses. b. If pain persists after one dose, administer a second dose. c. Swallow the tablet with small sips of water. d. Take the first tablet while sitting or lying down.

ANS: D Because nitroglycerin can cause hypotension, patients should be cautioned to take them while sitting or lying down. If pain is not better or has worsened 5 minutes after the first dose, patients should call 911. The tablets must dissolve under the tongue and should not be swallowed.

12. The nurse is preparing to administer interleukin-2 to a patient who has cancer. The patient reports shortness of breath. The nurse assesses clear breath sounds, a respiratory rate of 22 breaths per minute, a heart rate of 80 beats per minute, an oxygen saturation of 93% on room air, and a blood pressure of 92/68 mm Hg. The nurse will perform which action? a. Administer the dose as ordered. b. Administer oxygen while giving the dose. c. Discuss permanently discontinuing this treatment with the provider. d. Hold the dose and notify the provider.

ANS: D Because of pulmonary symptoms associated with interleukin-2, the drug should be held if the patient has an oxygen saturation <94% on room air. It may be given when the patient's oxygen saturation improves. The drug does not need to be permanently discontinued.

12. A patient who has been taking a benzodiazepine as a sleep aid for several months wishes to stop taking the medication. The nurse will suggest that the patient taper the dose gradually to avoid which effect? a. Depression b. Hangover c. Hypnotic rebound d. Withdrawal

ANS: D Benzodiazepines cause tolerance which means that abrupt cessation can result in withdrawal symptoms such as tremors and muscle twitching. A hangover is residual drowsiness that occurs the day after taking a hypnotic.

4. A patient who has Parkinson's disease is being treated with the anticholinergic medication benztropine (Cogentin). The nurse will tell the patient that this drug will have which effect? a. Helping the patient to walk faster b. Improving mental function c. Minimizing symptoms of bradykinesia d. Reducing some of the tremors

ANS: D Benztropine is given to reduce rigidity and some of the tremors. It does not enhance walking or reduce bradykinesia or improve mental function.

7. A patient is receiving bleomycin (Blenoxane) as part of a chemotherapeutic regimen to treat leukemia. During intravenous administration of this drug, what will the nurse observe the patient closely for? a. Hypotension and visual disturbances b. Pain and blistering at the IV site c. Pink to red urine d. Shortness of breath and wheezing

ANS: D Bleomycin can cause anaphylaxis, so patients should be monitored for respiratory distress. Pain and blistering at the IV site is common to antitumor antibiotics except for bleomycin. Urine color changes occur with doxorubicin. Vincristine causes hypotension and visual disturbances.

8. The nurse is caring for a patient who has metabolic alkalosis and is experiencing fluid overload. The provider orders acetazolamide (Diamox). The patient reports right-sided flank pain after taking this medication. The nurse suspects that this patient has developed which condition? a. Gout b. Hemolytic anemia c. Metabolic acidosis d. Renal calculi

ANS: D Carbonic anhydrase inhibitors, such as acetazolamide, are used to treat patients who are in metabolic alkalosis and need a diuretic. They can cause electrolyte imbalance, metabolic acidosis, hemolytic anemia, and renal calculi. This patient has right-sided flank pain, which occurs with renal calculi.

10. The nurse is preparing to administer piperacillin to a patient to treat an infection caused by pseudomonas. The nurse learns that the patient receives corticotropin to treat multiple sclerosis. The nurse will request an order for a. a different antibiotic. b. blood glucose monitoring. c. cardiac monitoring. d. serum electrolytes.

ANS: D Corticotropin can interact with piperacillin to cause hypokalemia, so serum electrolytes should be monitored. It is not necessary to change the antibiotic. Blood glucose monitoring and cardiac monitoring are not indicated.

11. The nurse is caring for a patient who is receiving desmopressin acetate (DDAVP). Which assessments are important while caring for this patient? a. Blood pressure and serum potassium b. Heart rate and serum calcium c. Lung sounds and serum magnesium d. Urine output and serum sodium

ANS: D Desmopressin is an antidiuretic hormone. The nurse should monitor intake and output as well as serum sodium levels.

10. A patient asks the nurse about using dextromethorphan for cough. What information will the nurse include when teaching this patient about this drug? a. "It does not cause sedation except at high doses." b. "It may be used to treat cough for up to 2 weeks." c. "It is non-narcotic, and it is OK to consume alcohol while taking this drug." d. "It should not be taken by patients who have chronic obstructive pulmonary disease (COPD)."

ANS: D Dextromethorphan is contraindicated in patients with COPD. It may cause sedation at low doses. If a cough lasts longer than 1 week, patients should be instructed to contact their provider and not to continue to treat with over-the-counter antitussives. Alcohol can cause excess sedation when taken with dextromethorphan.

11. The nurse is teaching a patient who is receiving vincristine (Oncovin) about long-term management of the treatment regimen. Which information will the nurse provide in teaching the patient? a. "If you experience numbness of your hands, it will eventually resolve." b. "If your IV starts to hurt, you should pull the IV out immediately." c. "You should ask for antinausea medication at the first sign of nausea." d. "You should report difficulty buttoning your clothes to your provider."

ANS: D Difficulty buttoning clothing is a sign of peripheral neuropathy and should be reported. Numbness of hands may resolve after chemotherapy is stopped, but it may never resolve. If the IV infiltrates, the infusion should be stopped and the needle left in until attempts to aspirate residual vesicant are performed. Antinausea medication should be given prior to beginning the infusion.

1. A patient who has atrial fibrillation is taking digoxin. The nurse expects which medication to be given concurrently to treat this condition? a. Hydrochlorothiazide (HydroDIURIL) b. Inamrinone (Inocor) c. Milrinone (Primacore) d. Warfarin (Coumadin)

ANS: D Digoxin is given for atrial fibrillation to restore a normal heart rhythm. To prevent thromboemboli, warfarin is given concurrently. Hydrochlorothiazide is a diuretic medication. Inamrinone and milrinone are inotropic agents that would be used instead of digoxin.

15. The nurse is caring for an older adult who is receiving diphenoxylate with atropine (Lomotil) to treat severe diarrhea. The nurse will monitor this patient closely for which effect? a. Bradycardia b. Fluid retention c. Nervousness and tremors d. Respiratory depression

ANS: D Diphenoxylate is an opium agonist and can cause respiratory depression. Children and older adults are more susceptible to this effect. It contains atropine, so it will increase heart rate. It does not contribute to fluid retention. Lomotil causes central nervous system depression and will not cause nervousness and tremors

3. The nurse is providing teaching to the parents of a 5-year-old child who will begin taking phenytoin (Dilantin). What information will the nurse include when teaching these parents about their child's medication? a. "Drug interactions are uncommon with phenytoin." b. "There are very few side effects associated with this drug." c. "The therapeutic range of phenytoin is between 15 and 30 mcg/mL." d. "Your child may need a higher dose than expected."

ANS: D Drug dosage for phenytoin is age-related and children, who have a rapid metabolism, may need higher doses than those used for newborns and adults. Phenytoin has many drug interactions and many side effects. The therapeutic range is 10-20 mcg/mL.

5. A patient has an infection of the eyelash follicles and in the gland on the eyelid margin. The nurse recognizes these symptoms as being consistent with which condition? a. Blepharitis b. Chalazion c. Endophthalmitis d. Hordoleum

ANS: D Hordoleum is a local infection of eyelash follicles and glands on the eyelid margin. Blepharitis is an infection of the margins of the eyelid. Chalazion is an infection of the glands of the eyelids that may produce cysts. Endophthalmitis is an infection of the structures of the inner eye.

7. The nurse is preparing to administer intravenous temsirolimus (Torisel). To prevent a common adverse drug effect, the nurse will expect to administer which type of drug? a. An antibiotic b. An anticoagulant c. An antiemetic d. An antihistamine

ANS: D Hypersensitivity reactions to temsirolimus are common, and pretreatment with antihistamines is recommended. Other drugs are given as needed but not prophylactically.

17. A patient who has begun taking nifedipine (Procardia) to treat variant angina has had a recurrent blood pressure of 90/60 mm Hg or less. The nurse will anticipate that the provider will a. add digoxin to the drug regimen. b. change to a beta blocker. c. order serum liver enzymes. d. switch to diltiazem (Cardizem).

ANS: D Hypotension is a common effect of calcium channel blockers and is more common with nifedipine. It is less common with diltiazem, so the provider may order that drug. Adding digoxin, changing to a beta blocker, or ordering serum liver enzymes are not indicated.

7. The nurse performs a medication history and learns that the patient takes a thiazide diuretic and digoxin (Lanoxin). The nurse will question the patient to ensure that the patient is also taking which medication? a. Cortisone b. Lidocaine c. Nitroglycerin d. Potassium

ANS: D If a patient is taking digoxin and a potassium-wasting diuretic such as thiazide, the patient should also take a potassium supplement to prevent hypokalemia that could result in digoxin toxicity. It is not necessary to take cortisone, lidocaine, or nitroglycerin unless the patient has symptoms that warrant these drugs.

7. A patient exhibits ptosis of both eyes, and the provider orders edrophonium (Tensilon). The nurse notes immediate improvement of the ptosis. The nurse understands that this patient most likely has which disorder? a. Cerebral palsy b. Multiple sclerosis c. Muscle spasms d. Myasthenia gravis

ANS: D Improvement of symptoms after administration of edrophonium is diagnostic for myasthenia gravis.

6. A patient experiences severe muscle weakness, and the provider orders edrophonium bromide (Tensilon). The patient begins to show improved muscle strength within a few minutes after administration of this drug. The nurse anticipates the provider will order which drug? a. Atropine sulfate b. Edrophonium bromide (Tensilon) c. Intravenous immune globulin (IVIG) d. Pyridostigmine HCl (Mestinon)

ANS: D In this case, edrophonium is used to diagnose myasthenia gravis. Since symptoms improved with the AChE inhibitor, the patient will benefit from a longer-acting AChE inhibitor such as pyridostigmine. Atropine is given for AChE inhibitor overdose. Edrophonium is very short-acting, so it will not be used for treatment. IVIG is used when other AChE inhibitors fail.

7. The nurse will administer parenteral insulin to a patient who will receive a mixture of NPH (Humulin NPH) and regular (Humulin R). The nurse will give this medication via which route? a. Intradermal b. Intramuscular c. Intravenous d. Subcutaneous

ANS: D Insulin is given by the subcutaneous route. Only regular insulin may be given IV.

16. The nurse is preparing to begin a medication regimen for a patient who will receive intravenous ampicillin and gentamicin. Which is an important nursing action? a. Administer each antibiotic to infuse over 15 to 20 minutes. b. Order serum peak and trough levels of ampicillin. c. Prepare the schedule so that the drugs are given at the same time. d. Set up separate tubing sets for each drug labeled with the drug name and date.

ANS: D Intravenous aminoglycosides can be given with penicillins and cephalosporins but should not be mixed in the same container. The IV line should be flushed between antibiotics, or separate tubing sets may be set up. Gentamicin must be infused over 30 to 60 minutes. It is not necessary to measure ampicillin peak and trough levels. Giving the drugs at the same time increases the risk of mixing them together.

1. The nurse is caring for a postoperative patient. The nurse will anticipate administering which medication to this patient to help prevent thrombus formation caused by slow venous blood flow? a. Alteplase (Activase) b. Aspirin c. Clopidogrel (Plavix) d. Low-molecular-weight heparin

ANS: D Low-molecular-weight heparin is an anticoagulant, which is used to inhibit clot formation and is used prophylactically to prevent postoperative deep vein thrombosis. Alteplase is a thrombolytic, which is used to break down clots after they form; alteplase is contraindicated in any patient with recent surgery. Aspirin and clopidogrel are antiplatelet drugs and are used to prevent arterial thrombosis.

7. An elderly patient reports using Maalox frequently to treat acid reflux. The nurse should notify the patient's provider to request an order for which laboratory tests? a. Liver enzymes and serum calcium b. Liver enzymes and serum magnesium c. Renal function tests and serum calcium d. Renal function tests and serum magnesium

ANS: D Maalox contains magnesium and carries a risk of hypermagnesemia, especially with decreased renal function. Older patients have an increased risk of poor renal function, so this patient should especially be evaluated for hypermagnesemia.

1. The nurse is caring for a patient who is receiving an intravenous antibiotic. The nurse notes that the provider has ordered serum drug peak and trough levels. The nurse understands that these tests are necessary for which type of drugs? a. Drugs with a broad spectrum b. Drugs with a narrow spectrum c. Drugs with a broad therapeutic index d. Drugs with a narrow therapeutic index

ANS: D Medications with a narrow therapeutic index have a limited range between the therapeutic dose and a toxic dose. It is important to monitor these medications closely by evaluating regular serum peak and trough levels.

4. The nurse is caring for a patient who is receiving intravenous mechlorethamine (Mustargen). The patient reports pain at the IV site, and the nurse assesses swelling and pallor at the site. What action will the nurse take? a. Administer an analgesic medication. b. Apply a warm compress to the site. c. Slow the infusion and notify the provider. d. Stop the infusion immediately.

ANS: D Methchlorethamine is a severe vesicant and can cause tissue necrosis if it infiltrates into the tissues. The nurse should stop the infusion.

10. The nurse provides teaching for patient who will begin taking montelukast sodium (Singulair). The patient reports sensitivity to aspirin. Which statement by the patient indicates a need for further teaching? a. "I will need to have periodic laboratory tests while taking this medication." b. "I will not take ibuprofen for pain or fever while taking this drug." c. "I will take one tablet daily at bedtime." d. "I will use this as needed for acute symptoms."

ANS: D Montelukast and other leukotriene receptor antagonists are not used to treat acute symptoms. Because they can affect liver enzymes, periodic liver function tests should be performed. Patients taking this drug should not use ibuprofen or aspirin for pain or fever if they have an aspirin sensitivity. Patients will achieve maximum effectiveness if the drug is taken in the evening.

13. The nurse is performing an admission assessment on a patient who has been taking carisoprodol (Soma) for 3 weeks to treat muscle spasms. The patient reports that the muscle spasms have resolved. The nurse will contact the provider to discuss a. changing to cyclobenzaprine (Flexeril). b. continuing the carisoprodol for 1 more week. c. discontinuing the carisoprodol now. d. ordering a taper of the carisoprodol.

ANS: D Muscle relaxants can cause drug dependence and should not be withdrawn abruptly. The nurse should discuss a drug taper.

1. The nurse is preparing to care for a patient who has myasthenia gravis. The nurse will be alert to symptoms affecting which body system in this patient? a. Cardiovascular system and postural muscles b. Central nervous system (CNS), memory, and cognition c. Gastrointestinal system (GI) and lower extremity muscles d. Respiratory system and facial muscles

ANS: D Myasthenia gravis causes fatigue and muscular weakness of the respiratory system, facial muscles, and extremities. It does not directly affect the cardiovascular system, CNS, or GI systems.

2. A 40-year-old woman is diagnosed with myasthenia gravis, and her provider recommends removal of her thymus gland. She asks the nurse why this would be helpful. The nurse will explain that removal of the thymus gland may a. increase binding of acetylcholine (ACh) molecules to ACh receptors. b. increase the amount of ACh available at neuromuscular junction sites. c. reduce the number of acetylcholine receptor sites. d. reduce the autoimmune destruction of ACh receptor sites.

ANS: D Myasthenia gravis is an autoimmune disorder involving an antibody response against a subunit of the ACh receptor site. Since the thymus is involved in systemic immunity, it is thought that removing the thymus can inhibit this process. It does not increase binding of ACh molecules to receptors or increase the amount of ACh or reduce the number of ACh receptor sites.

2. The nurse is preparing to administer isoproterenol (Isuprel) to a patient who is experiencing an acute bronchospasm. The nurse understands that, because isoproterenol is a nonselective beta-adrenergic agonist, the patient will experience which effects? a. Alpha- and beta-adrenergic agonist effects b. Anticholinergic effects c. A shorter duration of therapeutic effects d. Cardiac and pulmonary effects

ANS: D Non-selective beta-adrenergic agonists affect both beta1 and beta2 receptors, causing both tachycardia and bronchodilation. Alpha receptors are not affected.

5. An oncology home care nurse is preparing to administer a chemotherapeutic agent to a patient at the patient's home. What precautions will the nurse take while administering the IV chemotherapeutic agent? a. Clear a counter space for preparation of the solution. b. Don a surgical mask while administering the drug. c. Take surgical scrubs to wear during the infusion. d. Wear an impermeable, disposable gown when hanging the drug.

ANS: D Nurses should take precautions when handling cytotoxic drugs if inhalation, ingestion, or contact with skin and mucous membranes is possible. When hanging an IV solution, it is possible to splash solution onto the skin, so the nurse should wear a disposable, impermeable gown. If the nurse has to prepare a solution at home, a plastic-backed pad should be used as a surface. When there is a risk of aerosol exposure, a National Institute for Occupational Safety and Health-approved respirator is necessary. Surgical masks do not provide adequate respiratory protection. Surgical scrubs are permeable.

10. The parent of a junior high-school child who has type 1 diabetes asks the nurse if the child can participate in sports. The nurse will tell the parent a. that strenuous exercise is not recommended for children with diabetes. b. that the child must be monitored for hyperglycemia while exercising. c. to administer an extra dose of regular insulin prior to exercise. d. to send a snack with the child to eat just prior to exercise.

ANS: D Patients generally need less insulin with increased exercise, so the child should consume a snack to prevent hypoglycemia. Exercise is an integral part of diabetes management. Hypoglycemia is more likely to occur, and extra insulin is not indicated.

16. A patient who has hyperthyroidism will begin treatment with an antithyroid medication. The patient asks the nurse about dietary requirements. The nurse will counsel the patient to avoid which food(s)? a. Fava beans b. Foods high in purine c. Grapefruit d. Shellfish

ANS: D Patients should be advised about the effects of iodine and its presence in foods such as shellfish. There is no need to avoid fava beans, purine, or grapefruit.

11. A patient who has oral candidiasis will begin using nystatin suspension to treat the infection. What information will the nurse include when teaching this patient? a. "Coat the buccal mucosa with the drug and then rinse your mouth." b. "Gargle with the nystatin and then spit it out without swallowing." c. "Mix the suspension with 4 ounces of water and then drink it." d. "Swish the liquid in your mouth and then swallow after a few minutes."

ANS: D Patients should be taught to swish the suspension in the mouth to coat the tongue and buccal mucosa and then swallow the medication. It should not be spit out, diluted with water, or swallowed with water.

16. The nurse is teaching a patient who is receiving chloroquine (Aralen) for malaria prophylaxis. Which statement by the patient indicates a need for further teaching? a. "I may experience hair discoloration while taking this drug." b. "I should not take this drug with lemon juice." c. "I should use sunscreen while taking this drug." d. "If I have gastrointestinal upset, I should take an antacid."

ANS: D Patients should not take these drugs with antacids.

20. A patient who takes an oral sulfonylurea medication will begin taking fluconazole (Diflucan). The nurse will expect to monitor which lab values in this patient? a. Blood urea nitrogen (BUN) and creatinine b. Electrolytes c. Fluconazole levels d. Glucose

ANS: D Patients taking sulfonylurea drugs may have altered serum glucose when taking antifungal medications.

19. The nurse provides teaching for a patient who will begin taking phenytoin. Which statement by the patient indicates understanding of the teaching? a. "If I develop a rash, I should take diphenhydramine to control the itching." b. "If I experience bleeding gums, I should stop taking the medication immediately." c. "I may develop diabetes while I am taking this medication." d. "I should not be alarmed if my urine turns reddish-brown."

ANS: D Phenytoin will cause reddish-brown colored urine. Patients should be counseled to report a rash to the provider because it could be a serious adverse reaction. Bleeding gums are common, but patients should never stop taking anticonvulsants abruptly, or they may develop seizures. Changes in blood glucose may occur but do not necessarily result in diabetes.

2. A patient has a blood pressure of 135/85 mm Hg on three separate occasions. The nurse understands that this patient should be treated with a. a beta blocker. b. a diuretic and a beta blocker. c. a diuretic. d. lifestyle changes.

ANS: D Prehypertension is defined as a systolic pressure of 120 to 139 and a diastolic pressure between 80 and 89. Drug therapy is recommended if the blood pressure is greater than 20/10 over the goal, which would be140/90. Prehypertension is generally treated first with lifestyle changes.

5. The nurse is caring for a patient who has postoperative nausea and vomiting. The surgeon has ordered promethazine HCl (Phenergan). Which aspect of this patient's health history would be of concern? a. Asthma b. Diabetes c. GERD d. Glaucoma

ANS: D Promethazine is contraindicated in patients with glaucoma since it is an anticholinergic medication. It should be used with caution in patients with asthma. The other two conditions are not concerning with this medication.

13. The nurse is caring for a patient who will begin taking omeprazole (Prevacid) 20 mg per day for 4 to 8 weeks to treat gastroesophageal reflux disease esophagitis. The nurse learns that the patient takes digoxin. The nurse will contact the provider for orders to a. decrease the dose of omeprazole. b. increase the dose of digoxin. c. increase the omeprazole to 60 mg per day. d. monitor for digoxin toxicity.

ANS: D Proton pump inhibitors can enhance the effects of digoxin, so patients should be monitored for digoxin toxicity. Changing the dose of either medication is not indicated prior to obtaining lab results that are positive for digoxin toxicity.

11. One hour after receiving intravenous morphine sulfate, a patient reports generalized itching. The nurse assesses the patient and notes clear breath sounds, no rash, respirations of 14 breaths per minute, a heart rate of 68 beats per minute, and a blood pressure of 110/70 mm Hg. Which action will the nurse take? a. Administer naloxone to reverse opiate overdose. b. Have resuscitation equipment available at the bedside. c. Prepare an epinephrine injection in case of an anaphylactic reaction. d. Reassure the patient that this is a common side effect of this drug.

ANS: D Pruritis is a common opioid side effect and can be managed with diphenhydramine. Patients developing anaphylaxis will have urticaria and hypotension, and these patients will need epinephrine and resuscitation. Respiratory depression is a sign of morphine overdose, which will require naloxone.

7. An older adult has difficulty falling asleep. The nurse understands that which sedative hypnotic is appropriate for this patient? a. Butabarbital (Butisol) b. Flurazepam (Dalmane) c. Secobarbital (Seconal) d. Temazepam (Restoril)

ANS: D Short- to intermediate-acting benzodiazepines such as temazepam are recommended for older adults and are considered safer than barbiturates.

3. The nurse is preparing to administer bethanechol (Urecholine) to a patient who is experiencing urinary retention. The nurse notes that the patient has a blood pressure of 90/60 mm Hg and a heart rate of 98 beats per minute. The nurse will perform which action? a. Administer the drug and monitor urine output. b. Administer the medication and monitor vital signs frequently. c. Give the medication and notify the provider of the increased heart rate. d. Hold the medication and notify the provider of the decreased blood pressure.

ANS: D Side effects of this medication are a decrease in the pulse rate and vasodilation, which can exacerbate bradycardia and hypotension. The nurse should hold the drug and notify the provider.

13. The patient asks the nurse about storing insulin. Which response by the nurse is correct? a. "All insulin vials must be refrigerated." b. "Insulin will last longer if kept in the freezer." c. "Opened vials of insulin must be discarded." d. "Some combination pens do not require refrigeration."

ANS: D Some combination pens do not require refrigeration after first use. Storing insulin in the freezer is not recommended. Opened vials may either be kept at room temperature for a month or refrigerated for 3 months.

14. The nurse is caring for a patient who experiences a rapid rise in blood pressure. The nurse will contact the provider to discuss administering which medication? a. Amlodipine (Norvasc) b. Nifedipine (Procardia) c. Nifedipine extended release (Procardia XL) d. Verapamil (Calan)

ANS: B The short-acting nifedipine is used to treat rapid rises in blood pressure but cannot be used for out-patient treatment at high dosages because of an increased risk for sudden cardiac death. The other drugs are not used for rapid rise in BP

10.The client is being treated with etretinate (Tegison, Soriatane). She has been using the medication for 6 weeks and is concerned because she has not seen an improvement in her symptoms. The most accurate response from the nurse is that it may be _____ months for the client to notice an improvement in her symptoms. a. 2 b. 8 c. 4 d. 6

ANS: D Treatment with etretinate may take up to 6 months to produce a change in the client's symptoms.

11. The nurse is administering two drugs to a patient and learns that both drugs are highly protein-bound. The nurse may expect a. decreased bioavailability of both drugs. b. decreased drug effects. c. decreased drug interactions. d. increased risk of adverse effects.

ANS: D Two drugs that are highly protein-bound will compete for protein-binding sites, leaving more free drug in circulation and an increased risk of adverse effects as well as increased bioavailability, increased drug effects, and increased drug interactions.

12. The nurse administers nalbuphine (Nubain) to a patient who is experiencing severe pain. Which statement by the patient indicates a need for further teaching about this drug? a. "I may experience unusual dreams while taking this medication." b. "I may need to use a laxative when taking this drug." c. "I should ask for assistance when I get out of bed." d. "I should expect to have more frequent urination."

ANS: D A common side effect of opioid agents is urinary retention. Patients should notify the nurse if they cannot void. Side effects may include unusual dreams, constipation, and dizziness.

11. The nurse is teaching a patient about the use of a transdermal nitroglycerin patch. Which statement by the patient indicates understanding of the teaching? a. "I will apply the patch as needed when I experience anginal pain." b. "I will remove the old patch and replace it with a new one at bedtime each day." c. "I should rotate sites when changing the patch to prevent skin irritation." d. "When I am symptom-free, I may stop using the patch on a regular basis."

11. The nurse is teaching a patient about the use of a transdermal nitroglycerin patch. Which statement by the patient indicates understanding of the teaching? a. "I will apply the patch as needed when I experience anginal pain." b. "I will remove the old patch and replace it with a new one at bedtime each day." c. "I should rotate sites when changing the patch to prevent skin irritation." d. "When I am symptom-free, I may stop using the patch on a regular basis."

12. A 35-year-old woman reports lethargy, difficulty remembering things, facial edema, dry skin, and cessation of menses. The nurse notes a heart rate of 60 beats per minute and a weight increase of 5 pounds from a previous visit. The nurse will notify the provider of which possible condition? a. Cretinism b. Early menopause c. Hyperthyroidism d. Myxedema

12. A 35-year-old woman reports lethargy, difficulty remembering things, facial edema, dry skin, and cessation of menses. The nurse notes a heart rate of 60 beats per minute and a weight increase of 5 pounds from a previous visit. The nurse will notify the provider of which possible condition? a. Cretinism b. Early menopause c. Hyperthyroidism d. Myxedema

15. A patient will begin taking amoxicillin. The nurse should instruct the patient to avoid which foods? a. Green leafy vegetables b. Beef and other red meat c. Coffee, tea, and colas d. Acidic fruits and juices

ANS: D Acidic fruits and juices should be avoided while the client is being treated with amoxicillin because amoxicillin can be irritating to the stomach. Stomach irritation will be increased with the ingestion of citrus and acidic foods. Amoxicillin may also be less effective when taken with acidic fruit or juice.

1. The nurse is caring for a patient who has asthma and administers a selective beta2-adrenergic agonist to treat bronchospasm. The nurse will expect this drug to also cause which side effect? a. Increased blood glucose b. Increased blood pressure c. Increased heart rate d. Increased gastrointestinal (GI) motility

ANS: A Drugs that act on beta2 receptors activate glyconeogenesis in the liver causing increased blood glucose. Selective beta2 drugs act on beta2 receptors only and not on beta1 receptors, so they do not cause increased blood pressure or increased heart rate. Adrenergic agonists cause decreased GI motility.

9. The nurse is preparing to give a dose of dasatinib (Sprycel) to a patient. The nurse notes that the patient has just completed a meal. The tablet is in the packaging but is broken in two pieces. What is the correct action by the nurse? a. Administer the medication with an antacid. b. Return the tablet to the pharmacy. c. Wait for 2 hours before giving the medication. d. Wear gloves when handling the tablet.

9. The nurse is preparing to give a dose of dasatinib (Sprycel) to a patient. The nurse notes that the patient has just completed a meal. The tablet is in the packaging but is broken in two pieces. What is the correct action by the nurse? a. Administer the medication with an antacid. b. Return the tablet to the pharmacy. c. Wait for 2 hours before giving the medication. d. Wear gloves when handling the tablet.

20. The nurse administers albuterol to a patient who has asthma. The albuterol acts by stimulating beta2-adrenergic receptors to cause bronchodilation. The nurse understands that albuterol is a beta-adrenergic a. agonist. b. antagonist. c. inhibitor. d. depressant.

ANS: A An agonist medication is one that stimulates a certain type of cell to produce a response.

23. The nurse is preparing to administer the first dose of digoxin (Lanoxin) to a patient and notes that the dose ordered is much higher than the usual recommended dose. Which action will the nurse perform? a. Administer the dose as ordered. b. Give the dose and monitor for toxicity. c. Hold the dose until reviewing it with the provider. d. Refuse to give the dose.

ANS: A Digoxin requires a loading dose when first prescribed

7. The nurse is preparing an injectable drug and wants to administer it for rapid absorption. How will the nurse give this medication? a. IM into the deltoid muscle b. IM into the gluteal muscle c. SubQ into abdominal tissue d. SubQ into the upper arm

ANS: A Drugs given IM are absorbed faster in muscles that have more blood vessels, such as the deltoid, rather than those with fewer blood vessels, such as the gluteals. Subcutaneous routes are used when absorption needs to be slower and more sustained

12.Silver sulfadiazine (Silvadene) is used for the treatment of second- and third-degree burns. The highest priority nursing intervention related to this drug is to monitor for: a. crystalluria. b. dehydration. c. headaches. d. hypertension.

ANS: A Extended use of silver sulfadiazine may lead to crystalluria.

9. The parent of an adolescent who has taken methylphenidate 20 mg/day for 6 months for attention deficit/hyperactivity disorder (ADHD) brings the child to clinic for evaluation of a recent onset of nausea, vomiting, and headaches. The parent expresses concern that the child seems less focused and more hyperactive than before. What will the nurse do next? a. Ask the child whether the drug is being taken as prescribed. b. Contact the provider to discuss increasing the dose to 30 mg/day. c. Recommend taking the drug with meals to reduce gastrointestinal side effects. d. Report signs of drug toxicity to the patient's provider.

ANS: A Nausea, vomiting, and headaches can occur with drug withdrawal, along with a recurrence of symptoms. The nurse should ask the child about drug compliance. Methylphenidate should be taken 30 to 45 minutes before meals, not with meals.

12. A patient who has osteoarthritis with mild to moderate pain asks the nurse about taking over-the-counter ibuprofen (Motrin). What will the nurse tell this patient? a. "It may take several weeks to achieve therapeutic effects." b. "Unlike aspirin, there is no increased risk of bleeding with ibuprofen." c. "Take ibuprofen twice daily for maximum analgesic benefit." d. "Combine ibuprofen with acetaminophen for best effect."

ANS: A OTC NSAIDs can be effective for mild to moderate arthritis pain, but the effects may not appear for several weeks. NSAIDs carry a risk for bleeding. Ibuprofen is taken every 4 hours or QID. Ibuprofen should not be combined with aspirin or acetaminophen.

3. The nurse administers proparacaine HCl (Ophthaine) drops to a patient prior to an eye examination. What sign will the nurse look for to determine when the examination can begin? a. Absence of the blink reflex b. Blurred vision c. Drying of the corneal epithelium d. Photophobia

ANS: A Ophthaine is a topical anesthetic for the eye and causes loss of the blink reflex. Drying of the corneal epithelium is a side effect but occurs later as a result of the loss of the blink reflex. Blurred vision and photophobia result from mydriasis and miosis.

14. Which patient may require a higher than expected dose of an opioid analgesic? a. A patient with cancer b. A patient with a concussion c. A patient with hypotension d. A patient 3 days after surgery

ANS: A Opioids are titrated for oncology patients until pain relief is achieved or the side effects become intolerable, and extremely high doses may be required. Patient with closed head injuries should receive reduced doses of opioids if at all to reduce the risk of increased intracranial pressure. Patients with hypotension should receive reduced doses to prevent further decrease in blood pressure. Patients who are 3 days post-operation should not be experiencing severe pain.

3. The nurse is teaching a patient about taking hydrochlorothiazide. Which statement by the patient indicates a need for further teaching? a. "I may need extra sodium and calcium while taking this drug." b. "I should eat plenty of fruits and vegetables while taking this medication." c. "I should take care when rising from a bed or chair when I'm on this medication." d. "I will take the medication in the morning to minimize certain side effects."

ANS: A Patients do not need extra sodium or calcium while taking thiazide diuretics. Thiazide diuretics can lead to hypokalemia, so patients should be counseled to eat fruits and vegetables that are high in potassium. Patients can develop orthostatic hypotension and should be counseled to rise from sitting or lying down slowly. Taking the medication in the morning helps to prevent nocturia-induced insomnia.

12. The nurse has just begun administering intravenous streptokinase (Streptase). The nurse assesses vital signs and notes a temperature of 37° C, a heart rate of 70 beats per minute, and a blood pressure of 88/58 mm Hg. The nurse will contact the provider to a. request an adjustment of the streptokinase dose. b. request an order for aminocaproic acid (Amicar). c. request epinephrine to prevent anaphylaxis. d. report potential hemorrhage in this patient.

ANS: A Patients receiving streptokinase may experience hypotension when it is first administered and may require an adjustment in dosage. Aminocaproic acid is used to stop bleeding. Epinephrine is given for anaphylaxis, which is characterized by difficulty breathing. A patient with hemorrhage would typically also have tachycardia.

2. Prior to administration of interferon alpha, the nurse will administer which medications? a. Acetaminophen and diphenhydramine b. Heparin and meperidine c. Lorazepam and furosemide d. Narcotic analgesics and loratadine

ANS: A Patients receiving these drugs should be premedicated with acetaminophen to reduce chills and fever and with diphenhydramine to reduce nausea.

11. The nurse is preparing to give sargramostim to a patient who has acute myelogenous leukemia (AML). The nurse assesses a heart rate of 78 beats per minute and a blood pressure of 120/70 mm Hg. The patient reports shortness of breath and has a cough and bilateral crackles. What will the nurse do next? a. Contact the provider; discuss giving a lower dose. b. Contact the pharmacist; request a bronchodilator. c. Contact the pharmacist; request an order for furosemide. d. Contact the provider; suggest administration of antibiotics.

ANS: A Patients receiving this drug can experience sequestration of granulocytes in the pulmonary circulation and may experience dyspnea. The sargramostim infusion should be reduced in half if this occurs. Bronchospasm, pulmonary edema, and infection are not common side effects.

9. A patient, who has intermittent claudication, has been taking 400 mg of pentoxifylline (Trental) three times daily with meals for 2 weeks. The patient calls the clinic and reports mild flushing, occasional gastrointestinal upset, and continued pain in both legs. How will the nurse advise the patient? a. "Expect side effects to diminish as drug effects increase in several weeks." b. "Notify the provider of the continued pain and request increasing the dose." c. "Take a daily aspirin tablet to enhance the effects of pentoxifylline." d. "Take the medication 1 hour before or 2 hours after a meal."

ANS: A Patients should be counseled that the desired therapeutic effects may take to 3 months. This patient's side effects are mild and therefore do not warrant discontinuing the drug. This patient is receiving the maximum recommended dose. Aspirin is not indicated. Taking the medication with meals and not on an empty stomach minimizes gastrointestinal effects.

2. The nurse is counseling a patient who will begin taking a sulfonamide drug to treat a urinary tract infection. What information will the nurse include in teaching? a. "Drink several quarts of water daily." b. "If stomach upset occurs, take an antacid." c. "Limit sun exposure to no more than 1 hour each day." d. "Sore throat is a common, harmless side effect."

ANS: A Patients should drink several quarts of water daily while taking sulfonamides to prevent crystalluria. Patients should not take antacids with sulfonamides. Patients should not go out into the sun. Sore throat should be reported.

2. The nurse is teaching a female patient who will begin taking 2 tablets of 325 mg acetaminophen every 4 to 6 hours as needed for pain. Which statement by the patient indicates understanding of the teaching? a. "I may take acetaminophen up to 6 times daily if needed." b. "I should increase the dose of acetaminophen if I drink caffeinated coffee." c. "If I take oral contraceptive pills, I should use back-up contraception." d. "It is safe to take acetaminophen with any over-the-counter medications."

ANS: A The maximum daily dose of acetaminophen is 4000 mg. If this patient takes 650 mg/dose 6 times daily, this amount is safe. Taking acetaminophen with caffeine increases the effect of the acetaminophen. Taking acetaminophen with OCPs decreases the effect of the acetaminophen but does not diminish the effect of the OCP. Many over-the-counter medications contain acetaminophen, so patients should be advised to read labels carefully to avoid overdose.

8. A patient who is taking the tyrosine kinase inhibitor sunitinib (Sutent) calls to report red, painful, and swollen palms and soles of feet. The nurse will perform which action? a. Notify the patient's provider of this adverse reaction. b. Reassure the patient that these are common side effects. c. Recommend taking acetaminophen for discomfort. d. Suggest taking diphenhydramine to help with the swelling.

ANS: A The nurse should notify the provider if the patient reports these symptoms, since they may indicate erythrodysesthesia. Reassuring the patient or recommending OTC treatments is not indicated.

6. The nurse is caring for a young adult patient who is receiving a first dose of flurazepam (Dalmane) as a sedative-hypnotic medication. What intervention will be included in the nurse's plan of care for this patient? a. Instituting a bed alarm system to prevent falls b. Reassuring the patient that nightmares are not a usual effect c. Reporting a urine output greater than 1500 mL/day d. Teaching the patient that this drug may be used for 6 to 8 weeks

ANS: A The nurse should use a bed alarm for older patients and younger patients receiving a hypnotic for the first time. Patients may experience vivid dreams and nightmares. Urine output should be greater than 1500 mL/day, so this does not warrant reporting. This drug should be used short-term.

3. A patient is taking ibuprofen 400 mg every 4 hours to treat moderate arthritis pain and reports that it is less effective than before. What action will the nurse take? a. Counsel the patient to discuss a prescription NSAID with the provider. b. Recommend adding aspirin to increase the antiinflammatory effect. c. Suggest asking the provider about a short course of corticosteroids. d. Tell the patient to increase the dose to 800 mg every 4 hours.

ANS: A The patient should discuss another NSAID with the provider if tolerance has developed to the over-the-counter NSAID. Patients should not take aspirin with NSAIDs because of the increased risk of bleeding and gastrointestinal upset. Steroids are not the drugs of choice for arthritis because of their side effects and are not used unless inflammation is severe. A prescription NSAID would be used prior to starting corticosteroids. Increasing the dose will increase side effects but may not increase desired effects. The maximum dose per day is 2400 mg, which would most likely be exceeded when increasing the dose to 800 mg every 4 hours.

8. The nurse administers a dose of digoxin (Lanoxin) to a patient who has heart failure and returns to the room later to reassess the patient. Which finding indicates that the medication is effective? a. Decreased dyspnea b. Decreased urine output c. Increased blood pressure d. Increased heart rate

ANS: A The patient should show improvement in breathing and oxygenation. Urine output should increase. Blood pressure and heart rate will decrease.

2. A nursing student asks why the anticoagulant heparin is given to patients who have disseminated intravascular coagulation (DIC) and are at risk for excessive bleeding. The nurse will explain that heparin is used in this case for which reason? a. To decrease the risk of venous thrombosis b. To dissolve blood clots as they form c. To enhance the formation of fibrous clots d. To preserve platelet function

ANS: A The primary use of heparin for patients with DIC is to prevent venous thrombosis, which can lead to pulmonary embolism or stroke. Heparin does not break down blood clots, enhance the formation of fibrous clots, or preserve platelet function.

8. The nurse is caring for a postoperative patient and notes that the patient received atropine sulfate preoperatively. Which assessment finding would prompt the nurse to notify the provider? a. Absent bowel sounds b. Drowsiness c. Dry mouth d. Heart rate of 78 beats per minute

ANS: A These are all side effects of atropine. Absent bowel sounds can indicate a paralytic ileus. The other side effects are not harmful.

13. A child who weighs 10 kg will begin taking oral trimethoprim-sulfamethoxazole (TMP-SMX). The liquid preparation contains 40 mg of TMP and 200 mg of SMX per 5 mL. The nurse determines that the child's dose should be 8 mg of TMP and 40 mg of SMX/kg/day divided into two doses. Which order for this child is correct? a. 5 mL PO BID b. 5 mL PO daily c. 10 mL PO BID d. 10 mL PO daily

ANS: A This child should receive (10 kg ´ 8 mg) 80 mg of TMP and (10 kg ´ 40 mg) 400 mL of SMX per day. When divided into two doses, the correct dose is 40 mg TMP and 200 mg SMX, or 5 mL per dose.

6. The nurse is caring for a male patient with myasthenia gravis who will begin taking ambenonium chloride (Mytelase). When performing a health history, the nurse will be concerned about a history of which condition in this patient? a. Benign prostatic hypertrophy b. Chronic constipation c. Erectile dysfunction d. Upper respiratory infection

ANS: A This drug is a reversible cholinesterase inhibitor and is given to increase muscle strength. Cholinesterase inhibitors are contraindicated in patients with urinary tract obstruction.

1. An older patient exhibits a shuffling gait, lack of facial expression, and tremors at rest. The nurse will expect the provider to order which medication for this patient? a. Carbidopa-levodopa (Sinemet) b. Donepezil (Aricept) c. Rivastigmine (Exelon) d. Tacrine (Cognex)

ANS: A This patient is exhibiting signs of Parkinson's disease and should be treated with carbidopa-levodopa. The other drugs are used to treat Alzheimer's disease.

13. The nurse is preparing to administer benztropine (Cogentin) to a patient who has Parkinson's disease. When performing an assessment, which aspect of the patient's history would cause the nurse to hold the medication and notify the provider? a. Asthma b. Glaucoma c. Irritable bowel syndrome d. Motion sickness

ANS: B Patients who have glaucoma should not take anticholinergic medications.

1. Which are characteristic signs of inflammation? (Select all that apply.) a. Edema b. Erythema c. Heat d. Numbness e. Pallor f. Paresthesia

ANS: A, B, C Edema, erythema, and heat are signs of inflammation. The other three are signs of neurocirculatory compromise.

1. A patient who recently began having mild symptoms of GERD is reluctant to take medication. What measures will the nurse recommend to minimize this patient's symptoms? (Select all that apply.) a. Avoiding hot, spicy foods b. Avoiding tobacco products c. Drinking a glass of red wine with dinner d. Eating a snack before bedtime e. Taking ibuprofen with food f. Using a small pillow for sleeping g. Wearing well-fitted clothing

ANS: A, B, E Hot, spicy foods aggravate gastric upset, tobacco increases gastric secretions, and ibuprofen on an empty stomach increases gastric secretions, so patients should be taught to avoid these actions. Alcohol should be avoided since it increases gastric secretions. Eating at bedtime increases reflux, as does laying relatively flat to sleep, or wearing fitted clothing.

1. The nurse teaches a patient about antihypertensive medication. Which statements by the patient indicate understanding of the teaching? (Select all that apply.) a. "I should be careful when I stand up from a chair." b. "I should not add extra salt to my foods." c. "If I have side effects, I should stop taking the drug immediately." d. "If my blood pressure returns to normal, I can stop taking this drug." e. "I may need to take a combination of drugs, including diuretics." f. "I will not need to make lifestyle changes since I am taking a medication."

ANS: A, B, E The patient receiving an antihypertensive medication should be warned to rise slowly to avoid orthostatic hypotension. Patients should be counseled to continue to make lifestyle changes, including decreasing sodium. Often, more than one medication is required. Patients should not stop taking the drug abruptly to avoid rebound hypertension and will not stop the drug when blood pressure returns to normal.

1. The nurse caring for a patient who is taking an adrenergic agent will expect which side effects? (Select all that apply.) a. Dilated pupils b. Increased heart rate c. Increase gastrointestinal motility d. Vasodilation e. Bronchospasm f. Relaxed uterine muscles

ANS: A, B, F Adrenergic agents stimulate the sympathetic nervous system, evoking the "fight or flight" response. This response increases those functions needed to respond to stress (increased heart rate to perfuse muscles, bronchodilation to increase oxygen exchange). Adrenergic drugs shunt blood away from the reproductive tract and gastrointestinal organs as these functions are not needed during a fight or flight response.

1. Which actions can contribute to bacterial resistance to antibiotics? (Select all that apply.) a. Frequent use of antibiotics b. Giving large doses of antibiotics c. Skipping doses d. Taking a full course of antibiotics e. Treating viral infections with antibiotics

ANS: A, C, E Frequent use of antibiotics increases the exposure of bacteria to an antibiotic and results in acquired resistance. Skipping doses of an antibiotic can lead to incomplete treatment of an infection, and the remaining bacteria may develop acquired resistance. Treating viral infections with antibiotics is unnecessary and may cause acquired resistance to develop from unneeded exposure to a drug. Infections adequately treated with an antibiotic do not result in resistance.

1. The nurse is caring for a patient who is receiving fluorouracil (5-FU) to treat pancreatic cancer. Which interventions are included in the nurse's plan of care for this patient? (Select all that apply.) a. Apply ice to the IV site if the patient reports pain. b. Administer antiemetics when the patient reports nausea. c. Counsel the patient to use waxed dental floss. d. Discourage visits with people who have respiratory infections. e. Offer ice chips frequently. f. Restrict to nothing by mouth during intravenous drug administration.

ANS: A, C, E If the patient reports pain at the IV site, the nurse should apply ice and notify the provider. Patients should use waxed dental floss to avoid bleeding of the gums. Ice chips help with oral pain. Antiemetics should be given prior to administration of the drug. Visitors with active infections should be restricted. Patients do not need to be NPO during the IV infusion.

1. A client with myasthenia gravis is experiencing a cholinergic crisis. Which symptoms are associated with this condition? (Select all that apply.) a. Bradycardia b. Rash c. Vomiting d. Fever e. Drooling f. Weakness

ANS: A, C, E, F Bradycardia, drooling, and weakness can all occur with cholinergic crisis.

1. A client is being treated for tuberculosis. Which medications are used to treat this condition? (Select all that apply.) a. Streptomycin sulfate b. Amoxicillin (Amoxil) c. Ethambutol (Myambutol) d. Gentamicin (Garamycin) e. Rifabutin (Mycobutin) f. Ethionamide (Trecator-SC) g. Pyrazinamide

ANS: A, C, E, F, G Streptomycin sulfate, ethambutol (Myambutol), rifabutin (Mycobutin), ethionamide (Trecator-SC), and pyrazinamide are used to treat tuberculosis. The other medications are not used.

1. Which diseases are caused by herpes viruses? (Select all that apply.) a. Chicken pox b. Hepatitis c. Influenza d. Mononucleosis e. Shingles

ANS: A, D, E Herpes viruses cause chicken pox, mononucleosis, and shingles.

8. A patient asks the nurse about taking over-the-counter sleeping aids. The nurse will tell the patient that the active ingredient in these products is often a(n) a. antiemetic. b. antihistamine. c. barbiturate. d. benzodiazepine.

ANS: B The primary ingredient in OTC sleep aids is an antihistamine such as diphenhydramine, not barbiturates or benzodiazepines.

16. The nurse is preparing to administer a drug that is ordered to be given twice daily. The nurse reviews the medication information and learns that the drug has a half-life of 24 hours. What will the nurse do next? a. Administer the medication as ordered. b. Contact the provider to discuss daily dosing. c. Discuss every-other-day dosing with the provider. d. Hold the medication and notify the provider.

ANS: B A drug with a longer half-life should be given at longer intervals to avoid drug toxicity.

8.Contact dermatitis may be caused by chemical or plant irritation. What nonpharmacologic measure may aid in alleviating the problem? a. Determining causative agent b. Cleansing the skin area immediately c. Wearing protective gloves or clothing d. Applying a sterile dressing over the involved area

ANS: B Cleansing is one of the chief methods to decrease the irritation that has been caused by contact dermatitis.

13.A client is ordered to receive isotretinoin. What is a priority diagnostic test for the nurse to complete before beginning therapy? a. Blood glucose level b. Pregnancy test c. Serum electrolytes d. Complete blood count

ANS: B Isotretinoin is highly teratogenic and includes strict guidelines related to ensuring safe use. Female clients who are sexually active will be asked to take a pregnancy test.

13. The nurse gives a medication to a patient with a history of liver disease. The nurse will monitor this patient for a. decreased drug effects. b. increased drug effects. c. decreased therapeutic range. d. increased therapeutic range.

ANS: B Liver diseases such as cirrhosis and hepatitis alter drug metabolism by inhibiting the drug-metabolizing enzymes in the liver. When the drug metabolism rate is decreased, excess drug accumulation can occur and lead to toxicity.

2. A patient who has asthma is diagnosed with hypertension. The nurse understands that which drug will be safe to give this patient? a. Pindolol (Visken) b. Metoprolol (Lopressor) c. Nadolol (Corgard) d. Propranolol (Inderal)

ANS: B Metoprolol is a selective adrenergic blocker that has a greater affinity for receptors that decrease heart rate and blood pressure and is less likely to cause bronchospasm. The other adrenergic blockers are not selective and can cause bronchoconstriction.

5.Upon assessment of the burned client, the nurse notes that the client is exhibiting mottled, blistered skin and is complaining of intense pain. These findings are congruent with which degree of burn injury? a.First b. second c. Third d. Fourth

ANS: B Second-degree burns are characterized by mottled, blistered skin, and the client is typically in intense pain.

15. If a drug has a half-life of 12 hours and is given twice daily starting at 0800 on a Monday, when will a steady state be achieved? a. 0800 on Tuesday b. 0800 on Wednesday c. 0800 on Thursday d. 0800 on Friday

ANS: B Steady-state levels occur at 3 to 5 half-lives. Wednesday at 0800 is 4 half-lives from the original dose.

19. The nurse understands that the length of time needed for a drug to reach the minimum effective concentration (MEC) is the a. duration of action. b. onset of action. c. peak action time. d. time response curve.

ANS: B The onset of action is the time it takes to reach the MEC. Duration of action is the length of time a drug has a pharmacologic effect. Peak action time occurs when the drug reaches its highest blood level. The time response curve is an evaluation of the other three measures.

4. The nurse administers bethanechol (Urecholine) to a patient to treat urinary retention. After 30 minutes, the patient voids 800 mL of urine and reports having a loose stool but no cramping or gastrointestinal pain. The patient's blood pressure is 110/70 mm Hg. The nurse will perform which action? a. Notify the provider of bethanechol adverse effects. b. Record the urine output and the blood pressure and continue to monitor. c. Request an order for intravenous atropine sulfate. d. Suggest another dose of bethanechol to the provider.

ANS: B The patient is exhibiting desired effects and mild side effects of bethanechol, so the nurse should record information and continue to monitor the patient. There is no need to notify the provider, give an antidote, or repeat the dose.

5. The nurse is preparing to administer an oral medication that is water-soluble. The nurse understands that this drug a. must be taken on an empty stomach. b. requires active transport for absorption. c. should be taken with fatty foods. d. will readily diffuse into the gastrointestinal tract.

ANS: B Water-soluble drugs require a carrier enzyme or protein to pass through the GI membrane.

8. Which antiviral medication improves symptoms of Parkinson's disease in some patients? a. Acyclovir (Zovirax) b. Amantadine HCl (Symmetrel) c. Interferon (INF) d. Zanamivir (Relenza)

ANS: B Amantadine is an antiviral drug that acts on dopamine receptors and is sometimes used to treat Parkinson's disease (PD). The other drugs listed do not work for PD patients.

2. The nurse is caring for a patient who has unexplained, recurrent vomiting and who is unable to keep anything down. Until the cause of the vomiting is determined, the nurse will anticipate administering which medications? a. Antibiotics and antiemetics b. Intravenous fluids and electrolytes c. Non-prescription antiemetics d. Prescription antiemetics

ANS: B Antiemetics can mask the underlying cause of vomiting and should not be used until the cause is determined unless vomiting is so severe that dehydration and electrolyte imbalance occurs. Nonpharmacologic measures, such as fluid and electrolyte replacement, should be used. Antibiotics are only used if an infectious cause is determined.

11. A patient who has parkinsonism will begin taking carbidopa-levodopa. What information will the nurse include when teaching this patient about this medication? a. "Call your health care provider immediately if your urine or perspiration turn a dark color." b. "Rise slowly from your bed or your chair to avoid dizziness and falls." c. "Take the drug with foods high in protein to improve drug delivery." d. "Discontinue the drug if you experience insomnia."

ANS: B Carbidopa-levodopa can cause orthostatic hypotension, so patients should be taught to take care when getting out of bed or a chair. Darkening of the urine and perspiration is a harmless side effect. Patients should take the drug with low-protein foods to improve drug transport to the CNS. Carbidopa-levodopa should not be discontinued abruptly because rebound parkinsonism may occur; insomnia is an expected adverse effect of the drug, and the patient should report this effect to his or her health care provider.

8. A patient is receiving the antitumor antibiotic doxorubicin (Adriamycin) to treat lung cancer. The patient is experiencing shortness of breath and palpitations. The nurse is concerned that the patient has developed which condition? a. Anemia b. Cardiotoxicity c. Hypersensitivity d. Pulmonary infection

ANS: B Cardiotoxicity is a known adverse effect of this drug and is manifested in shortness of breath, edema, and palpitations.

6. The nurse is caring for a patient who is hospitalized for an asthma exacerbation. The patient reports taking diphenhydramine at home at night to help with symptoms of allergic rhinitis and cough. The nurse will contact the patient's provider to request an order for which medication? a. Benzonatate (Tessalon Perles) b. Cetirizine (Zyrtec) c. Dextromethorphan hydrobromide (Benylin DM) d. Diphenhydramine (Benadryl)

ANS: B Cetirizine is an antihistamine, which is indicated for this patient's symptoms. Diphenhydramine is also an antihistamine but, because of its anticholinergic side effects, is contraindicated in patients with asthma. Benzonatate and dextromethorphan are anti-tussives and not antihistamines.

17. The nurse is preparing to care for a patient who has multiple sclerosis (MS). The nurse learns that the patient receives cyclophosphamide (Cytoxan). The nurse knows that this patient is in which stage of MS? a. Acute attack phase b. Chronic, progressive phase c. End-stage phase d. Remission-exacerbation phase

ANS: B Cyclophosphamide is used to treat MS patients who are in the chronic, progressive phase.

9. The nurse is providing teaching for a patient who will use intranasal dexamethasone (Decadron) after discharge home from the hospital. What information is important to include when teaching this patient about this drug? a. "Dexamethasone may be used for year-round symptoms." b. "Dexamethasone should be discontinued after 30 days." c. "Dexamethasone should not be taken with antihistamines." d. "Dexamethasone should not cause systemic steroid side effects."

ANS: B Dexamethasone should not be used longer than 30 days because longer use increases the risk of systemic side effects. Dexamethasone should not be used year-round. It may be used in conjunction with antihistamines. Systemic side effects may occur.

5. The parents of a 3-year-old child tell the nurse that they are planning to give their child diphenhydramine (Benadryl) on a flight to visit the child's grandparents to help the child sleep during the flight. What will the nurse tell the parents about giving this drug? a. Administer 25 mg of diphenhydramine when using to induce sleep. b. Diphenhydramine may have the opposite effect and could cause agitation. c. Give the diphenhydramine about 5 minutes prior to takeoff. d. Loratadine should be used instead of diphenhydramine to minimize side effects.

ANS: B Diphenhydramine can cause excitation in some children. Parents should be advised to expect this possible side effect. The correct dose of diphenhydramine for children at this age is 6.25 mg; 25 mg would be an overdose. Oral diphenhydramine has an onset of 15 to 45 minutes. Loratadine is a second-generation antihistamine and does not cause drowsiness.

2. The nurse is caring for a patient who is receiving diphenhydramine. The nurse notes that the patient has not voided for 12 hours. What action will the nurse take? a. Encourage the patient to drink more fluids. b. Evaluate the bladder to check for distension. c. Request an order for an intravenous fluid bolus. d. Request an order for urinary catheterization.

ANS: B Diphenhydramine has anticholinergic effects, including urinary retention. The nurse should assess for bladder distension to determine if this is the case. Encouraging the patient to drink more fluids or giving intravenous fluids may be necessary if the patient has oliguria secondary to dehydration. Urinary catheterization is not indicated until urinary retention has been identified.

9. A patient reports weakness of the extremities and diplopia. The nurse knows that these symptoms are characteristic of which condition? a. Cerebral palsy (CP) b. Multiple sclerosis (MS) c. Myasthenia gravis (MG) d. Parkinson's disease (PD)

ANS: B Diplopia and weakness of the extremities are two symptoms of MS. CP is characterized by muscle spasticity. MG involves generalized weakness, especially of facial muscles and respiratory muscles. PD manifests as tremors and difficulty moving and walking.

10. A male patient who has been taking a histamine2 blocker for several months reports decreased libido and breast swelling. What will the nurse do? a. Contact the provider to report possible drug toxicity. b. Reassure the patient that these symptoms will stop when the drug is discontinued. c. Request an order for serum hormone levels. d. Suggest that the patient see an endocrinologist.

ANS: B Drug-induced impotence and gynecomastia are reversible drug side effects. These signs do not indicate drug toxicity. Serum hormone levels and endocrinology evaluation are not indicated.

13. A patient is taking entacapone (Comtan) along with carbidopa-levodopa to treat parkinsonism. The nurse notes that the patient's urine is orange in color. The nurse will a. notify the provider of possible drug toxicity. b. reassure the patent that this is a harmless side effect. c. request an order for liver function tests. d. request an order for a urinalysis.

ANS: B Entacapone can cause the urine to be dark yellow to orange. It does not indicate drug toxicity, liver effects, or changes in renal function.

2. A nurse whose last flu vaccine was 1 year prior is exposed to the influenza A virus. The occupational health nurse will administer which medication? a. Acyclovir (Zovirax) b. Amantadine HCl (Symmetrel) c. Influenza vaccine d. Oseltamivir phosphate (Tamiflu)

ANS: B The primary use for amantadine is prophylaxis against influenza A. Acyclovir is used to treat herpes virus. Oseltamivir phosphate (Tamiflu) is to be taken once flu symptoms appear.

15. The nurse assesses an older patient 60 minutes after administering 4 mg of intravenous morphine sulfate (MS) for postoperative pain. The patient's analgesia order is for 2 to 5 mg of MS IV every 2 hours. The nurse notes that the patient is lying very still. The patient's heart rate is 96 beats per minute, respiratory rate is 14 breaths per minute, and blood pressure is 140/90 mm Hg. When asked to rate the level of pain, the patient replies "just a 5." The nurse will perform which action? a. Give 3 mg of MS at the next dose. b. Give 5 mg of MS at the next dose. c. Request an order for an oral opioid to give now. d. Request an order for acetaminophen to give now.

ANS: B Older patients often minimize pain when asked, so the nurse should evaluate nonverbal cues to pain such as elevated heart rate and blood pressure and the fact that the patient is lying very still. The nurse should increase the dose the next time the pain medication is given.

6. The nurse receives the following order for insulin: IV NPH (Humulin NPH) 10 units. The nurse will perform which action? a. Administer the dose as ordered. b. Clarify the insulin type and route. c. Give the drug subcutaneously. d. Question the insulin dose.

ANS: B Only regular insulin can be given intravenously. The nurse should clarify the order. It is not correct to give Humulin NPH insulin IV. The nurse should not administer the drug by a different route without first discussing with the provider.

5. The nurse is caring for a patient who has myasthenia gravis (MG) and is receiving pyridostigmine bromide (Mestinon). The nurse notes ptosis of both eyelids and observes that the patient has difficulty swallowing. What action will the nurse perform next? a. Contact the provider to request an order for atropine sulfate. b. Contact the provider to request an order for edrophonium chloride (Tensilon). c. Report signs of cholinergic crisis to the provider. d. Report signs of myasthenic crisis to the provider.

ANS: B Overdosing and underdoing of AChE inhibitors have similar symptoms: muscle weakness, dyspnea, and dysphagia. Edrophonium may be used to diagnose MG or to distinguish between myasthenic crisis and cholinergic crisis since it is a very short-acting AChE inhibitor. When given, if the symptoms are alleviated, the cause is myasthenic crisis; if symptoms worsen, it is cholinergic crisis. Since patients can have similar symptoms, the nurse cannot report one or the other to the provider without more information.

3. The spouse of a patient newly diagnosed with mild, unilateral symptoms of Parkinson's disease (PD) asks the nurse what, besides medication, can be done to manage the disease. The nurse will a. counsel the spouse that parkinsonism is a normal part of the aging process in some people. b. recommend exercise, nutritional counseling, and group support to help manage the disease. c. tell the spouse that the disease will not progress if mild symptoms are treated early. d. tell the spouse that medication therapy can be curative if drugs are begun in time.

ANS: B PD is a progressive disorder. Nonpharmacologic measures can lessen symptoms and help patients and families cope with the disorder. Although the aging process may contribute to the development of PD, it is not necessarily a normal part of aging. Treatment may slow the progression but does not arrest or cure the disease.

20. A patient is receiving a glucocorticoid medication to treat an inflammatory condition, and the provider has ordered a slow taper in order to discontinue this medication. The nurse explains to the patient that this is done to prevent which condition? a. Acromegaly b. Adrenocortical insufficiency c. Hypertensive crisis d. Thyroid storm

ANS: B Patients receiving glucocorticoids stop making their own cortisol. These drugs should be tapered slowly to allow the body to resume making this hormone. Acromegaly is associated with growth hormone hypersecretion. Hypertensive crisis and thyroid storm are associated with thyroid replacement.

19. The nurse provides teaching for a patient who will begin taking allopurinol. Which statement by the patient indicates understanding of the teaching? a. "I should increase my vitamin C intake." b. "I will get yearly eye exams." c. "I will increase my protein intake." d. "I will limit fluids to prevent edema."

ANS: B Patients taking allopurinol can have visual changes with prolonged use and should have yearly eye exams. It is not necessary to increase vitamin C. Protein can increase purine intake, which is not recommended. Patients should consume extra fluids.

14. The nurse is caring for an 80-year-old patient who has Alzheimer's disease who will begin taking rivastigmine (Exelon). What will the nurse include in the plan of care for this patient? a. Administer the drug once daily. b. Assist the patient to stand and walk. c. Give the drug with food to increase absorption. d. Use nonsteroidal anti-inflammatory drugs (NSAIDs) instead of acetaminophen for pain.

ANS: B Patients taking rivastigmine for Alzheimer's disease are at risk for falls and loss of balance. Caregivers should assist with standing and walking. The drug is taken twice daily, and it should be taken on an empty stomach. NSAIDs increase gastrointestinal side effects.

7. A patient has been taking atorvastatin (Lipitor) for several months to treat hyperlipidemia. The patient reports muscle weakness and tenderness. The nurse will counsel the patient to a. ask the provider about switching to simvastatin. b. contact the provider to report these symptoms. c. start taking ibuprofen to combat these effects. d. stop taking the medication immediately.

ANS: B Patients taking statins should report immediately any muscle aches or weakness, which can lead to rhabdomyolysis, a muscle disintegration that can become fatal. All statins carry this risk, so changing to another statin is not indicated. Ibuprofen may be useful, but notifying the provider is essential. Patients should not abruptly discontinue statins without discussing this with the provider.

16. A patient asks the nurse the best way to prevent traveler's diarrhea. The nurse will provide which recommendation to the patient? a. "Ask your provider for prophylactic antibiotics." b. "Drink bottled water and eat only well-cooked meats." c. "Eat fresh, raw fruits and vegetables." d. "Take loperamide (Imodium) every day."

ANS: B Patients traveling to areas with potential traveler's diarrhea should be taught to drink bottled water and eat meats that are well-cooked. Prophylactic antibiotics are not recommended. Patients should eat cooked, washed fruits and vegetables. Loperamide can increase exposure to pathogens by slowing motility.

2. A patient who has chronic liver disease reports contact with a person who has tuberculosis (TB). The nurse will counsel this patient to contact the provider to discuss a. a chest x-ray. b. a TB skin test. c. liver function tests (LFTs). d. prophylactic antitubercular drugs.

ANS: B Patients who have exposure to TB should have a TB skin test. A chest x-ray is performed if the skin test is positive. LFTs do not need to be done simply because of TB exposure. This patient is not a candidate for antitubercular drug prophylaxis.

13. The nurse is caring for a postoperative patient who is receiving alteplase tPA (Activase) after developing a blood clot. The nurse notes a heart rate of 110 beats per minute and a blood pressure of 90/60 mm Hg. The nurse will perform which action? a. Ask the patient about itching or shortness of breath. b. Assess the surgical dressing for bleeding. c. Evaluate the patient's urine output and fluid intake. d. Recheck the patient's vital signs in 15 minutes.

ANS: B Tachycardia and hypotension indicate bleeding. The nurse should check the patient's surgical dressing to assess for bleeding. These signs do not indicate anaphylaxis. They may indicate dehydration, but bleeding is the more likely cause of fluid volume deficit. The nurse should continue to evaluate vital signs, but it is imperative that the nurse assess the patient to explore the potential cause.

3. A patient with chronic obstructive pulmonary disease (COPD) has increasing dyspnea and is being evaluated for heart failure (HF). Which test will be ordered to help differentiate between dyspnea due to lung dysfunction and dyspnea due to HF? a. Atrial natriuretic hormone (ANH) level b. Brain natriuretic peptide (BNP) level c. Cardiac enzymes d. Electrocardiogram (ECG)

ANS: B The BNP is used to differentiate that dyspnea is due to HF and not lung dysfunction. The other tests will all be a part of the diagnostic workup but do not help with this distinction.

3. The parent of an 18-month-old toddler calls the clinic to report that the child has vomited 5 times that day. The nurse determines that the child has had three wet diapers in the past 6 hours. What will the nurse recommend for this child? a. Administering an OTC antiemetic medication such as diphenhydramine b. Giving frequent, small amounts of Pedialyte c. Keeping the child NPO until vomiting subsides d. Taking the child to the emergency department for IV fluids

ANS: B The child is not dehydrated as evidenced by adequate wet diapers, so nonpharmacologic measures, such as oral fluids, are recommended. Antiemetics are not recommended unless dehydration occurs. Intravenous fluids are given when dehydration is present.

4. A patient who will begin taking trimethoprim-sulfamethoxazole (TMP-SMX) asks the nurse why the combination drug is necessary. The nurse will explain that the combination is used to a. broaden the antibacterial spectrum. b. decrease bacterial resistance. c. improve the taste. d. minimize toxic effects.

ANS: B The combination drug is used to decrease bacterial resistance to sulfonamides. It does not broaden the spectrum, improve the taste, or decrease toxicity.

15. The patient has been ordered treatment with rimantadine (Flumadine). The patient has renal impairment. The nurse anticipates what change to the dose of medication? a. Increased b. Decreased c. Unchanged d. Held

ANS: B The dosage of the medication will be decreased when the patient has renal impairment.

9. A patient who has completed the first phase of a three-drug regimen for tuberculosis has a positive sputum acid-bacilli test. The nurse will tell the patient that a. drug resistance has probably occurred. b. it may be another month before this test is negative. c. the provider will change the pyrazinamide to ethambutol. d. there may be a need to remain in the first phase of therapy for several weeks.

ANS: B The goal is for the patient's sputum test to be negative 2 to 3 months after the therapy. The positive test does not indicate drug resistance. The provider will not change the drugs or keep the patient in the first phase longer than planned.

14. The nurse is discussing celecoxib (Celebrex) with a patient who will use the drug to treat dysmenorrhea. What information will the nurse include in teaching? a. "Do not take the medication during the first 2 days of your period." b. "The initial dose will be twice the amount of subsequent doses." c. "Take this medication with food to minimize gastrointestinal upset." d. "Take the drug on a regular basis to prevent dysmenorrhea."

ANS: B The initial dose of Celebrex is twice that of subsequent doses. The medication should not be taken just before a period. It does not need to be taken with food. It is taken as needed.

17. The nurse is caring for a patient who is being treated for hypothyroidism. The patient reports insomnia, nervousness, and flushing of the skin. Before notifying the provider, the nurse will perform which action? a. Assess serum glucose to evaluate possible hypoglycemia. b. Check the patient's heart rate to assess for tachycardia. c. Perform an assessment of hydration status. d. Take the patient's temperature to evaluate for infection.

ANS: B The patient has signs of a thyroid crisis, which can occur with excess ingestion of thyroid hormone. The nurse should evaluate heart rate before notifying the provider. These are not symptoms of hypoglycemia. The symptoms are not indicative of infection.

12. The nurse is preparing to administer an angiotensin-converting enzyme (ACE) inhibitor to a patient who has hypertension. The nurse notes peripheral edema and swelling of the patient's lips. The patient has a blood pressure of 160/80 mm Hg and a heart rate of 76 beats per minute. What is the nurse's next action? a. Administer the dose and observe carefully for hypotension. b. Hold the dose and notify the provider of a hypersensitivity reaction. c. Notify the provider and request an order for a diuretic medication. d. Request an order for serum electrolytes and renal function tests.

ANS: B The patient has signs of angioedema which indicates a hypersensitivity reaction. The nurse should hold the dose and notify the provider. Giving the dose will make the reaction more serious. These are not signs of edema, so a diuretic is not indicated. Electrolytes and renal function tests are not indicated.

11. A patient has administered regular insulin 30 minutes prior but has not received a breakfast tray. The patient is experiencing nervousness and tremors. What is the nurse's first action? a. Administer glucagon. b. Give the patient orange juice. c. Notify the kitchen to deliver the tray. d. Perform bedside glucose testing.

ANS: B The patient is symptomatic and has hypoglycemia. The nurse should give orange juice. Glucagon is given for patients unable to ingest carbohydrates. The kitchen should be notified, and bedside glucose testing should be performed, but only after the patient is given carbohydrates.

10. The nurse assumes care of a patient in the post-anesthesia care unit (PACU). The patient had abdominal surgery and is receiving intravenous morphine sulfate for pain. The patient is asleep and has not voided since prior to surgery. The nurse assesses a respiratory rate of 10 breaths per minute and notes hypoactive bowel sounds. The nurse will contact the surgeon to report which condition? a. Paralytic ileus b. Respiratory depression c. Somnolence d. Urinary retention

ANS: B The patient's respiratory rate of 10 breaths per minute is lower than normal and is a sign of respiratory depression, which is a common adverse effect of opioid analgesics. The other effects may occur with opioids but are also not expected this soon after abdominal surgery.

4. A patient who is newly diagnosed with type 1 diabetes mellitus asks why insulin must be given by subcutaneous injection instead of by mouth. The nurse will explain that this is because a. absorption is diminished by the first-pass effects in the liver. b. absorption is faster when insulin is given subcutaneously. c. digestive enzymes in the gastrointestinal tract prevent absorption. d. the oral form is less predictable with more adverse effects.

ANS: C Insulin, growth hormones, and other protein-based drugs are destroyed in the small intestine by digestive enzymes and must be given parenterally. Because insulin is destroyed by digestive enzymes, it would not make it to the liver for metabolism with a first-pass effect. Subcutaneous tissue has fewer blood vessels, so absorption is slower in such tissue. Insulin is given subcutaneously because it is desirable to have it absorb slowly.

2. The nurse is preparing to administer an oral medication and wants to ensure a rapid drug action. Which form of the medication will the nurse administer? a. Capsule b. Enteric-coated pill c. Liquid suspension d. Tablet

ANS: C Liquid drugs are already in solution, which is the form necessary for absorption in the GI tract. The other forms must disintegrate into small particles and then dissolve before being absorbed.

6.Methoxsalen (Oxsoralen) is a drug used for the treatment of psoriasis. Clients using this agent should not be in direct sunlight for which reason? a. Skin could become lightened or blanched. b. Psoriasis would spread. c. Exposed skin would burn or blister. d. Skin would become sensitive to light.

ANS: C Methoxsalen (Oxsoralen) will produce burning or blistering of the skin if the area is exposed to direct sunlight.

1. Which drugs will go through a pharmaceutic phase after it is administered? a. Intramuscular cephalosporins b. Intravenous vasopressors c. Oral analgesics d. Subcutaneous antiglycemics

ANS: C When drugs are administered parenterally, there is no pharmaceutic phase, which occurs when a drug becomes a solution that can cross the biologic membrane.

4. The nurse is caring for a 7-year-old child who has difficulty concentrating and completing tasks and who cannot seem to sit still. Which diagnostic test may be ordered to assist with a diagnosis of attention deficit/hyperactivity disorder (ADHD) in this child? a. Computerized tomography (CT) of the head b. Electrocardiogram (ECG) c. Electroencephalogram (EEG) d. Magnetic resonance imaging (MRI) of the brain

ANS: C A child with ADHD may have abnormal EEG findings. CT, MRI, and ECG tests are not diagnostic for ADHD.

7. A patient who is taking trimethoprim-sulfamethoxazole (TMP-SMX) calls to report developing an all-over rash. The nurse will instruct the patient to perform which action? a. Increase fluid intake. b. Take diphenhydramine. c. Stop taking TMP-SMX immediately. d. Continue taking the medication.

ANS: C A rash can indicate a serious drug reaction. Patients should stop taking the drug immediately and notify the provider.

4. A patient who has seasonal allergies with a runny nose during the daytime reports increasing nighttime symptoms of coughing and sneezing that are interfering with sleep. The provider recommends diphenhydramine (Benadryl) at bedtime. What information will the nurse include when teaching the patient about this medication? a. "Avoid fluids at bedtime to prevent urinary retention." b. "This will help clear your daytime symptoms, too." c. "You should be able to sleep better when you take this medication." d. "You should take this medication on an empty stomach."

ANS: C A side effect of diphenhydramine is drowsiness. Patients whose nighttime symptoms clear should be able to sleep better, especially with drowsiness side effects. Avoiding fluids does not prevent urinary retention. The half-life of diphenhydramine is short, so drug effects will not last through the next day. There is no need to take the medication on an empty stomach.

5. The nurse understands that a medication such as carvedilol (Coreg) may not be effective in an African-American patient because of its effects on a. cardiac contractility. b. heart rate. c. renin release. d. vascular resistance.

ANS: C African Americans are more likely to be susceptible to low-renin hypertension. Beta blockers reduce heart rate, contractility, and renin release, and there is a greater hypotensive response in patients with higher renin levels. Changes in heart rate, contractility, and vascular resistance explain why there is some response in this group.

15. The nurse is caring for a 70-kg patient who is receiving gentamicin (Garamycin) 85 mg 4 times daily. The patient reports experiencing ringing in the ears. The nurse will contact the provider to discuss a. decreasing the dose to 50 mg QID. b. giving the dose 3 times daily. c. obtaining a serum drug level. d. ordering a hearing test.

ANS: C Aminoglycosides can cause ototoxicity. Any changes in hearing should be reported to the provider so that serum drug levels can be monitored. The dose is correct for this patient's weight (5 mg/kg/day in 4 divided doses). A hearing test is not indicated unless changes in hearing persist.

11. A patient who has recently begun taking captopril (Capoten) to treat hypertension calls a clinic to report a persistent cough. The nurse will perform which action? a. Instruct the patient to go to an emergency department because this is a hypersensitivity reaction. b. Reassure the patient that this side effect is nothing to worry about and will diminish over time. c. Schedule an appointment with the provider to discuss changing to an angiotensin II receptor blocker (ARB). d. Tell the patient to stop taking the drug immediately since this is a serious side effect of this drug.

ANS: C An angiotensin-converting enzyme (ACE) inhibitor, such as captopril, can cause a constant, irritated cough. The cough will stop with discontinuation of the drug, and many patients can switch to an ARB medication. It does not indicate a hypersensitivity reaction. The cough will not diminish while still taking the drug. The patient does not need to stop taking the drug immediately.

11. The parent of an obese 10-year-old child asks the nurse about medications to aid in weight loss. Which response by the nurse is correct? a. "Anorexiants are often used to 'jump start' a weight loss regimen in children." b. "Children are able to use over-the-counter anorexiants on a long-term basis." c. "Children under 12 years of age should not use weight loss drugs." d. "Side effects of anorexiants occur less often in children."

ANS: C Anorexiants should not be given to children under age 12 years.

11. A patient will take an anthelmintic medication and asks the nurse about side effects. The nurse will tell the patient that anthelmintic drugs a. can cause hepatic toxicity. b. cause orthostatic hypotension. c. commonly have gastrointestinal (GI) side effects. d. have many serious adverse reactions.

ANS: C Anthelmintic drugs have many GI side effects, including anorexia, nausea, vomiting, diarrhea, and cramps. Adverse reactions do not occur frequently.

4. The nurse is caring for a patient who has myasthenia gravis (MG) and takes pyridostigmine bromide (Mestinon) 60 mg every 4 hours. The patient's last dose was 45 minutes prior. The nurse notes severe muscle weakness, excess salivation, fasciculations of facial muscles, and pupil constriction. The nurse will perform which action? a. Assess the patient for signs of ptosis. b. Notify the provider to discuss an order for intravenous immune globulin (IVIG). c. Obtain an order for atropine sulfate. d. Request an order for an extra dose of pyridostigmine.

ANS: C Severe muscle weakness, excess salivation, fasciculations of facial muscles, and pupil constriction are the major signs of cholinergic crisis, caused by excess pyridostigmine. The antidote is atropine, so the nurse should obtain an order to give this. Ptosis is sign of myasthenic crisis. IVIG is given to treat symptoms of MG and not used for cholinergic crisis. Giving extra pyridostigmine would increase the symptoms.

10. A patient will begin taking simvastatin (Zocor) to decrease serum cholesterol. When teaching the patient about this medication, the nurse will counsel the patient to take which action? a. Return to the clinic annually for laboratory testing. b. Take care when rising from a sitting to standing position. c. Take the medication in the evening for best effect. d. Use ibuprofen as needed for muscle aches and pain.

ANS: C Simvastatin is given in the evening. Laboratory tests are performed every 3 to 6 months, not annually. Statins do not cause postural hypotension. Patients taking statins should report muscle aches and weakness immediately.

18. The nurse is caring for a 6-year-old child who had surgery that morning. The child is awake and lying very still in bed and won't respond when the nurse asks about pain. The nurse will perform which action? a. Ask the child to rate the pain on a scale of 1 to 10. b. Encourage the child to request pain medication when needed. c. Evaluate the child's pain using an "ouch" scale. d. Plan to administer pain medication if the child begins to cry.

ANS: C Some children will not verbalize discomfort even when they have severe pain because they fear injections. Nurses may use an "ouch" scale or a faces scale to evaluate pain if the child won't respond. Waiting for severe pain is not appropriate.

9. A patient is receiving bone marrow transplantation for cancer and receives filgrastim (Neupogen). The patient reports abdominal pain in the left upper quadrant. The nurse will perform which action? a. Administer acetaminophen 650 mg. b. Administer an antiemetic medication. c. Report a potentially life-threatening event. d. Request an order for cardiac enzyme levels.

ANS: C Splenic rupture can occur with this drug and is manifested by pain in the left upper quadrant. The nurse should report the abdominal pain to the provider so the patient can be evaluated for splenic rupture.

3. A female patient who is taking trimethoprim-sulfamethoxazole (TMP-SMZ) (Bactrim, Septra) to treat a urinary tract infection reports vaginal itching and discharge. The nurse will perform which action? a. Ask the patient if she might be pregnant. b. Reassure the patient that this is a normal side effect. c. Report a possible superinfection to the provider. d. Suspect that the patient is having a hematologic reaction.

ANS: C Superinfection can occur with a secondary infection. Vaginal itching and discharge is a sign of superinfection. This is not symptomatic of pregnancy. These are not common side effects and do not indicate a hematologic reaction.

12. The nurse is caring for a patient who is taking trimethoprim-sulfamethoxazole (TMP-SMX). The nurse learns that the patient takes an angiotension-converting enzyme (ACE) inhibitor. To monitor for drug interactions, the nurse will request an order for which laboratory test(s)? a. A complete blood count b. BUN and creatinine c. Electrolytes d. Glucose

ANS: C TMP-SMX can result in hyperkalemia when taken with an ACE inhibitor.

10. The nurse is caring for a patient who is ordered to receive PO trimethoprim-sulfamethoxazole (TMP-SMX) 160/800 QID to treat a urinary tract infection caused by E. coli. The nurse will contact the provider to clarify the correct a. dose. b. drug. c. frequency. d. route.

ANS: C TMP-SMX is taken twice daily. This is the correct dose, drug, and route to treat this condition.

16. The nurse is teaching a family member about an elderly parent's new prescription for tacrine (Cognex) to treat Alzheimer's disease (AD). The family member asks what to expect from this drug. The nurse will respond that the patient will a. demonstrate improved ambulation. b. have reversal of all symptoms. c. have decreased deterioration of cognition. d. show improved communication ability.

ANS: C Tacrine can help to increase cognitive function for patients with mild to moderate AD. For the most part, drugs to treat AD do not result in improvement of symptoms but help slow the progress.

5. The nurse is preparing to administer trimethoprim-sulfamethoxazole (TMP-SMX) to a patient who is being treated for a urinary tract infection. The nurse learns that the patient has type 2 diabetes mellitus and takes a sulfonylurea oral antidiabetic drug. The nurse will monitor this patient closely for which effect? a. Headaches b. Hypertension c. Hypoglycemia d. Superinfection

ANS: C Taking oral antidiabetic agents (sulfonylurea) with sulfonamides increases the hypoglycemic effect. Sulfonylureas do not increase the incidence of headaches, hypertension, or superinfection when taken with sulfonamides. Examples of antidiabetic sulfonylurea medications are glipizide, glimepride, glyburide, tolaamide, and tolbutamide.

1. The nurse is teaching a patient who will begin receiving targeted therapy for cancer. The patient asks how targeted therapy differs from other types of chemotherapies. The nurse will explain that targeted therapy a. damages cancer cell DNA to prevent cell replication. b. directly kills or damages cancerous cells. c. interferes with specific molecules in cancer cells. d. prevents metastasis of cancer cells.

ANS: C Targeted therapy differs from traditional cancer chemotherapy by taking advantage of biologic features particular to cancer cells and targeting specific mechanisms. They block the growth and spread of cancer by interfering with specific molecules within the cancer cells. Traditional chemotherapeutic agents damage cell DNA of cancer cells as well as normal cells. Targeted therapies do not directly kill or damage cancer cells or prevent metastasis.

19. A patient who has Wolff-Parkinson-White syndrome is given intravenous adenosine (Adenocard). The nurse will explain that the medication is effective because it a. controls atrial flutter. b. deepens myocardial excitability. c. prevents multifocal ventricular contractions. d. prolongs repolarization.

ANS: D Adenosine is a class III drug that prolongs repolarization by increasing the refractory period and prolonging the action potential to slow heart rate. It does not control atrial flutter, deepen myocardial excitability, or prevent multifocal contractions.

1. The nurse is performing a medication history on a patient who reports using phentermine HCl (Suprenza) 15 mg/day for the past 3 months as an appetite suppressant. The nurse will contact the patient's provider to discuss a. changing the medication to phentermine-topiramate (Qsymia). b. increasing the dose to 37.5 mg/day since tolerance has likely occurred. c. initiating a slow taper of the phentermine. d. stopping the drug immediately since long-term use is not recommended.

ANS: C The nurse should discuss a gradual taper of the medication with the provider. Patients using anorexiants should not stop taking them abruptly because depression and withdrawal symptoms may occur. Phentermine-topiramate is recommended for short-term use only. Patients should not use these medications longer than 12 weeks, so increasing the dose is not indicated.

12. A nurse is teaching a patient how to use phenylephrine (Neo-Synephrine) nasal spray. To avoid systemic absorption, the nurse teaches the patient to perform which action? a. Apply pressure to the nose after spraying. b. Administer the spray while in the supine position. c. Insert the spray while sitting up. d. Exhale deeply while injecting the nasal spray.

ANS: C The patient should insert the spray while sitting up to avoid it being absorbed systemically.

16. The nurse is teaching a patient about taking colchicine to treat gout. What information will the nurse include when teaching this patient about this drug? a. Avoid all alcohol except beer. b. Include salmon in the diet. c. Increase fluid intake. d. Take on an empty stomach.

ANS: C The patient who is taking colchicine should increase fluid intake to promote uric acid excretion and prevent renal calculi. Foods rich in purine should be avoided, including beer, and some sea foods, such as salmon. Gastric irritation is a common problem, so colchicine should be taken with food.

6. The nurse caring for a patient who has tuberculosis and who is taking isoniazid, rifampin, and streptomycin reviews the medical record and notes the patient's sputum cultures reveal resistance to streptomycin. The nurse will anticipate that the provider will take which action? a. Add ethambutol (Myambutol). b. Change the streptomycin to clarithromycin. c. Change the streptomycin to kanamycin. d. Order renal function tests.

ANS: C The patient's current regimen is first-phase treatment. If resistance to streptomycin develops, the provider can change to kanamycin or to ciprofloxacin. Ethambutol is added if there is resistance to isoniazid. Clarithromycin is used during phase II. Renal function tests are not indicated.

11. The parent of a toddler asks the nurse what can be done to prevent otitis media. What will the nurse recommend? a. Administer diphenhydramine when the child has a runny nose. b. Give phenylephrine (Neo-Synephrine Ophthalmic) to prevent congestion. c. Keep the child's immunizations up to date. d. Remove cerumen with carbamide peroxide (Auro Ear Drops).

ANS: C The pneumococcal conjugate vaccine (PCV) protects children against S. pneumoniae and should be administered to all children to prevent otitis media. Antihistamines and decongestants have been shown to be ineffective in preventing otitis media. Removing cerumen helps to prevent otitis externa.

8. The nurse provides teaching to a patient who will begin taking simvastatin (Zocor) to treat hyperlipidemia. Which statement by the patient indicates understanding of the teaching? a. "I may have diarrhea as a result of taking this medication." b. "I may stop taking this medication when my lipid levels are normal." c. "I will need an annual eye examination while taking this medication." d. "I will increase my intake of vitamins A, D, and E while taking this medication.

ANS: C The statins can affect visual acuity, so patients should be counseled to have annual eye examinations for assessment of cataract formation. The bile acid sequestrants, not statins, cause diarrhea. Statin drug therapy is lifelong or until behavioral changes prove equally effective (uncommon). Bile acid sequestrants, not statins, decrease the absorption of fat-soluble vitamins.

12. The nurse is teaching a patient who is going on a cruise about the use of transdermal scopolamine . What information will the nurse include when teaching this patient? a. "Apply the patch as needed for nausea and vomiting." b. "Apply the patch to your upper arm." c. "Change the patch every 3 days." d. "Restrict fluids while using this patch."

ANS: C The transdermal scopolamine patch is designed to last for 72 hours. The patient should be taught to change it every 3 days. It works best when worn at all times and not just for symptomatic relief. The patch should be applied behind the ear. Patients should not restrict fluids.

6. A patient is receiving the erythropoietin-stimulating agent epoietin alfa (Procrit). Which assessment finding would cause the nurse to notify the patient's provider? a. Blood pressure of 90/65 mm Hg b. Headache and nausea c. Hemoglobin > 12 g/dL d. Infiltration of the IV

ANS: C There is an increased risk of death and serious cardiovascular events when the hemoglobin is greater than 12 g/dL. There is no need to notify the provider of the other findings.

1. The nurse is preparing to administer the first dose of hydrochlorothiazide (HydroDIURIL) 50 mg to a patient who has a blood pressure of 160/95 mm Hg. The nurse notes that the patient had a urine output of 200 mL in the past 12 hours. The nurse will perform which action? a. Administer the medication as ordered. b. Encourage the patient to drink more fluids. c. Hold the medication and request an order for serum BUN and creatinine. d. Request an order for serum electrolytes and administer the medication.

ANS: C Thiazide diuretics are contraindicated in renal failure. This patient has oliguria and should be evaluated for renal failure prior to administration of the diuretic—especially in the absence of known renal failure for this patient. Drinking more fluids will not increase urine output in patients with renal failure.

4. The nurse is caring for a patient who is to begin receiving a thiazide diuretic to treat heart failure. When performing a health history on this patient, the nurse will be concerned about a history of which condition? a. Asthma b. Glaucoma c. Gout d. Hypertension

ANS: C Thiazides block uric acid secretion and elevated levels can contribute to gout. Patients with a history of gout should take thiazide diuretics with caution; they may need behavioral and/or pharmacologic changes to their gout treatment.

14. A patient who takes warfarin (Coumadin) and digoxin (Lanoxin) develops hypothyroidism and will begin taking levothyroxine (Synthroid). The nurse anticipates which potential adjustments in dosing for this patient? a. Decreased digoxin and decreased warfarin b. Decreased digoxin and increased warfarin c. Increased digoxin and decreased warfarin d. Increased digoxin and increased warfarin

ANS: C Thyroid preparations increase the effect of oral anticoagulants, so the warfarin dose may need to be decreased. Levothyroxine can decrease the effectiveness of digoxin, so this dose may need to be increased.

2. The nurse is preparing to administer doses of hydrochlorothiazide (HydroDIURIL) and digoxin (Lanoxin) to a patient who has heart failure. The patient reports having blurred vision. The nurse notes a heart rate of 60 beats per minute and a blood pressure of 140/78 mm Hg. Which action will the nurse take? a. Administer the medications and request an order for serum electrolytes. b. Give both medications and evaluate serum blood glucose frequently. c. Hold the digoxin and notify the provider. d. Hold the hydrochlorothiazide and notify the provider.

ANS: C When thiazide diuretics are taken with digoxin, patients are at risk of digoxin toxicity because thiazides can cause hypokalemia. The patient has bradycardia and blurred vision, which are both signs of digoxin toxicity. The nurse should hold the digoxin and notify the provider. Serum electrolytes may be ordered, but the digoxin should not be given.

3. A patient who has tuberculosis asks the nurse why three drugs are used to treat this disease. The nurse will explain that multi-drug therapy is used to reduce the likelihood of a. disease relapse. b. drug hypersensitivity reactions. c. drug resistance. d. drug adverse effects.

ANS: C Without multi-drug therapy, patients easily develop resistance to antitubercular drugs. Using more than one antitubercular drug does not prevent relapse, hypersensitivity reactions, or adverse effects.

1. A patient with high cholesterol is ordered to take atorvastatin (Lipitor). What information will be included in the patient teaching? (Select all that apply.) a. Dietary management is not a priority with this medication. b. The medication should be taken on an empty stomach. c. The medicine should be taken with a full glass of water. d. The patient should watch for body aches or gastrointestinal upset as side effects. e. The patient should have renal function tests frequently. f. The patient should have liver function tests frequently.

ANS: C, D, F This medication is most effective with careful monitoring of diet. Atorvastatin does not affect renal function.

11.The nurse plans to monitor a client with second-degree burns for which adverse reaction to mafenide acetate (Sulfamylon)? a. Increased intraocular pressure b. Urinary retention c. Fluid retention d. Superinfection

ANS: D Adverse reactions to mafenide acetate (Sulfamylon) include superinfection, respiratory alkalosis, blistering, and metabolic acidosis.

10. The nurse is preparing to administer a drug and learns that it binds to protein at a rate of 90%. The patient's serum albumin level is low. The nurse will observe the patient for a. decreased drug absorption. b. decreased drug interactions. c. decreased drug toxicity. d. increased drug effects.

ANS: D Drugs that are highly protein-bound bind with albumin and other proteins, leaving less free drug in circulation. If a patient has a low albumin, the drug is not bound, and there is more free drug to cause drug effects. There would be increased absorption, increased interactions with other drugs, and increased toxicity.

7.A client enters the healthcare provider's office with complaints of verruca vulgaris (warts). What is the most accurate instructional point to include in a client teaching session? a. Warts can become malignant after 1 to 2 years and thus must be monitored closely. b. The only effective means of wart removal is by surgical excision. c. Drug therapy to remove a common wart may include systemic side effects. d. Electrodesiccation can be used to eradicate the common warts.

ANS: D Electrodesiccation, along with freezing and surgical excision, can be used to treat the common wart.

18. The nurse is preparing to administer a drug that is eliminated through the kidneys. The nurse reviews the patient's chart and notes that the patient has increased serum creatinine and blood urea nitrogen (BUN). The nurse will perform which action? a. Administer the drug as ordered. b. Anticipate a shorter than usual half-life of the drug. c. Expect decreased drug effects when the drug is given. d. Notify the provider and discuss giving a lower dose.

ANS: D Increased creatinine and BUN indicate decreased renal function so a drug that is eliminated through the kidneys can become toxic. The nurse should discuss a lower dose with the provider. The drug will have a longer half-life and will exhibit increased effects with decreased renal function.

21. The nurse is explaining to the patient why a nonspecific drug has so many side effects. Which statement by the patient indicates a need for further teaching? a. "Nonspecific drugs can affect specific receptor types in different body tissues." b. "Nonspecific drugs can affect a variety of receptor types in similar body tissues." c. "Nonspecific drugs can affect hormone secretion as well as cellular functions." d. "Nonspecific drugs require higher doses than specific drugs to be effective."

ANS: D Nonspecific drugs can act on one type of receptor but in different body tissues, or a variety of receptor types, or act on hormones to produce effects. Nonspecific drugs do not require higher doses.

24. The nurse administers a narcotic analgesic to a patient who has been receiving it for 1 day after orthopedic surgery. The patient reports no change in pain 30 minutes after the medication is given. The nurse recognizes that this patient is exhibiting a. drug-seeking behavior. b. drug tolerance. c. the placebo effect. d. tachyphylaxis.

ANS: D Tachyphylaxis is a rapid decrease in response, or acute tolerance. Tolerance to drug effects can occur with narcotics, requiring increased doses in order to achieve adequate drug effects. Nurses often mistake drug-seeking behavior for drug tolerance. The placebo effect occurs when the patient experiences a response with an inactive drug.

22. The nurse is preparing to give a dose of gentamicin to a patient and notes that the most recent serum gentamicin trough level was 2 mcg/mL. What will the nurse do next? a. Administer the drug as ordered. b. Administer the drug and monitor for adverse effects. c. Notify the provider to discuss decreasing the dose. d. Notify the provider to report a toxic drug level.

ANS: D The trough drug level for gentamicin should be less than 2 mcg/mL. The nurse should not administer the drug and should notify the provider of the toxic level.

3. A patient will be discharged home with albuterol (Proventil) to use for asthma symptoms. What information will the nurse include when teaching this patient about this medication? a. Failure to respond to the medication indicates a need for a higher dose. b. Monitor for hypoglycemia symptoms when using this medication. c. Palpitations are common with this drug even at normal, therapeutic doses. d. Overuse of this medication can result in airway narrowing and bronchospasm.

ANS: D Excessive use of an aerosol drug can occasionally cause severe paradoxical airway resistance, so patients should be cautioned against overuse. Excessive use can also lead to tolerance and loss of drug effectiveness, but patients should not increase the dose because of the risk of bronchospasm and the increased incidence of adverse effects such as tremors and tachycardia. Hyperglycemia can occur. Palpitations are common with increased doses but not at therapeutic doses.

5. The nurse is caring for a patient who is receiving intravenous dopamine (Intropin). The nurse notes erythema and swelling at the IV insertion site. What is the nurse's initial action? a. Apply warm soaks to the area. b. Monitor the patient closely for hypertension. c. Obtain an order for an electrocardiogram. d. Notify the provider of a need for phentolamine mesylate (Regitine).

ANS: D Extravasation of dopamine causes tissue necrosis; if extravasation occurs, the antidote phentolamine mesylate should be infiltrated into the area.

12. A patient has been taking famotidine (Pepcid) 20 mg bid to treat an ulcer but continues to have pain. The provider has ordered lansoprazole (Prevacid) 15 mg per day. The patient asks why the new drug is necessary, since it is more expensive. The nurse will explain that lansoprazole a. can be used for long-term therapy. b. does not interact with other drugs. c. has fewer medication side effects. d. is more potent than famotidine.

ANS: D Famotidine is a histamine2 (H2) blocker. When patients fail therapy with these agents, proton pump inhibitors, which can inhibit gastric acid secretion up to 90% greater than the H2 blockers, are used. Lansoprazole is not for long-term treatment and has drug interactions and drug side effects as do all other medications.

4. Which would be a contraindication for hormone therapy with somatropin (Genotropin) in a school-age child? a. Asthma b. Dwarfism c. Enuresis d. Prader-Willi syndrome

ANS: D Fatalities associated with risks of taking growth hormone with Prader-Willi syndrome have been reported, so it is contraindicated in patients with this syndrome. It is not contraindicated in patients with asthma or enuresis. Dwarfism is an indication for hormone therapy.

13. A patient will begin taking the antiepileptic drug ethosuximide (Zarontin) and asks the nurse whether to take the drug with or without food. The nurse will counsel the patient to take this medication a. at bedtime. b. 1 hour before meals. c. 2 hours after meals. d. with meals.

ANS: D Gastric irritation is common with ethosuximide, so patients should be counseled to take it with food. It is given twice daily.

5. A patient is admitted to the hospital, and the provider orders gemfibrozil (Lopid) 600 mg twice daily, 30 minutes prior to meals. The nurse learns that the patient takes warfarin (Coumadin) once daily. The nurse will contact the provider to discuss a. decreasing the dose of gemfibrozil. b. giving the warfarin at noon. c. increasing the dose of warfarin. d. ordering frequent INR levels.

ANS: D Gemfibrozil is highly protein-bound and competes for receptor sites with drugs such as warfarin. The anticoagulant dose should be decreased, and the INR should be closely monitored. Decreasing the dose of gemfibrozil is not recommended. Giving the warfarin at a different time of day does not change this drug interaction. The warfarin dose should be decreased not increased.

14. The nurse is preparing to administer intravenous gentamicin to an infant through an intermittent needle. The nurse notes that the infant has not had a wet diaper for several hours. The nurse will perform which action? a. Administer the medication and give the infant extra oral fluids. b. Contact the provider to request adding intravenous fluids when giving the medication. c. Give the medication and obtain a serum peak drug level 45 minutes after the dose. d. Hold the dose and contact the provider to request a serum trough drug level.

ANS: D Gentamicin can cause nephrotoxicity. When changes in urine output occur, the provider should be notified, and serum trough levels should be obtained to make sure the drug is not at a toxic level. If the drug level is determined to be safe, giving extra fluids either orally or intravenously may be indicated. Serum peak levels give information about therapeutic levels but are not a substitution for avoiding nephrotoxicity in the face of possible oliguria.

14. A patient who has insulin-dependent diabetes mellitus must take a glucocorticoid medication for osteoarthritis. When teaching this patient, the nurse will explain that there may be a need to a. decrease the glucocorticoid dose. b. decrease the insulin dose. c. increase the glucocorticoid dose. d. increase the insulin dose.

ANS: D Glucocorticoids can cause hyperglycemia, so the insulin dose may need to be increased. Changing the glucocorticoid dose is not recommended. Decreasing the insulin dose will only compound the hyperglycemic effects.

22. A patient who takes high-dose aspirin to treat arthritis will need to take prednisone to treat an acute flare of symptoms. What action will the nurse perform? a. Observe the patient for hypoglycemia. b. Monitor closely for increased urine output. c. Observe the patient for hypotension. d. Request an order for enteric-coated aspirin.

ANS: D Glucocorticoids can increase gastric distress, so an enteric-coated aspirin product is indicated. Glucocorticoids increase the risk of hypoglycemia, fluid retention, and hypertension.

2. The nurse is caring for a patient who is receiving growth hormone. Which assessment will the nurse monitor daily? a. Complete blood count b. Height and weight c. Renal function d. Serum glucose

ANS: D Growth hormone antagonizes insulin secretion, so serum glucose should be monitored

13. A patient who uses transdermal nitroglycerin reports having headaches. The nurse will counsel the patient to perform which action? a. Call 911 when this occurs. b. Notify the provider. c. Reapply the patch three times daily. d. Take acetaminophen as needed.

ANS: D Headaches are one of the most common side effects of nitroglycerin, but they may become less frequent; acetaminophen is generally recommended for pain. The headaches are not an emergency, and the patient does not need to call 911 or notify the provider. The patch is applied once daily.

7. The nurse is teaching a patient who will begin taking warfarin (Coumadin) for atrial fibrillation. Which statement by the patient indicates understanding of the teaching? a. "I should eat plenty of green, leafy vegetables while taking this drug." b. "I should take a nonsteroidal anti-inflammatory drug (NSAID) instead of acetaminophen for pain or fever." c. "I will take cimetidine (Tagamet) to prevent gastric irritation and bleeding." d. "I will tell my dentist that I am taking this medication."

ANS: D Patients taking warfarin should tell their dentists that they are taking the medication because of the increased risk for bleeding. Patients should avoid foods high in vitamin K, which can decrease the effects of warfarin. Patients should not take NSAIDs or cimetidine (Tagamet) because they can displace warfarin from protein-binding sites.

15. Which statement by a patient who will begin using an external insulin pump indicates understanding of this device? a. "I will have an increased risk for hypoglycemia." b. "I will leave this on when bathing or swimming." c. "I will not need to count carbohydrates anymore." d. "I will still need to monitor serum glucose."

ANS: D Patients using an insulin pump will still monitor serum glucose and count carbohydrates. The advantage of the pump is that it is programmed to deliver continuous rapid-acting insulin in varying amounts at different times throughout the day. Changes in food intake can alter the risk for hypoglycemia if the pump is not adjusted accordingly. They must be removed when bathing or swimming.

8. The nurse is providing teaching for a patient who will begin using tobramycin ointment (Nebcin) 0.5 inches 3 times daily. The patient currently uses pilocarpine HCl (Isopto Carpine) drops to treat glaucoma. Which statement by the patient indicates a need for further teaching? a. "I should apply the third dose of tobramycin at bedtime each day." b. "I should instill the drops in the conjunctival sac of the lower eyelid." c. "I should not stop the medications without consulting my provider." d. "I should put the ointment on first and then instill the eyedrops."

ANS: D Patients using both drops and ointments should instill the drops prior to applying the ointment. Ointments should be applied at bedtime if possible. Drops should be instilled into the conjunctival sac of the lower lid. Patients should always consult with their provider before discontinuing any medication.

20. The nurse provides teaching for a patient who has a ventricular dysrhythmia who is prescribed acebutolol (Spectral) 200 mg twice daily. Which statement by the patient indicates understanding of the teaching? a. "Diuretics may decrease the effectiveness of this drug." b. "Dizziness, nausea, and vomiting indicate a severe reaction." c. "I should eat fruits and vegetables to increase potassium intake." d. "I should not stop taking this drug abruptly to avoid palpitations."

ANS: D Patients who stop taking this drug abruptly can experience palpitations. Diuretics do not decrease drug effectiveness. Dizziness and nausea and vomiting are common side effects.

10. The nurse is teaching a patient who has hypertension about long-term management of the disease and a beta blocker. The patient reports typically consuming 1 to 2 glasses of wine each evening with meals. How will the nurse respond? a. "Beta blockers and wine cause a reflex hypertension." b. "Four to 6 ounces of wine is considered safe with these medications." c. "Wine in moderation helps you relax and get better blood pressure control." d. "Wine increases the hypotensive effects of the beta blocker."

ANS: D Patients who take beta blockers should avoid all alcohol because it increases the hypotensive effects. It does not cause reflex hypertension.

3. A patient who is overweight is being evaluated for diabetes. The patient has a blood glucose level of 160 mg/dL and a hemoglobin A1c of 5.8%. The nurse understands that this patient has which condition? a. Diabetes mellitus b. Hypoglycemia c. Normal blood levels d. Prediabetes

ANS: D Patients with a hemoglobin A1c between 5.7% and 6.4% are considered to have prediabetes. A level of 6.5% or more indicates diabetes. The patient is hyperglycemic.

12. A patient asks the nurse about using loperamide (Imodium) to treat infectious diarrhea. Which response will the nurse give? a. "Loperamide results in many central nervous system (CNS) side effects." b. "Loperamide has no effect on infectious diarrhea." c. "Loperamide is taken once daily." d. "Loperamide may prolong the symptoms."

ANS: D Patients with infectious diarrhea should be cautioned about using loperamide since slowing transit through the intestines may prolong the exposure to the infectious agent. Loperamide causes less CNS depression than other antidiarrheals. It is taken after each loose stool.

16. The patient will begin taking penicillin G procaine (Wycillin).The nurse notes that the solution is milky in color. What action will the nurse take? a. Call the pharmacist and report the milky color. b. Add normal saline to dilute the medication. c. Call the physician and report the milky appearance. d. Administer the medication as ordered by the physician.

ANS: D Penicillin G procaine (Wycillin) has a milky appearance; therefore, the appearance should not concern the nurse.

7. A patient who is taking isoniazid (INH) as part of a two-drug tuberculosis treatment regimen reports tingling of the fingers and toes. The nurse will recommend discussing which treatment with the provider? a. Adding pyrazinamide b. Changing to ethambutol c. Increasing oral fluid intake d. Taking pyridoxine (B6)

ANS: D Peripheral neuropathy is an adverse reaction to INH, so pyridoxine is usually given to prevent this. It is not necessary to change medications. Increasing fluids will not help with this.

1. Which person should be treated with prophylactic antitubercular medication? a. A child who attends the same school with a child who has tuberculosis b. A nurse who is working in a hospital c. An individual who is HIV-positive with a negative TB skin test d. A patient who has close contact with someone who has tuberculosis

ANS: D Personal contact with a person having a diagnosis of tuberculosis is required to indicate prophylactic treatment with antitubercular therapy. Attending the same school does not necessarily mean close contact occurs. Health care professionals do not need prophylactic treatment. HIV-positive individuals with negative TB skin tests do not need prophylaxis.

9. A patient who has heart failure receives digoxin (Lanoxin) and an angiotension-converting enzyme (ACE) inhibitor. The patient will begin taking spironolactone (Aldactone). The patient asks why the new drug is necessary. The nurse will tell the patient that spironolactone will be given for which reason? a. To enhance potassium excretion b. To increase cardiac contractility c. To minimize fluid losses d. To provide cardioprotective effects

ANS: D Spironolactone is a potassium-sparing diuretic that blocks production of aldosterone, causing improved heart rate variability and decreased myocardial fibrosis. It is given in congestive heart failure for its cardioprotective effects. Spironolactone does not directly alter cardiac contractility but may slightly decrease contractility if fluid volume is decreased. It is a mild diuretic but is not given in this instance to minimize fluid losses.

8. The emergency department nurse is caring for a patient who has received morphine sulfate for severe pain following an injury. The nurse performs a drug history and learns that the patient takes St. John's wort for symptoms of depression. The nurse will observe this patient closely for an increase in which opioid adverse effect? a. Constipation b. Pruritis c. Respiratory depression d. Sedation

ANS: D St. John's wort can increase the sedative effects of opioids. It does not enhance other side effects.

6. The nurse is preparing to give a dose of trimethoprim-sulfamethoxazole (TMP-SMX) and learns that the patient takes warfarin (Coumadin). The nurse will request an order for a. a decreased dose of TMP-SMX. b. a different antibiotic. c. an increased dose of warfarin. d. coagulation studies.

ANS: D Sulfonamides can increase the anticoagulant effects of warfarin. The nurse should request INR levels. An increased dose of warfarin would likely lead to toxicity and to undesirable anticoagulation.

17. The patient has been ordered to receive Sudafed to treat nasal congestion. The nurse performing an admission assessment learns that the patient has diabetes mellitus. What action is appropriate for the nurse to take? a. Administer the medication as ordered. b. Contact the provider to discuss a lower dose. c. Give the medication and monitor serum glucose closely. d. Hold the medication and contact the provider.

ANS: D Sympathetic drugs should not be taken by patients with diabetes. The medication should not be given.

12. The nurse is teaching a patient who has Parkinson's disease about the side effects of carbidopa-levodopa. Which statement by the patient indicates a need for further teaching? a. "I may experience urinary retention, dry mouth, and constipation." b. "I may feel dizzy at first, but this side effect will go away with time." c. "I should report nightmares and mental disturbances to my provider." d. "I should take the drug with food to increase absorption."

ANS: D Taking carbidopa-levodopa with food decreases absorption of the drug, although gastrointestinal distress may decrease when the medication is taken with food. Cholinergic side effects are common. Orthostatic hypotension occurs early and will resolve over time. Nightmares and mental disturbances should be reported.

15. A patient who has Parkinson's disease will begin treatment with benztropine (Cogentin). Which symptom of Parkinson's disease would be a contraindication for this drug? a. Drooling b. Muscle rigidity c. Muscle weakness d. Tardive dyskinesia

ANS: D Tardive dyskinesia is a contraindication for this drug.

11. The nurse is caring for a 7-year-old patient who will receive oral antibiotics. Which antibiotic order will the nurse question for this patient? a. Azithromycin (Zithromax) b. Clarithromycin (Biaxin) c. Clindamycin (Cleocin) d. Tetracycline (Sumycin)

ANS: D Tetracyclines should not be given to children younger than 8 years of age because they irreversibly discolor the permanent teeth.

1. A female patient has serum lipid levels performed, which reveal a total cholesterol of 285 mg/dL, triglycerides of 188 mg/dL, a low-density lipoprotein (LDL) of 175 mg/dL, and a high-density lipoprotein (HDL) of 40 mg/dL. The patient's blood pressure is 138/72 mm Hg. The nurse may expect the provider to order which medication for this patient? a. Amlodipine and atorvastatin (Caduet) b. Colestipol HCl (Colestid) c. Fenofibrate (TriCor) d. Niacin and lovastatin (Advicor)

ANS: D The combination drug of niacin and lovastatin is indicated for hypercholesterolemia and mixed dyslipidemia. Niacin raises HDL, so would be helpful in this patient who has low HDL. Combination drugs are used to enhance the antihyperlipidemic effect. Amlodipine and atorvastatin in combination are used for patients with hyperlipidemia and elevated blood pressure. Colestipol HCl is used to reduce cholesterol and LDL levels but has no effect on HDL or triglycerides. Fenofibrate is used to treat type IV and V hyperlipidemia, characterized by elevated very-low-density lipoprotein and triglycerides.

13. A female patient will receive doxycycline to treat a sexually transmitted illness (STI). What information will the nurse include when teaching this patient about this medication? a. Nausea and vomiting are uncommon adverse effects. b. The drug may cause possible teratogenic effects. c. Increase intake of dairy products with each dose of this medication. d. Use a backup method of contraception if taking oral contraceptives.

ANS: D The desired action of oral contraceptives can be lessened when taken with tetracyclines, so patients taking oral contraceptives should be advised to use a backup contraception method while taking tetracyclines. Nausea and vomiting are common adverse effects. Doxycycline should not be taken with dairy products. Tetracycline may cause teratogenic effects.

6. The nurse is caring for a patient who is receiving furosemide (Lasix) and an aminoglycoside antibiotic. The nurse will be most concerned if the patient reports which symptom? a. Dizziness b. Dysuria c. Nausea d. Tinnitus

ANS: D The interaction of furosemide and an aminoglycoside can produce ototoxicity in the patient. Tinnitus is a sign of ototoxicity. Dizziness can occur as a result of diuretic therapy but not necessarily as a result of this combination. Dysuria and nausea are not common signs of these drugs interacting.

9. An older adult patient reports frequent nighttime awakening because of arthritis pain and asks the nurse about taking an over-the-counter product to help with this problem. The nurse will recommend that the patient discuss which medication with the provider? a. Ibuprofen (Motrin) b. Nytol c. Sominex d. Tylenol PM

ANS: D The main sleep problem experienced by older adults is frequent nighttime awakening. To alleviate pain and aid sleep, the OTC drug Tylenol PM, which contains diphenhydramine and acetaminophen may be taken. Ibuprofen occasionally helps if it can alleviate the discomfort that hinders sleep.

1. The nurse is evaluating a patient 2 hours after giving a dose of 30 mg of codeine with acetaminophen for postoperative pain after abdominal surgery. The patient reports a pain level of 7 on a scale of 1 to 10. The nurse notes a heart rate of 110 beats per minute, a respiratory rate of 28 breaths per minute, and a blood pressure of 180/90 mm Hg. Which action will the nurse take? a. Administer the next dose of codeine one hour early. b. Ask the provider if the codeine dose can be increased. c. Contact the provider to ask if a dose of ibuprofen may be given now. d. Request an order for oxycodone with acetaminophen (Percocet).

ANS: D The patient is showing signs of moderate to severe pain unrelieved by codeine, so the nurse should request a more potent opioid analgesic such as oxycodone. Codeine is effective for mild to moderate pain so will not be effective for this patient even if the dose is increased. The medication should not be given more frequently than every 4 hours. Ibuprofen is used for musculoskeletal pain and not postoperative pain.

3. A patient arrives in the emergency department after developing a rash, runny nose, and sneezing after eating strawberries. What action will the nurse expect to take first? a. Administer diphenhydramine. b. Administer epinephrine. c. Assess for urinary retention. d. Assess heart rate, respiratory rate, and lung sounds.

ANS: D The patient probably has a food allergy, since eating strawberries is the precipitating event. The nurse should assess cardiac and respiratory status to determine whether the patient is developing an anaphylactic reaction. Diphenhydramine will be given for mild allergic symptoms of rash, runny nose, and sneezing, but epinephrine must be given for anaphylaxis. Urinary retention is a side effect of diphenhydramine and will be assessed if diphenhydramine is given.

11. The nurse is preparing to give a dose of a cephalosporin medication to a patient who has been receiving the antibiotic for 2 weeks. The nurse notes ulcers on the patient's tongue and buccal mucosa. Which action will the nurse take? a. Hold the drug and notify the provider. b. Obtain an order to culture the oral lesions. c. Gather emergency equipment to prepare for anaphylaxis. d. Report a possible superinfection side effect of the cephalosporin.

ANS: D The patient's symptoms may indicate a superinfection and should be reported to the physician so it can be treated; however, the drug does not need to be held. It is not necessary to culture the lesions. The symptoms do not indicate impending anaphylaxis.

9. The nurse is caring for a patient who has experienced head trauma in a motor vehicle accident. The patient is having excessive output of dilute urine. The nurse will notify the provider and will anticipate administering which medication? a. Calcifediol (Calderol) b. Corticotropin (Acthar) c. Prednisolone (AK-Pred) d. Vasopressin (Pitressin)

ANS: D The posterior pituitary gland secretes antidiuretic hormone (ADH) (vasopressin). When there is a deficiency of ADH, sometimes caused by head trauma, patients excrete large amounts of dilute urine. ADH replacement is necessary to prevent fluid imbalance. Calcifediol is used to treat parathyroid disorders. Corticotropin and prednisolone do not prevent diuresis.

14. A patient is ordered to receive a nitrate to relieve stable angina. What side effect(s) will the nurse anticipate in a patient receiving this medication? a. Nausea and vomiting b. Increased blood pressure c. Pruritus and skin rash d. Pounding headache

ANS: D The pounding headache is related to vasodilation of the cerebral vessels. Nitrates decrease blood pressure.

6. The nurse is preparing to administer an antibiotic to a patient who has been receiving the antibiotic for 2 days after a culture was obtained. The nurse notes increased erythema and swelling, and the patient has a persistent high fever of 39° C. What is the nurse's next action? a. Administer the antibiotic as ordered. b. Contact the provider to request another culture. c. Discuss the need to add a second antibiotic with the provider. d. Review the sensitivity results from the patient's culture.

ANS: D The sensitivity results from the patient's culture will reveal whether the organism is sensitive or resistant to a particular antibiotic. The patient is not responding to the antibiotic being given, so the antibiotic should be held and the provider notified. Another culture is not indicated. Antibiotics should be added only when indicated by the sensitivity.

15. The nurse performs a preoperative assessment on a patient and asks about alcohol use. The patient asks why this information is important. The nurse will explain that patients who consume increased amounts of alcohol a. may have a prolonged postoperative recovery time. b. may not be eligible for surgery. c. may not receive inhaled gases for anesthesia. d. may require changes in anesthesia drug doses.

ANS: D The type and amount of anesthetics may need to be adjusted if patients consume large amounts of alcohol as well as for those who smoke, who are pregnant, or who are obese. These questions are asked prior to surgery so providers can plan for this.

11. The nurse is caring for a patient who is taking hydrochlorothiazide (HydroDIURIL) and digoxin (Lanoxin). Which potential electrolyte imbalance will the nurse monitor for in this patient? a. Hypermagnesemia b. Hypernatremia c. Hypocalcemia d. Hypokalemia

ANS: D Thiazide diuretics can cause hypokalemia, which enhances the effects of digoxin and can lead to digoxin toxicity. Thiazides can cause hypercalcemia.

12. The nurse provides teaching to a patient who will begin taking cyclobenzaprine (Flexeril) to treat muscle spasms. Which statement by the patient indicates a need for further teaching? a. "I may experience dizziness and drowsiness when I take this drug." b. "I should not consume alcohol while taking this medication." c. "I should take this medication with food to decrease stomach upset." d. "I will take this medication for three weeks and then stop taking it."

ANS: D This medication should not be stopped abruptly. Patients may experience dizziness and drowsiness. Alcohol will compound the central nervous system sedative effects. To decrease gastrointestinal upset, the nurse should counsel the patient to take it with food.

11. A patient who has been taking ranitidine (Zantac) continues to have pain associated with peptic ulcer. A noninvasive breath test is negative. Which treatment does the nurse expect the provider to order for this patient? a. Adding an over-the-counter antacid to the patient's drug regimen b. A dual drug therapy regimen c. Amoxicillin (Amoxil), clarithromycin (Biaxin), and omeprazole (Prilosec) d. Lansoprazole (Prevacid) instead of ranitidine

ANS: D This patient does not have H. pylori ulcer disease, so dual and triple drug therapy with antibiotics is not indicated. Patients who fail treatment with a histamine2 blocker should be changed to a proton pump inhibitor (PPI) such as lansoprazole. PPIs tend to inhibit gastric acid secretion up to 90% greater than the histamine antagonists.

7. A nursing student asks the nurse why patients who have parkinsonism receive a combination of carbidopa and levodopa. The nurse will explain that the combination product a. allows larger doses of levodopa to be given without causing increased adverse reactions. b. causes more levodopa to be converted to dopamine before crossing the blood-brain barrier. c. eliminates almost all drug side effects of both levodopa and carbidopa. d. reduces peripheral side effects by inhibiting decarboxylase in the peripheral nervous system.

ANS: D Without carbidopa, about 99% of levodopa is converted to dopamine before crossing the blood-brain barrier, causing peripheral adverse effects. When carbidopa is added, the enzyme decarboxylase is inhibited, allowing levodopa to cross into the brain before being converted to the active metabolite dopamine. The result is less levodopa required to achieve the desired effect. The drug still has many side effects, but the peripheral effects are lessened.

3. A patient has been receiving intravenous heparin. When laboratory tests are drawn, the nurse has difficulty stopping bleeding at the puncture site. The patient has bloody stools and is reporting abdominal pain.The nurse notes elevated partial thromboplastin time (PTT) and activated partial thromboplastin time (aPTT). Which action will the nurse perform? a. Ask for an order for oral warfarin (Coumadin). b. Obtain an order for protamine sulfate. c. Request an order for vitamin K. d. Suggest that the patient receive subcutaneous heparin.

Protamine sulfate is given as an antidote to heparin when patient's clotting times are elevated. Oral warfarin will not stop the anticoagulant effects of heparin. Vitamin K is used as an antidote for warfarin. Administering heparin by another route is not indicated when there is a need to reverse the effects of heparin.


Ensembles d'études connexes

Chapter 1 Quiz: What is Philosophy?

View Set

Mosby questions Quiz 401 Critical Respiratory

View Set

The Muscular System Chapter 10 Review

View Set

Chapter 15: Special Senses Study Guide

View Set

Chapter 16 review (World History)

View Set

Reading and Writing Text and Discourse

View Set